33994637 labor rounds 1and2

110
Labor Law 2 A2010 - 1 - Disini BACKGROUND MATERIAL FROM COMPULSORY ARBITRATION TO COLLECTIVE BARGAINING IN THE PHILIPPINES Cicero Calderon 2 Contrasting Approaches to the Regulation of Labor Relations 1. System of Compulsory Arbitration (1936) 2. Collective Bargaining COMPULSORY ARBITRATION Factors that Contributed to its Adoption - On the basis of Art. 2, sec. 5 and Art. 14, sec. 6 of the 1935 Constitution, the National Assembly of the Philippines in 1936, adopted compulsory arbitration by providing for a Court of Industrial Relation under C.A. No. 103 - The adoption of compulsory arbitration was a deliberate response of the policy-makers to the social scene characterized by acute agrarian and industrial unrest of disturbing proportions. Other factors contributing to the adoption were: 1. the poor state of organization of the workers and farm workers and the resultant 2. lack of effective collective bargaining 3. the customary reliance of the Filipino upon courts of justice in the adjudication of controversies of whatever nature 3. the desire to experiment with compulsory arbitration which had been rejected at least 3 times under American Rule 4. the apparent inadequacy or effectiveness of the Government’s policy of non-intervention in the area of labor management relations 5. the strong influence of the then President, Manuel L. Quezon who advocated its adoption before the National Assembly Court of Industrial Relation - The CIR was organized at the close of 1936 but started to function only in 1937. - The Court’s docket started to clog in 1947. This number rose very rapidly on the succeeding 6 years, from 137 in 1948 to over 1, 500 in 1953. - The adjudication of labor disputes was being delayed and that the main machinery to enforce the system of compulsory arbitration was unable, or was no longer able, to cope adequately and effectively with h responsibilities placed upon it. - Major factors responsible for the accumulation of cases: 1. Lack of adequate support from the Government itself 2. Consequent inefficiency of the Court 3. The extremely generous discretionary powers vested in the Court to grant continuances, modify its orders and hear motions for reconsideration (sitting as full court) and to other aspects of its procedure. Weaknesses of the System of Compulsory Arbitration 1. There were varied concepts of what constituted “fair and just wage,” “just or unjust cause for dismissal,” public interest,” “social justice” and other technical abstractions. Under such conditions instability and confusion characterized labor management relations. 2. A judicial approach was adopted to the settlement of labor disputes, with the result that policy considerations and economic factors played a less important role than technicalities of law. 3. Excessive dependence on the CIR. 4. The system of compulsory arbitration and the law failed to provide adequate protection for the exercise of the right of self-organization by the workers. 5. The Supreme Court and the CIR failed to perceive that the best means of protecting the workers was to help them build strong unions free from company domination. 6. The legal provisions on strikes and the wide latitude given to the CIR in exercising powers to issue injunctions made it difficult to use strikes and other concerted activities, or the threat of them, as a means of improving the bargaining position of labor. 7. The CIR found that not all differences between workers and employers could be proper subjects of compulsory arbitration. 8. The failure of the CIR to resolve conflicting demands for recognition of unions for collective bargaining purposes did not lead to stability in labor management relations. An Evaluation of the System of Compulsory Arbitration - The basic objectives underlying the adoption of compulsory arbitration 1. To afford protection to labor because of the great disparity in bargaining power - On the whole, terms and conditions of employment were improved for those workers who had the courage and staying power to fight their case before the CIR. But its effectiveness in affording protection to labor was greatly reduced when disputes dragged out in interminable litigation and the Government failed to meet the increased demand for the Court’s services. 2. Stability in Labor Management Relations - Labor management greatly improved for a while. The effectiveness of the system in promoting stability of labor-management relations was greatly reduced, however, during the post-war years. - Under compulsory arbitration an interesting phenomenon developed: the emergence of a labor movement under the leadership of lawyers. 3. Stability of Society in General - The Cir appeared effective in playing the role from its organization until the Philippines were engulfed in the Second World War in 1941. - It was unable to play the role ineffectively during the post-war years. This failure contributed

Upload: bulayug

Post on 10-Apr-2015

101 views

Category:

Documents


1 download

TRANSCRIPT

Page 1: 33994637 Labor Rounds 1and2

Labor Law 2 A2010 - 1 - DisiniBACKGROUND MATERIAL

FROM COMPULSORY ARBITRATION TO COLLECTIVE BARGAINING IN THE PHILIPPINES

Cicero Calderon

2 Contrasting Approaches to the Regulation of Labor Relations 1. System of Compulsory Arbitration (1936) 2. Collective Bargaining

COMPULSORY ARBITRATION

Factors that Contributed to its Adoption - On the basis of Art. 2, sec. 5 and Art. 14, sec. 6 of the 1935 Constitution, the National Assembly of the Philippines in 1936, adopted compulsory arbitration by providing for a Court of Industrial Relation under C.A. No. 103 - The adoption of compulsory arbitration was a deliberate response of the policy-makers to the social scene characterized by acute agrarian and industrial unrest of disturbing proportions. Other factors contributing to the adoption were: 1. the poor state of organization of the workers and farm workers and the resultant 2. lack of effective collective bargaining 3. the customary reliance of the Filipino upon courts of justice in the adjudication of controversies of whatever nature 3. the desire to experiment with compulsory arbitration which had been rejected at least 3 times under American Rule 4. the apparent inadequacy or effectiveness of the Government’s policy of non-intervention in the area of labor management relations 5. the strong influence of the then President, Manuel L. Quezon who advocated its adoption before the National Assembly

Court of Industrial Relation- The CIR was organized at the close of 1936 but started to function only in 1937. - The Court’s docket started to clog in 1947. This number rose very rapidly on the succeeding 6 years, from 137 in 1948 to over 1, 500 in 1953. - The adjudication of labor disputes was being delayed and that the main machinery to enforce the system of compulsory arbitration was unable, or was no longer able, to cope adequately and effectively with h responsibilities placed upon it. - Major factors responsible for the accumulation of cases:1. Lack of adequate support from the Government itself2. Consequent inefficiency of the Court 3. The extremely generous discretionary powers vested in the Court to grant continuances, modify its orders and hear motions for reconsideration (sitting as full court) and to other aspects of its procedure.

Weaknesses of the System of Compulsory Arbitration 1. There were varied concepts of what constituted “fair and just wage,” “just or unjust cause for dismissal,” public interest,” “social justice” and other technical abstractions. Under such conditions instability and confusion characterized labor management relations. 2. A judicial approach was adopted to the settlement of labor disputes, with the result that policy considerations and economic factors played a less important role than technicalities of law. 3. Excessive dependence on the CIR. 4. The system of compulsory arbitration and the law failed to provide adequate protection for the exercise of the right of self-organization by the workers. 5. The Supreme Court and the CIR failed to perceive that the best means of protecting the workers was to help them build strong unions free from company domination. 6. The legal provisions on strikes and the wide latitude given to the CIR in exercising powers to issue injunctions made it difficult to use strikes and other concerted activities, or the threat of them, as a means of improving the bargaining position of labor. 7. The CIR found that not all differences between workers and employers could be proper subjects of compulsory arbitration.8. The failure of the CIR to resolve conflicting demands for recognition of unions for collective bargaining purposes did not lead to stability in labor management relations.

An Evaluation of the System of Compulsory Arbitration - The basic objectives underlying the adoption of compulsory arbitration 1. To afford protection to labor because of the great disparity in bargaining power - On the whole, terms and conditions of employment were improved for those workers who had the courage and staying power to fight their case before the CIR. But its effectiveness in affording protection to labor was greatly reduced when disputes dragged out in interminable litigation and the Government failed to meet the increased demand for the Court’s services. 2. Stability in Labor Management Relations - Labor management greatly improved for a while. The effectiveness of the system in promoting stability of labor-management relations was greatly reduced, however, during the post-war years. - Under compulsory arbitration an interesting phenomenon developed: the emergence of a labor movement under the leadership of lawyers.

3. Stability of Society in General - The Cir appeared effective in playing the role from its organization until the Philippines were engulfed in the Second World War in 1941. - It was unable to play the role ineffectively during the post-war years. This failure contributed to no small measure to the resurgence of agrarian unrest and strikes in the country.

THE ADOPTION OF COLLECTIVE BARGAINING- Collective bargaining was adopted as the labor relations policy of the Philippines on June 17, 1953, when R.A. No. 875 was approved. - Its adoption can be attributed to several factors the most significant of which were: 1. The rising discontent of labor groups with the operation of the CIR because of protracted delays in the adjudication cases. 2. The exposure of Filipino labor leaders to concepts of collective bargaining resulting either from contacts with foreign labor leaders visiting the Philippines or from visits made to other countries by several of the Filipino labor leaders3. The participation by the Philippines in the adoption of 2 important Conventions by the International .Labor Conference namely the Freedom of Association and Protection of the Right to Organise Convention, 1948, and the Right to Organise and Collective Bargaining Convention, 1949. 4. The influence of experience in the United States under the National Labor Relations Act of 1953 ( Wagner Act), which greatly encouraged the development of trade unions and collective bargaining in that country and the enactment of the Labor Management Relations Act of 1947 (Taft-Hartley).

Basic Features of the Act of 1953 1. Registration of Labor Organizations - to obtain the status of a legitimate labor organization and to avail itself of the right to be certified as the exclusive bargaining representative in an appropriate bargaining unit and act as such – a labor organization, association or union must register with the Department of Labor by filing with the Office of the Secretary of Labor notice of its organization and existence. 2. Certification Election - When a number of union seeks to bargain for a particular group of workers or when the employer doubts the claim of a particular union to be representative of the majority of the workers in his factory or establishment, the CIR shall order the Department of Labor to conduct a certification election by secret ballot, either on its own initiative or when it is petitioned to do so by the employer or by at least 10% of the employees in the appropriate unit. 3. Collective Bargaining Process - In recognition “the real industrial peace cannot be achieved by compulsion of law” and that sound stable industrial relations must rest, in keeping with the spirit of… democratic institutions, on an essentially voluntary basis” the Act provides that “no court of the Philippines shall have the power to set wages, rates of pay, hours of employment or conditions of employment except as in this Act is otherwise provided.” 4. Unfair Labor Practice - The Act prohibits certain practices on the part of employers and trade unions which it characterizes as “unfair labor practices.” 5. Procedure for the Prevention of Unfair Labor Practices - The Act gives the CIR, following a special procedure, the power to prevent unfair labor practices, with exclusive jurisdiction to hear all complaints relating thereto. 6. Labor Disputes in Industries Indispensable to the National Interest - In cases of labor disputes in industries indispensable to the national interest and certified as such by the President of the Philippines to the CIR, the Court is empowered by the Act to issue restraining orders enjoining an employer from locking out his employees and employees from striking. 7. The Labor Injunction - The Act provides that, except in labor disputes affecting the national, no restraining order or temporary or permanent injunction shall be issued in any case involving or growing out of a labor dispute except to restrain the commission of violence, fraud and other illegal acts committed in the course of the labor dispute. 8. Role of State Agencies - Though the Industrial Peace Act was designed to reduce the role of the State in the field of labor-management relations to a minimum, it also recognizes that the State cannot merely perform a passive role. 2 state agencies play an important part in the collective bargaining process, namely the Court of Industrial Relations and the Department of Labor and are charged with various functions under the Act.

Effects of the New Policy - Increase in the number of trade unions- Development of a more cohesive labor movement - Emergence of union leadership from the rank and file - Collective bargaining agreements - Labor disputes

Difficulties Encountered - Ease of registering unions

Page 2: 33994637 Labor Rounds 1and2

Labor Law 2 A2010 - 2 - Disini- Delay in determination of unfair labor practice cases- Delay in hearing petitions for certification elections - Excessive use of injunctions in labor disputes- Failure to adopt voluntary arbitration as a method of settling disputes.

Possible Avenues of Improvement - Requiring the officers of a union applying for registration to make a sworn statement that the union is free from employer support, control or influence. - The CIR should be given exclusive power to issue labor injunctions and conduct certification elections. The appeals to the Supreme Court should not be allowed in certification election cases. - The CIR should not be limited to granting of relief in the form of “cease and desist” orders or reinstatement with or without back pay in cases where workers have been dismissed for trade union membership or activity; it should be empowered to award damages in proper cases. - The Labor Management Advisory Council of the Department of Labor should bring the various Chambers of Commerce and the major labor \federations together to discuss the question of voluntary arbitration as a final step in the grievance machinery and a substitute for direct action in the enforcement of collective bargaining and assume responsibility for the adoption of appropriate measures for its general introduction. - The Conciliation Service should be provided with enough trained conciliators to ensure that the process of collective bargaining will operate smoothly. - Backing these measures is the program of workers’ education.

Conclusion - 3 important stages of development of the labor relations policy of the Philippines Government: 1. Before 1936 – pattern of non-interference on the part of the Government 2. As early as 1933 – when the world found itself in the throes of economic depression, Frank Murphy, stated before the Legislature that “ a responsibility rests upon government for control and guidance in a field that was formerly left to the automatic regulation of self-interest and individual ambition” and the policy of non-interference was completely abandoned when compulsory arbitration was adopted under C.A. 103. 3. Re-examination of policy and the adoption of collective bargaining under the Industrial Peace Act of 1953 reflected as new and new objectives.

I. INTRODUCTORY MATERIALS1.01 STATUTORY BASIS

B.F. GOODRICH PHILIPPINES, INC. vs. B.F. GOODRICH (MARIKINA FACTORY) CONFIDENTIAL & SALARIED EMPLOYEES UNION-NATU, B.F. GOODRICH (MAKATI OFFICE) CONFIDENTIAL &

SALARIED EMPLOYEES UNION-NATU, and COURT OF INDUSTRIAL RELATIONS

49 SCRA 532FERNANDO; February 28, 1973

Facts:The Goodrich Unions were seeking to be recognized as the bargaining agent of BF Goodrich Phils’ employees so that there could negotiations for a collective contract. BF Goodrich countered this by filing for two petitions for certification election with the CIR. Strike notices were sent to the company by the union demanding recognition and soon after a strike was actually held. The company then filed a case of illegal strike and unfair labor practice against the unions.

ISSUE:WON the determination of an unfair labor practice case, brought against unions, must precede the holding of a certification election

HELD: No.

REASONING:- If under the circumstances disclosed, management is allowed to have its way, the

result might be to dilute or fritter away the strength of an organization bent on a more zealous defense of labor's prerogatives.

- This is not to say that management is to be precluded from filing an unfair labor practice case. It is merely to stress that such a suit should not be allowed to lend itself as a means, whether intended or not, to prevent a truly free expression of the will of the labor group as to the organization that will represent it.

- There is no valid reason then for the postponement sought. This is one instance that calls for the application of the maxim, lex dilationes semper exhorret. (The law abhors delays.)

- The law clearly contemplates all the employees, not only some of them, to take part in the certification election. (Some of the employees could possibly lose such status, by virtue of a pending unfair labor practice case, if such case is to be resolved first before the election.)

- Another reason (re: no point in the postponement of said election) is that even if the company wins in the pending case, it does not mean that the employees involved automatically would lose their jobs making them ineligible to participate in the cert. election. (Ergo the respondent court decided in the negative.)

- Besides, it was said in General Maritime Stevedores' Union v. South Sea Shipping Line: the question of whether or not a certification election shall be held "may well be left to the sound discretion of the Court of Industrial Relations, considering the conditions involved in the case…."

FN 25 under page 542- As a matter of fact, the only American Supreme Court decision cited in the

petition, National Labor Relations Board v. A.J. Tower Co., 25 likewise, sustains the same principle (under Gen. Maritime case). It was there held that the discretion of the labor tribunal, in this case, the National Labor Relations Board of the United States, is not lightly to be interfered with. (The issue in that case, concerns the procedure used in elections under the National Labor Relations Act in which employees choose a statutory representative for purposes of collective bargaining. The propriety of the National Labor Relations Board's refusal to accept an employers post-election challenge to the eligibility of a voter who participated in a consent election must be determined. The First Circuit Court of Appeals set aside the Board's order and so the matter was then taken to the US SC on certiorari.)

- In reversing the Circuit Court of Appeals, Justice Murphy made clear the acceptance of such a doctrine in the light of the National Labor Relations Act thus: "As we have noted before, Congress has entrusted the Board with a wide degree of discretion in establishing the procedure and safeguards necessary to insure the fair and free choice of bargaining representatives by employees."

- In the United States as in the Philippines, the decision in such matters by the administrative agency is accorded the utmost respect. CJ Concepcion: “in such proceedings, the determination of what is an appropriate bargaining unit is "entitled to almost complete finality."

- The prevailing principle then on questions as to certification, as well as in other labor cases, is that only where there is a showing of clear abuse of discretion would this Tribunal be warranted in reversing the actuation of respondent Court. There is no showing of such a failing in this case.

DISPOSITIVE: The petition for certiorari is dismissed.

1.02 DEFINITIONS1.02.1 Employer and employee

FEATI UNIVERSITY V BAUTISTA18 SCRA 1191

ZALDIVAR; December 27, 1966

NATUREConsolidated cases/petitions for certiorari, prohibition w/ writ of preliminary injunction

FACTS-Jan 14, 1963: the President of the Faculty Club wrote to the President of the University a letter informing the latter of the organization of the Faculty Club as a labor union, duly registered with the Bureau of Labor Relations-Jan 22, 1963: another letter was sent, to which was attached a list of demands consisting of 26 items, and asking the President of the University to answer within ten days from date of receipt thereof.-The University questioned the right of the Faculty Club to be the exclusive representative of the majority of the employees and asked proof that the Faculty Club had been designated or selected as exclusive representative by the vote of the majority of said employees.-Feb 1, 1963: the Faculty Club filed with the Bureau of Labor Relations a notice of strike alleging as reason therefore the refusal of the University to bargain collectively with the representative of the faculty members.

25 329 US 324 (1946). Reference was made in the earlier portion of this opinion that the petition contains "copious references to National Labor Relations Board cases." While not to be discouraged as the Industrial Peace Act owes much to the National Labor Relations Act of 1935, commonly known as the Warner Act, as well as to the Norris-La Guardia Act of 1932, still their persuasive force would depend on the fuller discussion of the facts in each of the cases cited and the rulings arrived at. Such feature is conspicuously lacking in the petition.

Page 3: 33994637 Labor Rounds 1and2

Labor Law 2 A2010 - 3 - Disini-Feb 18, 1963: the members of the Faculty Club went on strike and established picket lines in the premises of the University, thereby disrupting the schedule of classes.-March 1, 1963: the Faculty Club filed Case No. 3666-ULP for unfair labor practice against the University, but which was later dismissed (on April 2, 1963 after Case 41-IPA was certified to the CIR).-March 7, 1963: a petition for certification election, Case No. 1183-MC, was filed by the Faculty Club in the CIR

ISSUES1. WON the definition of employer in RA875 covers an educational institution like Feati University2. WON the members of the Faculty Club are independent contractors (If they are, then they are not employees within the purview of the said Act.)

HELD1. YES. It is true that the SC has ruled that certain educational institutions and other juridical entities are beyond the purview of RA875 in the sense that the CIR has no jurisdiction to take cognizance of ULP charges against them, but the principal reason in ruling in those cases is that those entities are not organized, maintained and operated for profit and do not declare dividends to stockholders.-In the decisions in the cases of the Boy Scouts of the Philippines, the University of San Agustin, the UST, and LaConsolacion College, this Court was not unanimous in the view that the Industrial Peace Act (Republic Act No. 875) is not applicable to charitable, or non-profit organizations which include educational institutions not operated for profit. There are members of this Court who hold the view that the Industrial Peace Act would apply also to non-profit organizations or entities, the only exception being the Government, including any political subdivision or instrumentality thereof, in so far as governmental functions are concerned. However, in the Far Eastern University case this Court is unanimous in supporting the view that an educational institution that is operated for profit comes within the scope of the Industrial Peace Act. We consider it a settled doctrine of this Court, therefore, that the Industrial Peace Act is applicable to any organization or entity whatever may be its purpose when it was created. -TEST: Does the University operate as an educational institution for profit? Does it declare dividends for its stockholders? If it does not, it must be declared beyond the purview of Republic Act No. 875; but if it does, Republic Act No. 875 must apply to it. In this case, Feati University itself admits that it has declared dividends. CIR also found that the University is not for strictly educational purposes and that "It realizes profits and parts of such earning is distributed as dividends to private stockholders or individuals. Under this circumstance, and in consonance with the rulings in the decisions of this Court, above cited, it is obvious that Republic Act No. 875 is applicable to herein petitioner Feati University. -RA 875, Sec 2(c): The term employer includes any person acting in the interest of an employer, directly or indirectly, but shall not include any labor organization (otherwise than when acting as an employer) or any one acting in the capacity or agent of such labor organization. -It will be noted that in defining the term "employer" the Act uses the word "includes" and not the word "means". In using the word "includes" and not "means", Congress did not intend to give a complete definition of "employer", but rather that such definition should be complementary to what is commonly understood as employer. Congress intended the term to be understood in a broad meaning because, firstly, the statutory definition includes not only "a principal employer but also a person acting in the interest of the employer"; and, secondly, the Act itself specifically enumerated those who are not included in the term "employer", namely: (1) a labor organization (otherwise than when acting as an employer), (2) anyone acting in the capacity of officer or agent of such labor organization [Sec. 2(c)], and (3) the Government and any political subdivision or instrumentality thereof insofar as the right to strike for the purpose of securing changes or modifications in the terms and conditions of employment is concerned (Section 11). Among these statutory exemptions, educational institutions are not included; hence, they can be included in the term "employer". This Court, however, has ruled that those educational institutions that are not operated for profit are not within the purview of Republic Act No. 875.-RA 875 does not give a comprehensive but only a complementary definition of the term "employer". The term encompasses those that are in ordinary parlance "employers." What is commonly meant by "employer"? The term "employer" has been given several acceptations. The lexical definition is "one who employs; one who uses; one who engages or keeps in service;" and "to employ" is "to provide work and pay for; to engage one's service; to hire." [see full case for other definitions of the word employer as provided for by the Workmen's Compensation Act, the Minimum Wage Law, the Social Security Act, etc]-Jurisprudence: An employer is one who employs the services of others; one for whom employees work and who pays their wages or salaries (Black Law Dictionary, 4th ed., p. 618). -Under none of these definitions may the University be excluded. The University engaged the services of the professors, provided them work, and paid them compensation or salary for their services. Even if the University may be considered as a lessee of services under a contract between it and the members of its Faculty, still it is included in the term "employer". "Running through the word `employ' is the thought that there has been an agreement on the part of one person to perform a certain service in return for compensation to be paid by an employer.

2. NO.-RA 875, Section 2 (d): The term "employee" shall include any employee and shall not be limited to the “employee” of a particular employer unless the act explicitly states otherwise and shall include any individual whose work has ceased as a consequence of, or in connection with, any current labor dispute or because of any unfair labor practice and who has not obtained any other substantially equivalent and regular employment.-This definition, by the use of the term “include” is again complementary. This Court has defined the term "employer" as "one who employs the services of others; one for whom employees work and who pays their wages or salaries. Correlatively, an employee must be one who is engaged in the service of another; who performs services for another; who works for salary or wages.-It is admitted by the University that the striking professors and/or instructors are under contract to teach particular courses and that they are paid for their services. They are, therefore, employees of the University.-The contention of the University that the professors and/or instructors are independent contractors, because the University does not exercise control over their work, is likewise untenable. This Court takes judicial notice that a university controls the work of the members of its faculty; that a university prescribes the courses or subjects that professors teach, and when and where to teach; that the professors' work is characterized by regularity and continuity for a fixed duration; that professors are compensated for their services by wages and salaries, rather than by profits; that the professors and/or instructors cannot substitute others to do their work without the consent of the university; and that the professors can be laid off if their work is found not satisfactory. All these indicate that the university has control over their work; and professors are, therefore, employees and not independent contractors. -Moreover, even if university professors are considered independent contractors, still they would be covered by RA 875. This law modelled after the Wagner Act, or the National Labor Relations Act, of the United States, did not exclude "independent contractors" from the orbit of "employees". It was in the subsequent legislation the Labor Management Relation Act (Taft-Harley Act) that "independent contractors" together with agricultural laborers, individuals in domestic service of the home, supervisors, and others were excluded.

Disposition Petition for certiorari & prohibition with preliminary injunction dismissed. Writs prayed for therein denied. Writ of preliminary injunction dissolved. Costs against Feati University.

NYK INTERNATIONAL KNITWEAR CORP. PHILS. V NLRC (PUBLICO)

397 SCRA 607QUISUMBING; February 17, 2003

NATUREPetition for review on certiorari

FACTS- Petitioner NYK hired respondent Virginia Publico as a sewer. She was paid on a piece-rate basis, and was required to work from 8 AM to 12 midnight.- May 7, 1997 Publico left the work place early as she was not feeling well due to flu. Publico did not come to work the next day. Due to this absence, Publico was informed by Stephen Ng (owner of NYK) that she was dismissed.

ISSUE/S1. WON there was illegal dismissal;2. WON petitioners are solidarily liable to pay backwages and separation pay as there was no malice or bad faith.

HELD1. YESRatio The petitioners’ allegations of abandonment cannot stand the unswerving conclusion by the NLRC and Labor Arbiter. Reasoning Petitioners raised factual questions which are improper in a petition for review on certiorari. Finding of facts of the NLRC, particularly in a case where the NLRC and the Labor Arbiter are in agreement, are deemed binding and conclusive upon this Court.2. YESRatio Cathy Ng falls within the meaning of an “employer” as contemplated by the Labor Code, who may be held jointly and severally liable for the obligations of the corporation to its dismissed employees.Reasoning Since a corporation is an artificial person, it must have an officer who

Page 4: 33994637 Labor Rounds 1and2

Labor Law 2 A2010 - 4 - Disinican be presumed to be the employer, being the “person acting in the interest of the employer.”1 In this case, Cathy Ng, in her capacity as manager, is deemed the employer, and is thus solidarily liable regardless of absence malice. She cannot be exonerated from her liability in the payment to private respondent.

Disposition Instant petition is denied.

1.02.2 LABOR ORGANIZATION- LEGITIMATE LABOR ORGANIZATION

AIRLINE PILOTS ASSOCIATION OF THE PHILIPPINES V. CIR76 SCRA 274

CASTRO; April 15, 1977

Definition of Legitimate Labor Organization:Section 2(e) of R.A. 875 defines "labor organization" as any union or association of employees which exist, in whole or in part, for the purpose of the collective bargaining or dealing with employers concerning terms and conditions of employment." The emphasis of Industrial Peace Act is clearly on the pourposes for which a union or association of employees established rather than that membership therein should be limited only to the employees of a particular employer. Under Section 2(h) of R.A 875 "representative" is define as including "a legitimate labor organization or any officer or agent of such organization, whether or not employed by the employer or employee whom he represents." It cannot be overemphasized likewise that labor dispute can exist "regardless of whether the disputants stand in the proximate relation of employer and employee.”

DUNLOP SLAZENGER V SEC. OF LABOR (RUIZ)300 SCRA 120

PUNO; DECEMBER 11, 1998

NATUREPetition for certiorari

FACTS- Respondent union filed a Petition for Certification Election among the supervisory, office and technical employees of the petitioner company before the DOLE, Regional Office No. III.- Petitioner company filed a motion to dismiss based on 1) that the respondent union is comprised of supervisory and rank-and-file employees and cannot act as bargaining agent for the proposed unit; (2) that a single certification election cannot be conducted jointly among supervisory and rank-and-file employees; and (3) that the respondent union lacks legal standing since it failed to submit its books of accounts.- Respondent alleges that it is composed only of supervisory employees and that it has no obligation to attach its books of accounts since it is a legitimate labor organization.- The mediator arbiter granted the petition of the union. It said that the contention of the respondent that the petitioning union is composed of both supervisory and rank and file employees is not sufficient to dismiss the petition. It can be remedied thru the exclusion-inclusion proceedings wherein those employees who are occupying rank and file positions will be excluded from the list of eligible voters. The secretary of labor affirmed.

ISSUE/SWON the union can be composed of supervisory and rank and file employees

HELDNO. Ratio Article 245 of the Labor Code clearly provides that "supervisory employees shall not be eligible for membership in a labor organization of the rank-and-file employees.”Reasoning Public respondent gravely misappreciates the basic antipathy between the interest of supervisors and the interest of rank-and-file employees. There is a irreconcilability of their interests which cannot be cured even in the exclusion-inclusion proceedings.-Appropriate bargaining unit: Group of employees of a giver employer, composed of all or less than the entire body of employees, which the collective interests of all the employees, consistent with equity to the employer, indicate to be best suited to serve reciprocal rights and duties.

Disposition Petition is granted.

Lopez Sugar Corporation v. Sec. of Labor (and NATIONAL CONGRESS OF UNIONS IN THE SUGAR INDUSTRY OF THE

1 A.C. Ransom Labor union-CCLU v NLRC

PHILIPPINES (NACUSIP) and COMMERCIAL AND AGRO-INDUSTRIAL LABOR ORGANIZATION (CAILO))

Vitug ; August 1995247 SCRA 1

Facts-The Med-Arbiter, sustained by the Secretary of Labor and Employment, has ruled that Art. 257 is mandatory and give him no other choice than to conduct a certification election upon the receipt of the corresponding petition.

"Art. 257. Petitions in unorganized establishments. - In any establishment where there is no certified bargaining agent, a certification election shall automatically be conducted by the Med-Arbiter upon the filing of a petition by a legitimate labor organization."-National Congress of Unions in the Sugar Industry of the Philippines-TUCP ("NACUSIP-TUCP") filed with the Department of Labor and Employment ("DOLE") a petition for direct certification or for certification election to determine the sole and exclusive collective bargaining representative of the supervisory employees of herein petitioner, Lopez Sugar Corporation ("LSC"). NACUSIP-TUCP averred that it was a legitimate national labor organization; that LSC was employing 55 supervisory employees, the majority of whom were members of the union; that no other labor organization was claiming membership over the supervisory employees; that there was no existing collective bargaining agreement covering said employees; and that there was no legal impediment either to a direct certification of NACUSIP-TUCP or to the holding of a certification election.-LSC contended it. NACUSIP-TUCP submitted Charter Certificate No. 003-89, dated 20 July 1989, of the NACUSIP-TUCP Lopez Sugar Central Supervisory Chapter. -LSC appealed to the DOLE and asseverated that the order was a patent nullity and that the Med-Arbiter acted with grave abuse of discretion, Sec. of Labor denied it. Petition for certiorari was filed.

IssueWON the certification election should push through

HeldNo, because the labor organization is not legitimate.

It was held in Progressive Development Corporation vs. Secretary, Department of Labor and Employment:"But while Article 257 cited by the Solicitor General directs the automatic conduct of a certification election in an unorganized establishment, it also requires that the petition for certification election must be filed by a legitimate labor organization. Article 212(h) defines a legitimate labor organization as 'any labor organization duly registered with the DOLE and includes any branch or local thereof.' Rule 1, Section 1(j), Book V of the Implementing Rules likewise defines a legitimate labor organization as 'any labor organization duly registered with the DOLE and includes any branch, local or affiliate thereof .' "

Indeed, the law did not reduce the Med-Arbiter to an automaton which can instantly be set to impulse by the mere filing of a petition for certification election. He is still tasked to satisfy himself that all the conditions of the law are met, and among the legal requirements is that the petitioning union must be a legitimate labor organization in good standing.The petition for certification election, in the case at bench, was filed by the NACUSIP-TUCP, a national labor organization duly registered with the DOLE. The legitimate status of NACUSIP-TUCP might be conceded; being merely, however, an agent for the local organization (the NACUSIP-TUCP Lopez Sugar Central Supervisory Chapter), the federation's bona fide status alone would not suffice. The local chapter, as its principal, should also be a legitimate labor organization in good standing. Accordingly, in Progressive Development, we elucidated:"In the case of union affiliation with a federation, the documentary requirements are found in Rule II, Section 3(e), Book V of the Implementing Rules, which we again quote as follows:"'(c ) The local or chapter of a labor federation or national union shall have and maintain a constitution and by laws, set of officers and books of accounts. For reporting purposes, the procedure governing the reporting of independently registered unions, federations or national unions shall be observed.' "Since the 'procedure governing the reporting independently registered unions' refers to the certification and attestation requirements contained in Article 235, paragraph 2, it follows that the constitution and by-laws, set of officers and books of accounts submitted by the local and chapter must likewise comply with these requirements. The same rationale for requiring the submission of duly subscribed documents upon union registration exists in the case of union affiliation. Moreover, there is greater reason to exact compliance with the certification and attestation requirements because, as previously mentioned, several requirements applicable to independent union registration are no longer required in the case of the formation a local or chapter. The policy of the law in conferring greater bargaining power upon

Page 5: 33994637 Labor Rounds 1and2

Labor Law 2 A2010 - 5 - Disinilabor unions must be balanced with the policy of providing preventive measures against the commission of fraud."A local or chapter therefore becomes a legitimate labor organization only upon submission of the following to the BLR:"1) A charter certificate, within 30 days from its issuance by the labor federation or national union, and"2) The constitution and by-laws, a statement on the set of officers, and the books of accounts all of which are certified under oath by the secretary or treasurer, as the case may be, of such local or chapter, and attested to by its president."Absent compliance with these mandatory requirements, the local or chapter does not become legitimate labor organization."

The only document extant on record to establish the legitimacy of the NACUSIP-TUCP Lopez Sugar Central Supervisory Chapter is a charter certificate and nothing else. DispositionWHEREFORE, the assailed Decision of the Secretary of Labor, dated 06 March 1990, affirming that of the Med-Arbiter, is ANNULLED and SET ASIDE. The petition for certification election is dismissed.

1.02.3 LABOR DISPUTE FEATI UNIVERSITY V BAUTISTA

18 SCRA 1191ZALDIVAR; December 27, 1966

NATUREConsolidated cases/petitions for certiorari, prohibition w/ writ of preliminary injunction

FACTS-Jan 14, 1963: the President of the Faculty Club wrote to the President of the University a letter informing the latter of the organization of the Faculty Club as a labor union, duly registered with the Bureau of Labor Relations-Jan 22, 1963: another letter was sent, to which was attached a list of demands consisting of 26 items, and asking the President of the University to answer within ten days from date of receipt thereof.-The University questioned the right of the Faculty Club to be the exclusive representative of the majority of the employees and asked proof that the Faculty Club had been designated or selected as exclusive representative by the vote of the majority of said employees.-Feb 1, 1963: the Faculty Club filed with the Bureau of Labor Relations a notice of strike alleging as reason therefor the refusal of the University to bargain collectively with the representative of the faculty members.-Feb 18, 1963: the members of the Faculty Club went on strike and established picket lines in the premises of the University, thereby disrupting the schedule of classes.-March 1, 1963: the Faculty Club filed Case No. 3666-ULP for unfair labor practice against the University, but which was later dismissed (on April 2, 1963 after Case 41-IPA was certified to the CIR).-March 7, 1963: a petition for certification election, Case No. 1183-MC, was filed by the Faculty Club in the CIR.

ISSUESWON there is a labor dispute between the University and the Faculty Club

HELDYES. -RA 875 provides that the term "labor dispute" includes any controversy concerning terms, tenure or conditions of employment, or concerning the association or representation of persons in negotiating, fixing, maintaining, changing, or seeking to arrange terms or conditions of employment regardless of whether the disputants stand in proximate relation of employer and employees.-The test of whether a controversy comes within the definition of "labor dispute" depends on whether the controversy involves or concerns "terms, tenure or condition of employment" or "representation."-All the admitted facts show that the controversy between the University and the Faculty Club involved terms and conditions of employment, and the question of representation. Hence, there was a labor dispute between the University and the Faculty Club, as contemplated by Republic Act No. 875. -Recall: RA 875, sec10: When in the opinion of the President of the Philippines there exists a labor dispute in an industry indispensable to the national interest and when such labor dispute is certified by the President to the Court of Industrial Relations, said Court may cause to be issued a restraining order forbidding the employees to strike or the employer to lockout the employees, and if no other solution to the dispute is found, the Court may issue an order fixing the terms and conditions of employment.-To certify a labor dispute to the CIR is the prerogative of the President under the law, and this Court will not interfere in, much less curtail, the exercise of that prerogative. Once the jurisdiction is acquired pursuant to the presidential certification, the CIR may exercise its broad powers as provided in Commonwealth Act 103. All phases of the labor dispute and

the employer-employee relationship may be threshed out before the CIR, and the CIR may issue such order or orders as may be necessary to make effective the exercise of its jurisdiction. The parties involved in the case may appeal to the Supreme Court from the order or orders thus issued by the CIR.

Disposition Petition for certiorari & prohibition with preliminary injunction dismissed. Writs prayed for therein denied. Writ of preliminary injunction dissolved. Costs against Feati University.

SAN MIGUEL EMPLOYEES UNION V BERSAMIRA186 SCRA 496

MELENCIO-HERRERA; June 13, 1990

NATURESpecial civil action for certiorari

FACTS- SMC entered into contracts for merchandising services with Lipercon and D'Rite (L&D), independent contractors duly licensed by DOLE. In said contracts, it was expressly understood and agreed that the EEs employed by the contractors were to be paid by the latter and that none of them were to be deemed EEs or agents of SanMig. There was to be no employer-employee relation between the contractors and/or its workers, on the one hand, and SMC on the other.- Petitioner SMCEU-PTWGO (Union) is duly authorized representative of the monthly paid rank-and-file EEs of SMC. Their CBA provides that temporary, probationary, or contract EEs are excluded from the bargaining unit and outside scope of CBA.- Union advised SMC that some L&D workers had signed up for union membership and sought the regularization of their employment with SMC. Union alleged that this group of EEs, while appearing to be contractual workers of supposedly independent contractors, have been continuously working for SMC for a period of 6 months to 15 years and that their work is neither casual nor seasonal as they are performing work or activities necessary or desirable in the usual business or trade of SMC, and that there exists a "labor-only" contracting situation. It was then demanded that the employment status of these workers be regularized. This was not acted upon by SMC, and so Union filed a notice of strike, and then a second notice. - Series of pickets were staged by L&D workers in various SMC plants and offices. SMC RTC to enjoin the Union from: representing and or acting for and in behalf of the employees of L&D for the purposes of collective bargaining; calling for and holding a strike vote to compel plaintiff to hire the employees or workers of L&D, among others.- Union filed a Motion to Dismiss SMC's Complaint on the ground of lack of jurisdiction over the case/nature of the action, which motion was opposed by SMC, which was denied by respondent Judge. And after several hearings, issued Injunction. RTC reasoned that the absence of ER-EE relationship negates the existence of labor dispute, so court has jurisdiction to take cognizance of SMC's grievance. Hence, this action.

ISSUE1. WON RTC correctly assumed jurisdiction over the controversy and properly issued the Writ of Preliminary Injunction.

HELD1. NORe: Definition of Labor Dispute (p4 of Outline)Ratio A labor dispute can nevertheless exist “regardless of whether the disputants stand in the proximate relationship of employer and employee, provided the controversy concerns, among others, the terms and conditions of employment or a "change" or "arrangement" thereof” The existence of a labor dispute is not negatived by the fact that the plaintiffs and defendants do not stand in the proximate relation of employer and employee. (A212 LC)Reasoning Crucial to the resolution of the question on jurisdiction, is the matter of whether or not the case at bar involves, or is in connection with, or relates to a labor dispute. An affirmative answer would bring the case within the original and exclusive jurisdiction of labor tribunals to the exclusion of the regular Courts. In this case, the matter re terms, tenure and conditions of EE’s employment and the arrangement of those terms as well as the matter of representation bring these issues within the scope of a labor dispute. Hence it is the labor tribunals that have jurisdiction and not the regular courts

Page 6: 33994637 Labor Rounds 1and2

Labor Law 2 A2010 - 6 - DisiniRe: ER Functions and ULP (p30 of Outline)- As the case is indisputably linked with a labor dispute, jurisdiction belongs to the labor tribunals. So, Labor Arbiters have original and exclusive jurisdiction to hear and decide the following cases involving all workers including: [a] unfair labor practice cases; [b] those that workers may file involving wages, hours of work and other terms and conditions of employment; and [c] cases arising from any violation of A265 LC, including questions involving the legality of striker and lockouts. - SMC’s claim that the action is for damages under A19, 20 and 21 of CC is not enough to keep the case within the jurisdictional boundaries of regular Courts. That claim for damages is interwoven with a labor dispute. To allow the action filed below to prosper would bring about "split jurisdiction" which is obnoxious to the orderly administration of justice.- SC recognizes the proprietary right of SMC to exercise an inherent management prerogative and its best business judgment to determine whether it should contract out the performance of some of its work to independent contractors. However, the rights of all workers to self-organization, collective bargaining and negotiations, and peaceful concerted activities, including the right to strike in accordance with law (S3, A13, 1987 Constitution) equally call for recognition and protection. Those contending interests must be placed in proper perspective and equilibrium.

Disposition Petition is GRANTED.

NESTLE PHILS., INC. V NLRC (NUÑEZ)195 SCRA 340

GRIÑO-AQUINO; March 18, 1991

NATUREPetition for certiorari

FACTS- The private respondents, who were employed by Nestlé either as sales representatives or medical representatives, availed of the petitioner's car loan policy. Under that policy, the company advances the purchase price of a car to be paid back by the employee through monthly deductions from his salary, the company retaining the ownership of the motor vehicle until it shall have been fully paid for.- After having participated in an illegal strike, the private respondents were dismissed from service. Nestlé directed the private respondents to either settle the remaining balance of the cost of their respective cars, or return them to the company for proper disposition.- Private respondents failed and refused to avail of either option, so the company filed in the Regional Trial Court of Makati a civil suit to recover possession of the cars. The private respondents sought a temporary restraining order in the NLRC to stop the company from cancelling their car loans and collecting their monthly amortizations. The NLRC, en banc, granted their petition for injunction.- The company filed a motion for reconsideration, but it was denied for tardiness. Hence, this petition for certiorari alleging that the NLRC acted with grave abuse of discretion amounting to lack of jurisdiction when it issued a labor injunction without legal basis and in the absence of any labor dispute related to the same.

ISSUEWON there is a labor dispute between the petitioner and the private respondents

HELDNORatio Paragraph (1) of Article 212 of the Labor Code defines a labor dispute as follows:

"(1) 'Labor dispute' includes any controversy or matters concerning terms or conditions of employment or the association or representation of persons in negotiating, fixing, maintaining, changing or arranging the terms and conditions of employment, regardless of whether the disputants stand in the proximate relation of employer and employee."

Nestlé’s demand for payment of the private respondents' amortizations on their car loans, or, in the alternative, the return of the cars to the company, is not a labor, but a civil, dispute. It involves debtor-creditor relations, rather than employee-employer relations.

Reasoning Whether or not the private respondents remain as employees of the petitioner, there is no escape from their obligation to pay their outstanding accountabilities to the petitioner; and if they cannot afford it, to return the cars assigned to them. The options given to the private respondents are civil in nature arising from contractual obligations. There is no labor aspect involved in the enforcement of those obligations. The NLRC gravely abused its discretion and exceeded its jurisdiction by issuing the writ of injunction to stop the company from enforcing the civil obligation of the private respondents under the car loan agreements and from protecting its interest in the cars which, by the terms of those agreements, belong to it (the company) until their purchase price shall have been fully paid by the employee. The terms of the car loan agreements are not in issue in the labor case. The rights and obligations of the parties under those contracts may be enforced by a separate civil action in the regular courts, not in the NLRC.

Disposition Petition is granted.

1.03 LABOR RELATIONS POLICY1.03.1 Method Dispute Settlement

KIOK LOY VS NLRC (PAMBANSANG KILUSAN NG PAGGAWA)141 SCRA 179

CUEVAS: January 22, 1986

NATURE:

- Petition for certiorari to annul the decision of the National Labor Relations Commission

FACTS:- In a certification election held on October 3, 1978, the Pambansang Kilusang

Paggawa (Union for short) was subsequently certified in a resolution dated November 29, 1978 by the Bureau of Labor Relations as the sole and exclusive bargaining agent of the rank-and-file employees of Sweden Ice Cream Plant (Company for short). The Company's motion for reconsideration of the said resolution was denied on January 25, 1978.

- December 7, 1978, the Union furnished the Company with two copies of its proposed collective bargaining agreement. It also requested the Company for its counter proposals. Both requests were ignored and remained unacted upon by the Company.

- The Union, on February 14, 1979, filed a "Notice of Strike", with the Bureau of Labor Relations (BLR) on ground of unresolved economic issues in collective bargaining.

- Conciliation proceedings then followed during the thirty-day statutory cooling-off period.

- The Bureau of Labor Relations to certify the case to the National Labor Relations Commission for compulsory arbitration.

- The labor arbiter set the initial hearing for April 29, 1979. For failure however, of the parties to submit their respective position papers as required, the said hearing was cancelled and reset to another date.

- The Union submitted its position paper. - On July 20, 1979, the National Labor Relations Commission rendered its

decision declaring the respondent guilty of unjustified refusal to bargain - Petitioner contends that the National Labor Relations Commission acted without

or in excess of its jurisdiction or with grave abuse of discretion amounting to lack of jurisdiction in rendering the challenged decision.

- Petitioner further contends that the National Labor Relations Commission's finding of unfair labor practice for refusal to bargain is not supported by law

ISSUE/S:WON the respondent is guilty of unjustified refusal to bargain

HELD:YESunfair labor practice is committed when it is shown that the respondent employer, after having been served with a written bargaining proposal by the petitioning Union, did not even bother to submit an answer or reply to the said proposalRatioUnfair labor practice is committed when it is shown that the respondent employer, after having been served with a written bargaining proposal by the petitioning Union, did not even bother to submit an answer or reply to the said proposalReaspmomgCollective bargaining which is defined as negotiations towards a collective agreement, is one of the democratic frameworks under the New Labor Code, designed to stabilize the relation between labor and management and to create a climate of sound and stable industrial peace. It is a mutual responsibility of the employer and the Union and is characterized as a legal obligation. So much so that Article 249, par. (g) of the Labor Code makes it an unfair labor practice for an employer to refuse "to meet and convene promptly and expeditiously in good faith for the purpose of negotiating an agreement with respect to wages, hours of work, and all other terms and conditions of employment including proposals for adjusting any grievance or question arising under such an agreement and executing a contract incorporating such agreement, if requested by either party. While it is a mutual obligation of the parties to bargain, the employer, however, is not under any legal duty to initiate contract negotiation. The mechanics of collective bargaining is set in motion only when the following jurisdictional preconditions are present, namely, (1) possession of the status of majority representation of the employees' representative in accordance with any of the means of selection or designation provided for by the Labor Code; (2) proof of majority representation; and (3) a demand to bargain under Article 251, par. (a) of the New Labor Code .

Page 7: 33994637 Labor Rounds 1and2

Labor Law 2 A2010 - 7 - Disini- From the over-all conduct of petitioner company in relation to the task of negotiation, there can be no doubt that the Union has a valid cause to complain against its (Company's) attitude, the totality of which is indicative of the latter's disregard of, and failure to live up to, what is enjoined by the Labor Code to bargain in good faith.

DISPOSITION:- Petition dismissed

MANILA DIAMOND HOTEL EMPLOYEES UNION V CA (MANILA DIAMOND HOTEL)

447 SCRA 97AZCUNA; December 16, 2004

NaturePetition for review on certiorari of a decision of the Court of Appeals

Facts-Union filed a petition for certification election to be declared the exclusive bargaining representative of the Hotel’s employees. This petition was dismissed by DOLE for lack of legal requirements.-after a few months, Union sent a letter to Hotel informing it of its desire to negotiate for a collective bargaining agreement. This was rejected by the Hotel stating that the Union was not the employee’s bargaining agent as their petition for cetification election was denied.-Union filed a Notice of Strike with the NCMB alleging the Hotel’ refusal to bargain and for acts of unfair labor practices. NCMB summoned both parties and held series of dialogues. Union however suddenly went on strike-Secretary of DOLE assumed jurisdiction and ordered compulsory arbitration pursuant to art. 263 (g) of LC. And Union members were directed to return to work and for Hotel to accept them back. Hotel refused to accept the employees return. The order was modified (by a different Secretary) such that reinstatement was to be done only in the payroll.-Union filed for certiorari alleging grave abuse of discretion. Case was referred to the CA. CA affirmed that the “payroll reinstatement” was not a grave abuse of discretion.

ISSUEWON the CA commit grave abuse of discretion in affirming the validity of “payroll reinstatement”

HELD Yes-CA based its decision on the UST v NLRC case which affirms validity of payroll reinstatement. However, the UST case was made in light of a very important fact- the teachers could not be given back their academic assignments because the semester was already halfway. In the present case, there is no showing that the facts called for payroll reinstatement as an ALTERNATIVE remedy.-as to the nature of art.263(g), the State encourages an environment wherein employers and employees themselves must deal with their problems in a manner that mutually suits them best (as embodied in Art 3, Sec 3 of the Constitution). Hence a voluntary instead of compulsory mode of dispute settlement is the general rule.-Art. 263 (g) is an exception to this rule by allowing the Secretary of the DOLE to assume jurisdiction over a dispute involving an industry indispensable to the national interest. And under this rule, the law uses the phrase “under the same terms and conditions” which contemplates only ACTUAL REINSTATEMENT.

1.03.2 TRADE UNIONISMART. 211. Declaration of Policy. - A. It is the policy of the State:(b) To promote free trade unionism as an instrument for the enhancement of democracy and the promotion of social justice and development; (c) To foster the free and voluntary organization of a strong and united labor movement;

1.03.3 WORKER ENLIGHTENMENTVICTORIA V INCIONG

157 SCRA 339FERNAN; January 26, 1988

NATUREReview of Order of Labor Secretary reversing decision of NLRC

FACTS- Complainant Saturno Victoria is the president of the Far East Broadcasting Company Employees Union. On September 8, 1972, the said union declared a strike against respondent company for refusal to recognize them. On September 11, 1972, respondent filed with the Court of First Instance of Bulacan, Civil Case No. 750-V, for the issuance of an injunction and a prayer that the strike be declared illegal.- Strikers filed case with NLRC for reinstatement. Reinstatement granted without prejudice

to outcome of Civil Case 750-V.- Strike was declared illegal because company was a non-profit organization. Company dismissed complainant. - Complainant filed for illegal dismissal. NLRC granted. Sec. Inciong reversed.

ISSUE/S1. WON company should obtain clearance under Art. 267 before dismissing complainant.

HELD1. NOThe purpose in requiring a prior clearance from the Secretary of Labor in cases of shutdown or dismissal of employees, is to afford the Secretary ample opportunity to examine and determine the reasonableness of the request. Consequently, private respondent acted in good faith when it terminated the employment of petitioner upon a declaration of illegality of the strike.Ratio This is a matter of responsibility and of answerability. Petitioner as a union leader, must see to it that the policies and activities of the union in the conduct of labor relations are within the precepts of law and any deviation from the legal boundaries shall be imputable to the leader. He bears the responsibility of guiding the union along the path of law and to cause the union to demand what is not legally demandable, would foment anarchy which is a prelude to chaos. Reasoning As a strike is an economic weapon at war with the policy of the Constitution and the law at that time, a resort thereto by laborers shall be deemed to be a choice of remedy peculiarly their own and outside of the statute, and as such, the strikers must accept all the risks attendant upon their choice. If they succeed and the employer succumbs, the law will not stand in their way in the enjoyment of the lawful fruits of their victory. But if they fail, they cannot thereafter invoke the protection of the law for the consequences of their conduct unless the right they wished vindicated is one which the law will, by all means, protect and enforce.

Disposition WHEREFORE, the petition is dismissed. The decision of the acting Secretary of Labor is AFFIRMED in toto.

1.03.4 MACHINERY DISPUTE SETTLEMENT

ST MARTIN FUNERAL HOME V NLRC (ARICAYOS)495 SCRA 295

REGALADO; September 16, 1998

NATURESpecial civil action of certiorari

FACTS- stemmed from a complaint for illegal dismissal- Labor Arbiter in that case declared that no employer-employee relationship existed between the parties.- the private-respondent “employee” appealed to the NLRC, and the NLRC set aside the questioned decision and remanded the case to the labor arbiter- petitioner then filed a motion for reconsideration which the NLRC denied. Hence this present petition.

ISSUES1. Where should appeals from the NLRC be initially filed, considering that Section 9 of BP 129 seems to say that there are cases which fall within the appellate jurisdiction of the SC in accordance with the labor code, and not the CA

HELD1. The CAReasoning All references in the amended Section 9 of BP No. 129 to supposed appeals from the NLRC to the Supreme Court are interpreted and declared to mean and refer to petitions for certiorari under Rule 65.

Therefore, all such petitions should be initially filed in the CA in strict observance of the doctrine on the hierarchy of courts.Disposition The instant petition for certiorari is REMANDED, and all pertinent recorsd thereof ordered to be FORWARDED, to the CA for approporiate action and disposition.

DELTA VENTURES V. CABATO327 SCRA 521

QUISUMBING; MARCH 9, 2000

NATURE

Page 8: 33994637 Labor Rounds 1and2

Labor Law 2 A2010 - 8 - DisiniSpecial civil action for certiorari seeks to annul the Order Judge Cabato of the RTC, dismissing petitioner's amended third-party complaint, as well as the Order denying motion for reconsideration.

FACTSA Decision was rendered by LA declaring the Green Mountain Farm, Roberto Ongpin and Almus Alabe guilty of Illegal Dismissal and Unfair Labor Practice and ordering them to pay the complainants, in solidum plus attorney's fees in the amount of P10,000.00. Almus Alabe is also ordered to answer in exemplary damages in the amount of P5,000.00 each to all the complainants.

LA issued a writ of execution directing NLRC Deputy Sheriff Adam Ventura to execute the judgment. Sheriff Ventura then proceeded to enforce the writ by garnishing certain personal properties of respondents. Finding that said judgment debtors do not have sufficient personal properties to satisfy the monetary award, Sheriff Ventura proceeded to levy upon a real property registered in the name of Roberto Ongpin, one of the respondents in the labor case.

Before the scheduled auction sale, herein petitioner filed before the Commission a third-party claim asserting ownership over the property levied upon and subject of the Sheriff’s notice of sale. Labor Arbiter Rivera thus issued an order directing the suspension of the auction sale until the merits of petitioner's claim has been resolved.

However, petitioner filed with the RTC a complaint for injunction and damages, with a prayer for the issuance of a temporary restraining order against Sheriff Ventura. Judge Cabato issued a temporary restraining order, enjoining respondents in the civil case before him to hold in abeyance any action relative to the enforcement of the decision in the labor case. Further, petitioner filed with the Commission a manifestation questioning the latter's authority to hear the case, the matter being within the jurisdiction of the regular courts. The manifestation, however, was dismissed by Labor Arbiter Rivera.

ISSUEWON the trial court may take cognizance of the complaint filed by petitioner and consequently provide the injunctive relief sought.

HELDNO. Basic as a hornbook principle, jurisdiction over the subject matter of a case is conferred by law and determined by the allegations in the complaint which comprise a concise statement of the ultimate facts constituting the petitioner's cause of action. Ostensibly the complaint before the trial court was for the recovery of possession and injunction, but in essence it was an action challenging the legality or propriety of the levy vis-a-vis the alias writ of execution, including the acts performed by the Labor Arbiter and the Deputy Sheriff implementing the writ. The complaint was in effect a motion to quash the writ of execution of a decision rendered on a case properly within the jurisdiction of the Labor Arbiter, to wit: Illegal Dismissal and Unfair Labor Practice. Considering the factual setting, it is then logical to conclude that the subject matter of the third party claim is but an incident of the labor case, a matter beyond the jurisdiction of regional trial courts.

Precedent abound confirming the rule that said courts have no jurisdiction to act on labor cases or various incidents arising therefrom, including the execution of decisions, awards or orders. Jurisdiction to try and adjudicate such cases pertains exclusively to the proper labor official concerned under the Department of Labor and Employment. To hold otherwise is to sanction split jurisdiction which is obnoxious to the orderly administration of justice.

The broad powers granted to the Labor Arbiter and to the National Labor Relations Commission by Articles 217, 218 and 224 of the Labor Code can only be interpreted as vesting in them jurisdiction over incidents arising from, in connection with or relating to labor disputes, as the controversy under consideration, to the exclusion of the regular courts.

Having established that jurisdiction over the case rests with the Commission, we find no grave abuse of discretion on the part of respondent Judge Cabato in denying petitioner's motion for the issuance of an injunction against the execution of the decision of the National Labor Relations Commission.

1.03.5 INDUSTRIAL PEACEART. 211. Declaration of Policy. - A. It is the policy of the State: (f) To ensure a stable but dynamic and just industrial peace;

ART. 273. Study of labor-management relations. - The Secretary of Labor shall have the power and it shall be his duty to inquire into:

(a) the existing relations between employers and employees in the Philippines; (b) the growth of associations of employees and the effect of such associations upon employer-employee relations;

(c) the extent and results of the methods of collective bargaining in the determination of terms and conditions of employment; (d) the methods which have been tried by employers and associations of employees for maintaining mutually satisfactory relations; (e) desirable industrial practices which have been developed through collective bargaining and other voluntary arrangements; (f) the possible ways of increasing the usefulness and efficiency of collective bargaining for settling differences; (g) the possibilities for the adoption of practical and effective methods of labor-management cooperation; (h) any other aspects of employer-employee relations concerning the promotion of harmony and understanding between the parties; (i) the relevance of labor laws and labor relations to national development.

The Secretary of Labor shall also inquire into the causes of industrial unrest and take all the necessary steps within his power as may be prescribed by law to alleviate the same, and shall from time to time recommend the enactment of such remedial legislation as in his judgment may be desirable for the maintenance and promotion of industrial peace.

1.03.6 WORKER PARTICIPATION1987 CONSTI, ART XIII, Sec 3. The State shall afford full protection to labor, local and overseas, organized and unorganized, and promote full employment and equality of employment opportunities for all.

It shall guarantee the rights of all workers to self-organization, collective bargaining and negotiations, and peaceful concerted activities, including the right to strike in accordance with law. They shall be entitled to security of tenure, humane conditions of work, and a living wage. They shall also participate in policy and decision-making processes affecting their rights and benefits as may be provided by law.

The State shall promote the principle of shared responsibility between workers and employers and the preferential use of voluntary modes in settling disputes, including conciliation, and shall enforce their mutual compliance therewith to foster industrial peace.

The State shall regulate the relations between workers and employers, recognizing the right of labor to its just share in the fruits of production and the right of enterprises to reasonable returns to investments, and to expansion and growth.

ART. 211. Declaration of Policy. - A. It is the policy of the State: (g) To ensure the participation of workers in decision and policy-making processes affecting their rights, duties and welfare.

ART. 255. Exclusive bargaining representation and workers’ participation in policy and decision-making. - The labor organization designated or selected by the majority of the employees in an appropriate collective bargaining unit shall be the exclusive representative of the employees in such unit for the purpose of collective bargaining. However, an individual employee or group of employees shall have the right at any time to present grievances to their employer. Any provision of law to the contrary notwithstanding, workers shall have the right, subject to such rules and regulations as the Secretary of Labor and Employment may promulgate, to participate in policy and decision-making processes of the establishment where they are employed insofar as said processes will directly affect their rights, benefits and welfare. For this purpose, workers and employers may form labor-management councils: Provided, That the representatives of the workers in such labor-management councils shall be elected by at least the majority of all employees in said establishment. (As amended by Section 22, Republic Act No. 6715, March 21, 1989). ART. 277. Miscellaneous provisions. - (a) All unions are authorized to collect reasonable membership fees, union dues, assessments and fines and other contributions for labor education and research, mutual death and hospitalization benefits, welfare fund, strike fund and credit and cooperative undertakings. (As amended by Section 33, Republic Act No. 6715, March 21, 1989). (g) The Ministry shall help promote and gradually develop, with the agreement of labor organizations and employers, labor-management cooperation programs at appropriate levels of the enterprise based on the shared responsibility and mutual respect in order to ensure industrial peace and improvement in productivity, working conditions and the quality of working life. (Incorporated by Batas Pambansa Bilang 130, August 21, 1981).

PHIL. AIRLINES V NLRC (PALEA)225 SCRA 301

MELO; August 13, 1993

Page 9: 33994637 Labor Rounds 1and2

Labor Law 2 A2010 - 9 - DisiniNATUREPetition for certiorari

FACTS- On March 15, 1985, PAL completely revised its 1966 Code of Discipline. The Code was circulated among the employees and was immediately implemented, and some employees were subjected to the disciplinary measures.- The Philippine Airlines Employees Association (PALEA) filed a complaint before the NLRC contending that PAL, by its unilateral implementation of the Code, was guilty of unfair labor practice, specifically Paragraphs E and G of Art 249 and Art 253 of the Labor Code. PALEA alleged that copies of the Code had been circulated in limited numbers; that being penal in nature the Code must conform with the requirements of sufficient publication, and that the Code was arbitrary, oppressive, and prejudicial to the rights of the employees. It prayed that implementation of the Code be held in abeyance; that PAL should discuss the substance of the Code with PALEA; that employees dismissed under the Code reinstated and their cases subjected to further hearing; and that PAL be declared guilty of unfair labor practice and be ordered to pay damages.- PAL filed a MTD, asserting its prerogative as an employer to prescribe rules and regulations regarding employees' conduct in carrying out their duties and functions, and alleging that it had not violated the CBA or any provision of the Labor Code.

ISSUE1. WON the formulation of a Code of Discipline among employees is a shared responsibility of the employer and the employees

HELD1. YES.Ratio Employees have a right to participate in the deliberation of matters which may affect their rights and the formulation of policies relative thereto and one such matter is the formulation of a code of discipline.Reasoning It was only on March 2, 1989, with the approval of RA 6715, amending Art 211 of the Labor Code, that the law explicitly considered it a State policy "to ensure the participation of workers in decision and policy-making processes affecting their rights, duties and welfare." However, even in the absence of said clear provision of law, the exercise of management prerogatives was never considered boundless. Thus, in Cruz vs. Medina, it was held that management's prerogatives must be without abuse of discretion.- In San Miguel Brewery Sales Force Union vs. Ople, we upheld the company's right to implement a new system of distributing its products, but gave the following caveat: So long as a company's management prerogatives are exercised in good faith for the advancement the employer's interest and not for the purpose of defeating or circumventing the rights of the employee, under special laws or under valid agreements, this Court will uphold them.- All this points to the conclusion that the exercise of managerial prerogatives is not unlimited. It is circumscribed by limitations found in law, a CBA, or the general principles of fair play and justice. Moreover, it must be duly established that the prerogative being invoked is clearly a managerial one.- Verily, a line must be drawn between management prerogatives regarding business operations per se and those which affect the rights of the employees. In treating the latter, management should see to it that its employees are at least properly informed of its decisions or modes of action. PAL asserts that all its employees have been furnished copies of the Code, the LA and the NLRC found to the contrary, which finding, is entitled to great respect.- PALEA recognizes the right of the Company to determine matters of management policy and Company operations and to direct its manpower. Management of the Company includes the right to organize, plan, direct and control operations, to hire, assign employees to work, transfer employees from one department to another, to promote, demote, discipline, suspend or discharge employees for just cause; to lay-off employees for valid and legal causes, to introduce new or improved methods or facilities or to change existing methods or facilities and the right to make and enforce Company rules and regulations to carry out the functions of management. The exercise by management of its prerogative shall be done in a just, reasonable, humane and/or lawful manner.- Such provision in the CBA may not be interpreted as cession of employees' rights to participate in the deliberation of matters which may affect their rights and the formulation of policies relative thereto. And one such matter is the formulation of a code of discipline. Industrial peace cannot be achieved if the employees are denied their just participation in the discussion of matters affecting their rights.Disposition Petition is DISMISSED.

MANILA ELECTRIC CO.. V QUISUMBING (MEWA).326 SCRA 172

YNARES-SANTIAGO; February 22, 2000

NATURE Motion for Reconsideration

FACTS - Members of the Meralco Employees and Workers Association (MEWA) filed a motion for reconsideration of an earlier decision of this Court directing the parties to execute a CBA

incorporating the terms and conditions contained in the unaffected portions of the Secretary of Labor's orders, and prayed for certain modifications.

ISSUE/S1. WON the decision should be modified

HELD1. YESRatio (On the requirement of consultation imposed by the Secretary in cases of contracting out for 6 months or more) A line must be drawn between management prerogatives regarding business operations per se and those which affect the rights of employees, and in treating the latter, the employer should see to it that its employees are at least properly informed of its decision or modes of action in order to attain a harmonious labor-management relationship and enlighten the workers concerning their rights. Hiring of workers is within the employer's inherent freedom to regulate and is a valid exercise of its management prerogative subject only to special laws and agreements on the matter and the fair standards of justice.Reasoning The management cannot be denied the faculty of promoting efficiency and attaining economy by a study of what units are essential for its operation. It has the ultimate determination of whether services should be performed by its personnel or contracted to outside agencies. Absent proof that management acted in a malicious or arbitrary manner, the Court will not interfere with the exercise of judgment by an employer.Obiter- On increasing the wage awards: The Court does "not seek to enumerate in this decision the factors that should affect wage determination" because collective bargaining disputes particularly those affecting the national interest and public service "requires due consideration and proper balancing of the interests of the parties to the dispute and of those who might be affected by the dispute.” - On the retroactivity of the arbitral awards: The law is silent as to the retroactivity of a CBA arbitral award or that granted not by virtue of the mutual agreement of the parties but by intervention of the government. Despite the silence of the law, the Court rules herein that CBA arbitral awards granted after six months from the expiration of the last CBA shall retroact to such time agreed upon by both employer and the employees or their union. Absent such an agreement as to retroactivity, the award shall retroact to the first day after the six-month period following the expiration of the last day of the CBA should there be one. In the absence of a CBA, the Secretary's determination of the date of retroactivity as part of his discretionary powers over arbitral awards shall control- On the grant of a housing loan but not a cooperative loan: The award of loans for housing is justified because it pertains to a basic necessity of life. It is part of a privilege recognized by the employer and allowed by law. In contrast, providing seed money for the establishment of the employee's cooperative is a matter in which the employer has no business interest or legal obligation.Disposition Petition PARTIALLY GRANTED. The arbitral award was made to retroact and the award of wages was increased from P1,900 to P2,000 for the years 1995 and 1996, subject to the monetary advances granted by petitioner to its rank-and-file employees during the pendency of this case assuming such advances had

1.03.7 WAGE FIXINGART. 211. Declaration of Policy. - A. It is the policy of the State: B. To encourage a truly democratic method of regulating the relations between the employers and employees by means of agreements freely entered into through collective bargaining, no court or administrative agency or official shall have the power to set or fix wages, rates of pay, hours of work or other terms and conditions of employment, except as otherwise provided under this Code. (As amended by Section 3, Republic Act No. 6715, March 21, 1989).

ART. 263. Strikes, picketing and lockouts. - (a) It is the policy of the State to

encourage free trade unionism and free collective bargaining. (g) When, in his opinion, there exists a labor dispute causing or likely to cause a strike or lockout in an industry indispensable to the national interest, the Secretary of Labor and Employment may assume jurisdiction over the dispute and decide it or certify the same to the Commission for compulsory arbitration. Such assumption or certification shall have the effect of automatically enjoining the intended or impending strike or lockout as specified in the assumption or certification order. If one has already taken place at the time of assumption or certification, all striking or locked out employees shall immediately return-to-work and the employer shall immediately resume operations and readmit all workers under the same terms and conditions prevailing before the strike or lockout. The Secretary of Labor and Employment or the Commission may seek the assistance of law enforcement agencies to ensure compliance with this provision as well as with such orders as he may issue to enforce the same.

In line with the national concern for and the highest respect accorded to the right of patients to life and health, strikes and lockouts in hospitals, clinics and similar medical institutions shall, to every extent possible, be avoided, and all serious

Page 10: 33994637 Labor Rounds 1and2

Labor Law 2 A2010 - 10 - Disiniefforts, not only by labor and management but government as well, be exhausted to substantially minimize, if not prevent, their adverse effects on such life and health, through the exercise, however legitimate, by labor of its right to strike and by management to lockout. In labor disputes adversely affecting the continued operation of such hospitals, clinics or medical institutions, it shall be the duty of the striking union or locking-out employer to provide and maintain an effective skeletal workforce of medical and other health personnel, whose movement and services shall be unhampered and unrestricted, as are necessary to insure the proper and adequate protection of the life and health of its patients, most especially emergency cases, for the duration of the strike or lockout. In such cases, therefore, the Secretary of Labor and Employment may immediately assume, within twenty four (24) hours from knowledge of the occurrence of such a strike or lockout, jurisdiction over the same or certify it to the Commission for compulsory arbitration. For this purpose, the contending parties are strictly enjoined to comply with such orders, prohibitions and/or injunctions as are issued by the Secretary of Labor and Employment or the Commission, under pain of immediate disciplinary action, including dismissal or loss of employment status or payment by the locking-out employer of backwages, damages and other affirmative relief, even criminal prosecution against either or both of them.

The foregoing notwithstanding, the President of the Philippines shall not be precluded from determining the industries that, in his opinion, are indispensable to the national interest, and from intervening at any time and assuming jurisdiction over any such labor dispute in order to settle or terminate the sameRA 6727 “WAGE RATIONALIZATION ACT” Sec. 2. It is hereby declared the policy of the State to rationalize the fixing of minimum wages and to promote productivity-improvement and gain-sharing measures to ensure a decent standard of living for the workers and their families; to guarantee the rights of labor to its just share in the fruits of production; to enhance employment generation in the countryside through industry dispersal; and to allow business and industry reasonable returns on investment, expansion and growth.

1.03.8 LABOR INJUNCTIONART. 254. Injunction prohibited. - No temporary or permanent injunction or restraining order in any case involving or growing out of labor disputes shall be issued by any court or other entity, except as otherwise provided in Articles 218 and 264 of this Code. (As amended by Batas Pambansa Bilang 227, June 1, 1982).

ART. 218. Powers of the Commission. - The Commission shall have the power and authority:

(e) To enjoin or restrain any actual or threatened commission of any or all prohibited or unlawful acts or to require the performance of a particular act in any labor dispute which, if not restrained or performed forthwith, may cause grave or irreparable damage to any party or render ineffectual any decision in favor of such party: Provided, That no temporary or permanent injunction in any case involving or growing out of a labor dispute as defined in this Code shall be issued except after hearing the testimony of witnesses, with opportunity for cross-examination, in support of the allegations of a complaint made under oath, and testimony in opposition thereto, if offered, and only after a finding of fact by the Commission, to the effect:

(1) That prohibited or unlawful acts have been threatened and will be committed and will be continued unless restrained, but no injunction or temporary restraining order shall be issued on account of any threat, prohibited or unlawful act, except against the person or persons, association or organization making the threat or committing the prohibited or unlawful act or actually authorizing or ratifying the same after actual knowledge thereof; (2) That substantial and irreparable injury to complainant’s property will follow;

(3) That as to each item of relief to be granted, greater injury will be inflicted upon complainant by the denial of relief than will be inflicted upon defendants by the granting of relief;

(4) That complainant has no adequate remedy at law; and

(5) That the public officers charged with the duty to protect complainant’s property are unable or unwilling to furnish adequate protection.

Such hearing shall be held after due and personal notice thereof has been served, in such manner as the Commission shall direct, to all known persons against whom relief is sought, and also to the Chief Executive and other public officials of the province or city within which the unlawful acts have been threatened or committed, charged with the duty to protect complainant’s property: Provided, however, that if a complainant shall also allege that, unless a temporary restraining order shall be issued without notice, a substantial and irreparable injury to complainant’s property will be unavoidable, such a temporary restraining order may be issued upon testimony under oath, sufficient, if sustained, to justify the Commission in issuing a temporary injunction upon hearing after notice. Such a temporary restraining order shall be effective for no longer than twenty (20) days and shall become void at the expiration of said twenty (20) days. No such temporary restraining order or temporary injunction shall be issued except on condition that complainant shall first file an undertaking with adequate security in an amount to be fixed

by the Commission sufficient to recompense those enjoined for any loss, expense or damage caused by the improvident or erroneous issuance of such order or injunction, including all reasonable costs, together with a reasonable attorney’s fee, and expense of defense against the order or against the granting of any injunctive relief sought in the same proceeding and subsequently denied by the Commission. The undertaking herein mentioned shall be understood to constitute an agreement entered into by the complainant and the surety upon which an order may be rendered in the same suit or proceeding against said complainant and surety, upon a hearing to assess damages, of which hearing, complainant and surety shall have reasonable notice, the said complainant and surety submitting themselves to the jurisdiction of the Commission for that purpose. But nothing herein contained shall deprive any party having a claim or cause of action under or upon such undertaking from electing to pursue his ordinary remedy by suit at law or in equity: Provided, further, That the reception of evidence for the application of a writ of injunction may be delegated by the Commission to any of its Labor Arbiters who shall conduct such hearings in such places as he may determine to be accessible to the parties and their witnesses and shall submit thereafter his recommendation to the Commission. (As amended by Section 10, Republic Act No. 6715, March 21, 1989).

RA 8791 , Section 22. Strikes and Lockouts. — The banking industry is hereby declared as indispensable to the national interest and, not withstanding the provisions of any law to the contrary, any strike or lockout involving banks, if unsettled after seven (7) calendar days shall be reported by the Bangko Sentral to the Secretary of Labor who may assume jurisdiction over the dispute or decide it or certify the same to the National Labor Relations Commission for compulsory arbitration. However, the President of the Philippines may at any time intervene and assume jurisdiction over such labor dispute in order to settle or terminate the same. (6-E)

1.03.9 TRIPARTISMART. 275. Tripartism and tripartite conferences. - (a) Tripartism in labor relations is hereby declared a State policy. Towards this end, workers and employers shall, as far as practicable, be represented in decision and policy-making bodies of the government.

(b) The Secretary of Labor and Employment or his duly authorized representatives may, from time to time, call a national, regional, or industrial tripartite conference of representatives of government, workers and employers for the consideration and adoption of voluntary codes of principles designed to promote industrial peace based on social justice or to align labor movement relations with established priorities in economic and social development. In calling such conference, the Secretary of Labor and Employment may consult with accredited representatives of workers and employers. (As amended by Section 32, Republic Act No. 6715, March 21, 1989).

II. RIGHT TO SELF-ORGANIZATIONBASIS OF RIGHT

1. CONSTITUTION1987, Art III Sec 8.- The right of the people, including those employed in the public and private sectors, to form unions, associations, or societies for purposes not contrary to law shall not be abridged.

1987 CONSTI, ART XIII, Sec 3. The State shall afford full protection to labor, local and overseas, organized and unorganized, and promote full employment and equality of employment opportunities for all.It shall guarantee the rights of all workers to self-organization, collective bargaining and negotiations, and peaceful concerted activities, including the right to strike in accordance with law. They shall be entitled to security of tenure, humane conditions of work, and a living wage. They shall also participate in policy and decision-making processes affecting their rights and benefits as may be provided by law. The State shall promote the principle of shared responsibility between workers and employers and the preferential use of voluntary modes in settling disputes, including conciliation, and shall enforce their mutual compliance therewith to foster industrial peace. The State shall regulate the relations between workers and employers, recognizing the right of labor to its just share in the fruits of production and the right of enterprises to reasonable returns to investments, and to expansion and growth.

1935 , Art III Sec 6- The right to form associations or societies for purposes not contrary to law shall not be abridged.

1973, Art IV, Sec 7- Section 7. The right to form associations or societies for purposes not contrary to the law shall not be abridged.

2. STATUTORY

Page 11: 33994637 Labor Rounds 1and2

Labor Law 2 A2010 - 11 - DisiniART. 243. Coverage and employees’ right to self-organization. - All persons employed in commercial, industrial and agricultural enterprises and in religious, charitable, medical, or educational institutions, whether operating for profit or not, shall have the right to self-organization and to form, join, or assist labor organizations of their own choosing for purposes of collective bargaining. Ambulant, intermittent and itinerant workers, self-employed people, rural workers and those without any definite employers may form labor organizations for their mutual aid and protection. (As amended by Batas Pambansa Bilang 70, May 1, 1980).

ART. 244. Right of employees in the public service. - Employees of government corporations established under the Corporation Code shall have the right to organize and to bargain collectively with their respective employers. All other employees in the civil service shall have the right to form associations for purposes not contrary to law. (As amended by Executive Order No. 111, December 24, 1986).

ART. 245. Ineligibility of managerial employees to join any labor organization; right of supervisory employees. - Managerial employees are not eligible to join, assist or form any labor organization. Supervisory employees shall not be eligible for membership in a labor organization of the rank-and-file employees but may join, assist or form separate labor organizations of their own. (As amended by Section 18, Republic Act No. 6715, March 21, 1989).

UST FACULTY UNION V. BITONIO, JR.318 SCRA 185

PANGANIBAN; November 16, 1999

NATURESpecial civil action in the Supreme Court. Certiorari.

FACTS- Union announced a general assembly to elect next union officers. - TRO was issued by med-arbiter enjoining them from conducting election.- UST held a general faculty assembly, attended by both union members and non-members. Here, appellants were elected as new union officers by acclamation and clapping of hands.- Appellees filed instant petition to seek injunctive relief and to nullify results of election.- Bitonio upheld med-arbiter and said election was void. He rejected contention that it was a legitimate exercise of right to self organization

ISSUE/SBasis of right to self-organization (p5 of outline) / Workers with right of self-organization

HELDRatio Self-organization is a fundamental right to form, join or assist labor organizations for collective bargaining, mutual aid and protection. Whether employed for a definite period or not, employee shall be considered as such, beginning on 1 st day of service, for purposes of membership in a labor union. Corollary to this right is the prerogative not to join. Reasoning The election can’t be considered as exercise of right to self-organization because the petitioners’ frustration over the performance of the respondents could not justify the method they chose to impose their will on the union.

NATIONAL UNION OF BANK EMPLOYEES v MINISTER OF LABOR, PRODUCERS BANK OF THE PHILIPPINES

110 SCRA 275MAKASIAR; December 14, 1981

NATURE: MANDAMUSFACTS:-The Union filed this action to compel the public respondents to hold a certification election among the rank and file employees of Producers Bank.-The Bank agreed to recognize and negotiate with the Union as soon as the latter obtained its registration certificate as local union. -However, even if said certificate had already been secured, the Bank failed to submit its payroll of employees (which was required previously at the hearing for direct certification. )- The Med- Arbiter issued an order for the holding of a certification election, a proceeding which was sought to be suspended by the Bank on the grounds that a prejudicial question was pending re: cancellation of the Union’s registration for allegedly engaging in prohibited and unlawful activities.

ISSUE: WON a certification election should be held despite the pending petition to cancel the Union’s certificate of Registration

HELD: YES. The pendency of the petition for cancellation of the registration certificate of herein petitioner union is not a bar to the holding of a certification election.

Reasoning:

-The pendency of the petition for cancellation of the registration certificate of petitioner union founded on the alleged illegal strikes staged by the leaders and members of the intervenor union and petitioner union should not suspend the holding of a certification election, because there is no order directing such cancellation. - Aside from the fact that the petition for cancellation of the registration certificate of petitioner union has not yet been finally resolved, there is another fact that militates against the stand of the Bank, the liberal approach observed by this Court as to matters of certification election. - Atlas Free Workers Union (AFWU)-PSSLU Local vs. Hon. Carmelo C. Noriel, et al. : "[T]he Court resolves to grant the petition (for mandamus) in line with the liberal approach consistently adhered to by this Court in matters of certification election. The whole democratic process is geared towards the determination of representation, not only in government but in other sectors as well, by election. Thus, the Court has declared its commitment to the view that a certification election is crucial to the institution of collective bargaining, for it gives substance to the principle of majority rule as one ' of the basic concepts of a democratic policy" (National Mines and Allied Workers Union vs. Luna, 83 SCRA 610). - Scout Ramon V. Albano Memorial College vs. Noriel, et al.: ... The institution of collective bargaining is, to recall Cox, a prime manifestation of industrial democracy at work. The two parties to the relationship, labor and management, make their own rules by coming to terms. That is to govern themselves in matters that really count. As labor, however, is composed of a number of individuals, it is indispensable that they be represented by a labor organization of their choice. Thus may be discerned how crucial is a certification election. So our decisions from the earliest case of PLDT Employees Union v. PLDT Co. Free Telephone Workers Union to the latest, Philippine Communications, Electronics & Electricity Workers' Federation (PCWF) v. Court of Industrial Relations, had made clear. The same principle was again given expression in language equally emphatic in the subsequent case of Philippine Association of Free Labor Unions v. Bureau of Labor Relations: 'Petitioner thus appears to be woefully lacking in awareness of the significance of a certification election for the collective bargaining process. It is the fairest and most effective way of determining which labor organization can truly represent the working force. It is a fundamental postulate that the will of the majority, if given expression in an honest election with freedom on the part of the voters to make their choice, is controlling. No better device can assure the institution of industrial democracy with the two parties to a business enterprise, management and labor, establishing a regime of self-rule .' That is to accord respect to the policy of the Labor Code, indisputably partial to the holding of a certification election so as to arrive in a manner definitive and certain concerning the choice of the labor organization to represent the workers in a collective bargaining unit (emphasis supplied).

- Cancellation of the registration certificate is not the only resultant penalty in case of any violation of the Labor Code. (See Sec. 8 Rule II Book V of Labor Code, in relation to A273)

- As aptly ruled by respondent Bureau of Labor Relations Director Noriel: "The rights of workers to self-organization finds general and specific constitutional guarantees. Section 7, Article IV of the Philippine Constitution provides that the right to form associations or societies for purposes not contrary to law shall not be abridged. This right is more pronounced in the case of labor. Section 9, Article II (ibid) specifically declares that the State shall assure the rights of workers to self-organization, collective bargaining, security of tenure and just and humane conditions of work. Such constitutional guarantees should not be lightly taken much less easily nullified. A healthy respect for the freedom of association demands that acts imputable to officers or members be not easily visited with capital punishments against the association itself"

Dispositive: THE WRIT OF mandamus IS GRANTED; BLR DIR ORDERED TO CALL AND DIRECT THE IMMEDIATE HOLDING OF A CERTIFICATION ELECTION.

3. UNIVERSAL DECLARATION OF HUMAN RIGHTS

PREAMBLE

Whereas recognition of the inherent dignity and of the equal and inalienable rights of all members of the human family is the foundation of freedom, justice and peace in the world, Whereas disregard and contempt for human rights have resulted in barbarous acts which have outraged the conscience of mankind, and the advent of a world in which human beings shall enjoy freedom of speech and belief and freedom from fear and want has been proclaimed as the highest aspiration of the common people,

Page 12: 33994637 Labor Rounds 1and2

Labor Law 2 A2010 - 12 - DisiniWhereas it is essential, if man is not to be compelled to have recourse, as a last resort, to rebellion against tyranny and oppression, that human rights should be protected by the rule of law, Whereas it is essential to promote the development of friendly relations between nations, Whereas the peoples of the United Nations have in the Charter reaffirmed their faith in fundamental human rights, in the dignity and worth of the human person and in the equal rights of men and women and have determined to promote social progress and better standards of life in larger freedom, Whereas Member States have pledged themselves to achieve, in co-operation with the United Nations, the promotion of universal respect for and observance of human rights and fundamental freedoms, Whereas a common understanding of these rights and freedoms is of the greatest importance for the full realization of this pledge,

Now, Therefore THE GENERAL ASSEMBLY proclaims THIS UNIVERSAL DECLARATION OF HUMAN RIGHTS as a common standard of achievement for all peoples and all nations, to the end that every individual and every organ of society, keeping this Declaration constantly in mind, shall strive by teaching and education to promote respect for these rights and freedoms and by progressive measures, national and international, to secure their universal and effective recognition and observance, both among the peoples of Member States themselves and among the peoples of territories under their jurisdiction. Article 2.Everyone is entitled to all the rights and freedoms set forth in this Declaration, without distinction of any kind, such as race, colour, sex, language, religion, political or other opinion, national or social origin, property, birth or other status. Furthermore, no distinction shall be made on the basis of the political, jurisdictional or international status of the country or territory to which a person belongs, whether it be independent, trust, non-self-governing or under any other limitation of sovereignty.Article 23. (4) Everyone has the right to form and to join trade unions for the protection of his interests.

INTERNATIONAL COVENANT ON ECONOMIC, SOCIAL AND CULTURAL RIGHTSArticle 2 - 1. Each State Party to the present Covenant undertakes to take steps, individually and through international assistance and co-operation, especially economic and technical, to the maximum of its available resources, with a view to achieving progressively the full realization of the rights recognized in the present Covenant by all appropriate means, including particularly the adoption of legislative measures. 2. The States Parties to the present Covenant undertake to guarantee that the rights enunciated in the present Covenant will be exercised without discrimination of any kind as to race, colour, sex, language, religion, political or other opinion, national or social origin, property, birth or other status. 3. Developing countries, with due regard to human rights and their national economy, may determine to what extent they would guarantee the economic rights recognized in the present Covenant to non-nationals.

Article 8 1. The States Parties to the present Covenant undertake to ensure:

(a) The right of everyone to form trade unions and join the trade union of his choice, subject only to the rules of the organization concerned, for the promotion and protection of his economic and social interests. No restrictions may be placed on the exercise of this right other than those prescribed by law and which are necessary in a democratic society in the interests of national security or public order or for the protection of the rights and freedoms of others; (b) The right of trade unions to establish national federations or confederations and the right of the latter to form or join international trade-union organizations; (c) The right of trade unions to function freely subject to no limitations other than those prescribed by law and which are necessary in a democratic society in the interests of national security or public order or for the protection of the rights and freedoms of others; (d) The right to strike, provided that it is exercised in conformity with the laws of the particular country.

2. This article shall not prevent the imposition of lawful restrictions on the exercise of these rights by members of the armed forces or of the police or of the administration of the State. 3. Nothing in this article shall authorize States Parties to the International Labour Organisation Convention of 1948 concerning Freedom of Association and Protection of the Right to Organize to take legislative measures which would prejudice, or apply the law in such a manner as would prejudice, the guarantees provided for in that Convention.

INTERNATIONAL COVENANT ON CIVIL AND POLITICAL RIGHTSArticle 22

1. Everyone shall have the right to freedom of association with others, including the right to form and join trade unions for the protection of his interests. 2. No restrictions may be placed on the exercise of this right other than those which are prescribed by law and which are necessary in a democratic society in the interests of national security or public safety, public order (ordre public), the protection of public health or morals or the protection of the rights and freedoms of others. This article shall not prevent the imposition of lawful restrictions on members of the armed forces and of the police in their exercise of this right.

3. Nothing in this article shall authorize States Parties to the International Labour Organisation Convention of 1948 concerning Freedom of Association and Protection of the Right to Organize to take legislative measures which would prejudice, or to apply the law in such a manner as to prejudice, the guarantees provided for in that Convention.

4. ILO CONVENTION NO. 48

STANDARD CHARTERED BANK EMPLOYEES UNION (NUBE) V CONFESOR432 SCRA 371

CALLEJO, SR; JUNE 16, 2004

NATURE Special civil action in the SC. Certiorari.FACTSThe Union alleged that the Bank violated its duty to bargain, hence, committed ULP under Article 248 (g) when it engaged in surface bargaining without any intent of reaching an agreement, as evident in the Bank’s counter-proposals. It explained that of the 34 economic provisions it presented, the Bank made only 6 economic counterproposals. Further, as borne by the minutes of the meetings, the Bank, after indicating the economic provisions it had rejected, accepted, retained or were open for discussion, refused to make a list of items it agreed to include in the economic package.

ISSUES1. WON the Union was able to substantiate its claim of unfair labor practice against the Bank arising from the latter’s alleged “interference” with its choice of negotiator (P. 6)2. WON the Union was able to substantiate its claim of unfair labor practice against the Bank arising from the latter’s surface bargaining; making bad faith non-economic proposals; and refusal to furnish the Union with copies of the relevant data (P. 25)3. WON the petitioner is estopped from filing the instant action (P. 28)HELD1. No. Under the International Labor Organization Convention (ILO) No. 87 FREEDOM OF ASSOCIATION AND PROTECTION OF THE RIGHT TO ORGANIZE to which the Philippines is a signatory, “workers and employers, without distinction whatsoever, shall have the right to establish and, subject only to the rules of the organization concerned, to job organizations of their own choosing without previous authorization.” Workers’ and employers’ organizations shall have the right to draw up their constitutions and rules, to elect their representatives in full freedom to organize their administration and activities and to formulate their programs. Article 2 of ILO Convention No. 98 pertaining to the Right to Organize and Collective Bargaining, provides:

Article 21. Workers’ and employers’ organizations shall enjoy adequate protection against any acts or interference by each other or each other’s agents or members in their establishment, functioning or administration.2. In particular, acts which are designed to promote the establishment of workers’ organizations under the domination of employers or employers’ organizations or to support workers’ organizations by financial or other means, with the object of placing such organizations under the control of employers or employers’ organizations within the meaning of this Article.

The aforecited ILO Conventions are incorporated in our Labor Code, particularly in Article 243 thereof, which provides:

ART. 243. COVERAGE AND EMPLOYEES’ RIGHT TO SELF-ORGANIZATION. — All persons employed in commercial, industrial and agricultural enterprises and in religious, charitable, medical or educational institutions whether operating for profit or not, shall have the right to self-organization and to form, join, or assist labor organizations of their own choosing for purposes of collective bargaining. Ambulant, intermittent and itinerant workers, self-employed people, rural workers and those without any definite employers may form labor organizations for their mutual aid and protection.

Page 13: 33994637 Labor Rounds 1and2

Labor Law 2 A2010 - 13 - Disiniand Articles 248 and 249 respecting ULP of employers and labor organizations.

The said ILO Conventions were ratified on December 29, 1953. However, even as early as the 1935 Constitution, 44 the State had already expressly bestowed protection to labor as part of the general provisions. The 1973 Constitution, 45 on the other hand, declared it as a policy of the state to afford protection to labor, specifying that the workers’ rights to self-organization, collective bargaining, security of tenure, and just and humane conditions of work would be assured. For its part, the 1987 Constitution, aside from making it a policy to “protect the rights of workers and promote their welfare,” devotes an entire section, emphasizing its mandate to afford protection to labor, and highlights “the principle of shared responsibility” between workers and employers to promote industrial peace.

Article 248(a) of the Labor Code, considers it an unfair labor practice when an employer interferes, restrains or coerces employees in the exercise of their right to self-organization or the right to form association. The right to self-organization necessarily includes the right to collective bargaining.

Parenthetically, if an employer interferes in the selection of its negotiators or coerces the Union to exclude from its panel of negotiators a representative of the Union, and if it can be inferred that the employer adopted the said act to yield adverse effects on the free exercise to right to self-organization or on the right to collective bargaining of the employees, ULP under Article 248(a) in connection with Article 243 of the Labor Code is committed.

In order to show that the employer committed ULP under the Labor Code, substantial evidence is required to support the claim. Substantial evidence has been defined as such relevant evidence as a reasonable mind might accept as adequate to support a conclusion. In the case at bar, the Union bases its claim of interference on the alleged suggestions of Diokno to exclude Umali from the Union’s negotiating panel.

The circumstances that occurred during the negotiation do not show that the suggestion made by Diokno to Divinagracia is an anti-union conduct from which it can be inferred that the Bank consciously adopted such act to yield adverse effects on the free exercise of the right to self-organization and collective bargaining of the employees, especially considering that such was undertaken previous to the commencement of the negotiation and simultaneously with Divinagracia’s suggestion that the bank lawyers be excluded from its negotiating panel.

The records show that after the initiation of the collective bargaining process, with the inclusion of Umali in the Union’s negotiating panel, the negotiations pushed through. The complaint was made only on August 16, 1993 after a deadlock was declared by the Union on June 15, 1993.

2. No. The Union alleges that the Bank violated its duty to bargain; hence, committed ULP under Article 248(g) when it engaged in surface bargaining.

Surface bargaining is defined as “going through the motions of negotiating” without any legal intent to reach an agreement. 50 The resolution of surface bargaining allegations never presents an easy issue. The determination of whether a party has engaged in unlawful surface bargaining is usually a difficult one because it involves, at bottom, a question of the intent of the party in question, and usually such intent can only be inferred from the totality of the challenged party’s conduct both at and away from the bargaining table. 51 It involves the question of whether an employer’s conduct demonstrates an unwillingness to bargain in good faith or is merely hard bargaining.

The minutes of meetings from March 12, 1993 to June 15, 1993 do not show that the Bank had any intention of violating its duty to bargain with the Union. Records show that after the Union sent its proposal to the Bank on February 17, 1993, the latter replied with a list of its counter-proposals on February 24, 1993. Thereafter, meetings were set for the settlement of their differences. The minutes of the meetings show that both the Bank and the Union exchanged economic and non-economic proposals and counter-proposals.

The Union has not been able to show that the Bank had done acts, both at and away from the bargaining table, which tend to show that it did not want to reach an agreement with the Union or to settle the differences between it and the Union. Admittedly, the parties were not able to agree and reached a deadlock. However, it is herein emphasized that the duty to bargain “does not compel either party to agree to a proposal or require the making of a concession.” Hence, the parties’ failure to agree did not amount to ULP under Article 248(g) for violation of the duty to bargain.

We, likewise, find that the Union failed to substantiate its claim that the Bank refused to furnish the information it needed.

While the refusal to furnish requested information is in itself an unfair labor practice, and also supports the inference of surface bargaining, in the case at bar, Umali, [the federation president] in a meeting dated May 18, 1993, requested the Bank to validate its guestimates on the data of the rank and file. However, Umali failed to put his request in writing as provided for in Article 242(c) of the Labor Code:

Article 242. Rights of Legitimate Labor Organization . . .(c) To be furnished by the employer, upon written request, with the annual audited financial statements, including the balance sheet and the profit and loss statement, within thirty (30) calendar days from the date of receipt of the request, after the union has been duly recognized by the employer or certified as the sole and exclusive bargaining representatives of the employees in the bargaining unit, or within sixty (60) calendar days before the expiration of the existing collective bargaining agreement, or during the collective negotiation;

The Union, did not, as the Labor Code requires, send a written request for the issuance of a copy of the data about the Bank’s rank and file employees. Moreover, as alleged by the Union, the fact that the Bank made use of the aforesaid guestimates, amounts to a validation of the data it had used in its presentation.

3. No. The respondent Bank argues that the petitioner is estopped from raising the issue of ULP when it signed the new CBA.

Article 1431 of the Civil Code provides:

Through estoppel an admission or representation is rendered conclusive upon the person making it, and cannot be denied or disproved as against the person relying thereon.

A person, who by his deed or conduct has induced another to act in a particular manner, is barred from adopting an inconsistent position, attitude or course of conduct that thereby causes loss or injury to another.

In the case, however, the approval of the CBA and the release of signing bonus do not necessarily mean that the Union waived its ULP claim against the Bank during the past negotiations. After all, the conclusion of the CBA was included in the order of the SOLE, while the signing bonus was included in the CBA itself. Moreover, the Union twice filed a motion for reconsideration respecting its ULP charges against the Bank before the SOLE.

Dispositive Assailed order and resolutions affirmed.

EXTENT AND SCOPE OF RIGHT

REYES V TRAJANO209 SCRA 484

NARVASA; June 2, 1992

NATURESpecial civil action of certiorari

FACTS -The officer-in-charge of the Bureau of Labor Relations (Hon. Cresenciano Trajano) sustained the denial by the Med Arbiter of the right to vote of one hundred forty-one (141) members of the "Iglesia ni Kristo" (INK), all employed in the same company, at a certification election at which two (2) labor organizations were contesting the right to be the exclusive representative of the employees in the bargaining unit.-The certification election was authorized to be conducted by the Bureau of Labor Relations among the employees of Tri-Union Industries Corporation on October 20, 1987. The competing unions were the Tri-Union Employees Union-Organized Labor Association in Line Industries and Agriculture (TUEU-OLALIA), and Trade Union of the Philippines and Allied Services (TUPAS). Of the 348 workers initially deemed to be qualified voters, only 240 actually took part in the election, conducted under the supervision of the Bureau of Labor Relations. Among the 240 employees who cast their votes were 141 members of the INK.The ballots provided for three (3) choices. They provided for votes to be cast, of course, for either of the two (2) contending labor organizations, (a) TUPAS and (b) TUEU-OLALIA; and, conformably with established rule and practice, 1 for (c) a third choice: "NO UNION."The final tally of the votes showed the following results:

TUPAS 1TUEU-OLALIA 95NO UNION 1SPOILED 1CHALLENGED 141

ISSUE/SWON the right to self-organization includes the right NOT to join, affiliate with, or assist any union, and to disaffiliate or resign from a labor organizationHELD: YES. Ratio EXTENT AND SCOPE OF RIGHT TO SELF-ORGANIZATION. Logically, the right NOT to join, affiliate with, or assist any union, and to disaffiliate or resign from a

Page 14: 33994637 Labor Rounds 1and2

Labor Law 2 A2010 - 14 - Disinilabor organization, is subsumed in the right to join, affiliate with, or assist any union, and to maintain membership therein. Reasoning The right to form or join a labor organization necessarily includes the right to refuse or refrain from exercising said right. It is self-evident that just as no one should be denied the exercise of a right granted by law, so also, no one should be compelled to exercise such a conferred right. The fact that a person has opted to acquire membership in a labor union does not preclude his subsequently opting to renounce such membership.

PAN-AMERICAN WORLD AIRWAYS, INC. V. PAN-AMERICAN EMPLOYEES ASSN.

27 SCRA 1202 FERNANDO; April 29, 1969

NATURESpecial civil action for certiorari

FACTS -CIR could not agree to exclude from a return-to-work order five union officials of respondent Pan American Employees Association on the ground of having led an illegal strike, in itself, according to Pan-American World Airways, a sufficient cause for dismissal thus resulting in their losing their incentive and motivation for doing their jobs properly with the consequent fear that they could cause grave injury to it

ISSUE1. WON CIR committed grave abuse of discretion in ordering the return to work of the

union officials

HELD1. NO.

Ratio There is both a constitutional and statutory recognition that laborers have the right to form unions to take care of their interests vis-a-vis their employers.Reasoning The moment management displays what in this case appears to be a grave but unwarranted distrust in the union officials discharging their functions just because a strike was resorted to, then the integrity of the collective bargaining process itself is called into question. It would have been different if there were a rational basis for such fears, purely speculative in character. The record is bereft of the slightest indication that any danger, much less one clear and present, is to be expected from their return to work. If petitioner were to succeed in their unprecedented demand, the laborers in this particular union would thus be confronted with the sad spectacle of the leaders of their choice condemned as irresponsible, possibly even constituting a menace to the operations of the enterprise. That is an indictment of the gravest character, devoid of any factual basis. What is worse, the result, even if not intended, would be to call into question their undeniable right to choose their leaders, who must be treated as such with all the respect to which they are legitimately entitled. The fact that they would be paid but not be allowed to work is, to repeat, to add to the infamy that would thus attach, to them necessarily, but to respondent union equally.

Disposition PETITION DENIED.

UNION OF SUPERVISORS (R.B)-NATU V SECRETARY OF LABOR (REPUBLIC BANK)

109 SCRA 139MAKASIAR; November 12, 1981

NATUREPetition for review on certiorari of the order of the Secretary of Labor

FACTS-The Republic Bank Provident Fund was established for the benefit of the officers and employees of the Republic Bank. -The fund is supposed to be managed by a Board of Trustees composed of 5 members, of which 3, including the chairman, are to be designated by the bank president, and the other 2 are the presidents of the Republic Bank Union of Supervisors and of the Republic Bank Employees' Union-Mr. Norberto Luna, president of the petitioner union and ex-officio member of the fund's Board of Trustees, became the fund's administrator and secretary. During the three (3) years of his incumbency as administrator, the resources of the fund grew from P278,445.27 to P1,779,159.85-On February 12, 1974, a meeting of the PF Board of Trustees was held, attended by Mr. Restituto de Vera who had then just been designated to sit on the board, and who opened the meeting stating that they would like to have control of the funds of the PF and for that matter the administration of the Fund, because the Provident Fund PF is an entity of the Republic Bank considering that the main bulk of contributions is put up by the RB into the PF. -Mr. Luna, the erstwhile administrator and Secretary of the Fund, vigorously objected. -Messrs. Armando Abad (chairman) and Mario Galicia, the two other management-

appointed trustees sided with de Vera, and forced the issue of reorganization. -The same was carried by a vote of 3 to 2, with all the management appointed trustees voting for it, and the two labor representatives voting against -Mr. Luna moved that all the trustees execute a trust agreement and a bond in favor of the PF members to protect the interests of the PF Messrs. Abad, de Vera and Galicia counter argued against the proposal. -De Vera questioned Luna's apprehensions. In answer, Luna made the allegedly derogatory statements- Luna and Antonio Canizares the other labor representative walked out of the meeting.-The remaining 3 trustees unanimously elected Galicia as the new administrator -The bank moved for the dismissal of Luna, and sought clearance from the Sec. of Labor for his termination-Labor Sec issued an order granting the same, and this is the subject of this petition

ISSUEWON Mr. Luna's utterances and alleged acts of insubordination constitute just cause for his dismissal

HELDNO.Luna's remarks at the meeting of an official board are privileged in nature as a valid exercise of his constitutional freedom of expression. He addressed his remarks to the body that has jurisdiction over the question of management of the assets of the Provident Fund. Luna's remarks were intended to protect the interests of the members of the Provident Fund from what he honestly believed was a risky venture on the part of the management. -His protests could even be treated as union activity by the Industrial Peace Act, which assures the employees' right "to self-organization and to form, join or assist labor organizations of their own choosing and to engage in concerted activities for the purpose of collective bargaining and other mutual aid and protection ... " (Sec. 3, Rep. Act 875). -This is so because Luna's membership in the PF Board of Trustees was by virtue of his being president of the RB Union of Supervisors. The Provident Fund was itself created as a result of the union's collective bargaining agreement with the bank. Luna was therefore acting out his role as protector of his constituents when he voiced out his apprehension and protests over the plan of management. It matters not that he acted singly or individually. What is important is that he had been selected by the supervisors of respondent bank to be their president and representative in the PF Board of Trustees. His actuations as such should therefore be considered as legitimate exercise of the employees' right to self-organization and as an activity for their mutual aid and protection, aside from being privileged communication protected by the constitutional guarantee on free speech. His remarks were in defense of the interest of the Provident Fund, part of which comes from the contribution of the rank and file employees.

Disposition Petition is granted.

CENTRAL NEGROS ELECTRIC COOPERATIVE INC V DOLE SECRETARY

201 SCRA 584REGALADO; September 13, 1991

NATURESpecial civil action for certiorari

FACTS- Petitioner Central Negros Electric Cooperative, Inc. (CENECO) seeks to annul the order issued by then Acting Secretary of Labor Laguesma declaring the projected certification election unnecessary and directing petitioner CENECO to continue recognizing private respondent CENECO Union of Rational Employees (CURE) as the sole and exclusive bargaining representative of all the rank-and-file employees of petitioner's electric cooperative.- Previous events: Their CBA was valid for a term of 3 years; CURE then wrote CENECO proposing that negotiations be conducted for a new agreement. CENECO denied CURE's request on the ground that, under applicable decisions of the Supreme Court, employees who at the same time are members of an electric cooperative are not entitled to form or join a union.- CURE filed a petition for direct recognition or for certification election; CENECO filed a motion to dismiss on the ground of legal restraints. - Some employees of CENECO then filed for withdrawal of membership in the cooperative but CENECO contended that this cannot be allowed.

ISSUE/S1. WON the employees were allowed to withdraw membership from the cooperative so as to entitle them to form or join CURE for purposes of the negotiations for a collective bargaining agreement

Page 15: 33994637 Labor Rounds 1and2

Labor Law 2 A2010 - 15 - DisiniHELD1. YESRatio Membership in the cooperative is on a voluntary basis. Hence, withdrawal therefrom cannot be restricted unnecessarily. The right to join an organization necessarily includes the equivalent right not to join the same.

EXTENT AND SCOPE OF RIGHT TO SELF-ORGANIZATION- The right of the employees to self-organization is a compelling reason why their withdrawal from the cooperative must be allowed. As pointed out by CURE, the resignation of the member- employees is an expression of their preference for union membership over that of membership in the cooperative. The avowed policy of the State to afford fall protection to labor and to promote the primacy of free collective bargaining mandates that the employees' right to form and join unions for purposes of collective bargaining be accorded the highest consideration. - Membership in an electric cooperative which merely vests in the member a right to vote during the annual meeting becomes too trivial and insubstantial vis-a-vis the primordial and more important constitutional right of an employee to join a union of his choice. Besides, the 390 employees of CENECO, some of whom have never been members of the cooperative, represent a very small percentage of the cooperative's total membership of 44,000. It is inconceivable how the withdrawal of a negligible number of members could adversely affect the business concerns and operations of CENECO.

No Right of Self-Organization: WORKER/MEMBER OF COOPERATIVE- It was held in Batangas I Electric Cooperative Labor Union vs. Romeo A. Young that "employees who at the same time are members of an electric cooperative are not entitled to form or join unions for purposes of collective bargaining agreement, for certainly an owner cannot bargain with himself or his co-owners."- However, nowhere in said case is it stated that member-employees are prohibited from withdrawing their membership in the cooperative in order to join a labor union.

2. NO- By virtue of EO 111, which became effective on March 4, 1987, the direct certification originally allowed under Article 257 of the Labor Code has apparently been discontinued as a method of selecting the exclusive bargaining agent of the workers. The mere fact that no opposition is made does not warrant a direct certification. The most effective way of determining which labor organization can truly represent the working force is by certification election.

Disposition The questioned order is hereby ANNULLED and SET ASIDE. The med-arbiter is hereby ordered to conduct a certification election among the rank-and- file employees of CENECO with CURE and No Union as the choices therein.

WORKERS WITH RIGHT OF SELF-ORGANIZATION

1.Worker QualificationsART. 277. Miscellaneous provisions. - (c) Any employee, whether employed for a definite period or not, shall, beginning on his first day of service, be considered as an employee for purposes of membership in any labor union. (As amended by Section 33, Republic Act No. 6715).

UST FACULTY UNION V. BITONIO, JR.318 SCRA 185

PANGANIBAN; November 16, 1999

NATURESpecial civil action in the Supreme Court. Certiorari.

FACTS- Union announced a general assembly to elect next union officers. - TRO was issued by med-arbiter enjoining them from conducting election.- UST held a general faculty assembly, attended by both union members and non-members. Here, appellants were elected as new union officers by acclamation and clapping of hands.- Appellees filed instant petition to seek injunctive relief and to nullify results of election.- Bitonio upheld med-arbiter and said election was void. He rejected contention that it was a legitimate exercise of right to self organization

ISSUE/SBasis of right to self-organization (p5 of outline) / Workers with right of self-organization (p6 of outline) HELDRatio Self-organization is a fundamental right to form, join or assist labor organizations for collective bargaining, mutual aid and protection. Whether employed for a definite period or not, employee shall be considered as such, beginning on 1 st day of

service, for purposes of membership in a labor union. Corollary to this right is the prerogative not to join. Reasoning The election can’t be considered as exercise of right to self-organization because the petitioners’ frustration over the performance of the respondents could not justify the method they chose to impose their will on the union.

1. All EmployeesART. 243. Coverage and employees’ right to self-organization. - All persons employed in commercial, industrial and agricultural enterprises and in religious, charitable, medical, or educational institutions, whether operating for profit or not, shall have the right to self-organization and to form, join, or assist labor organizations of their own choosing for purposes of collective bargaining. Ambulant, intermittent and itinerant workers, self-employed people, rural workers and those without any definite employers may form labor organizations for their mutual aid and protection. (As amended by Batas Pambansa Bilang 70, May 1, 1980).

Non-Profit OrganizationFEU v TRAJANO(FEU-AFW)

152 SCRA 725PARAS, July 31, 1987

NATUREpetition for certiorari seeking to annul and set aside the decision affirming the Order of the Med-Arbiter for the holding of a certification election among the rank and file employees of FEU

FACTSFEU was a non-stock, non-profit medical institution and before 1980, workers of these institutions were not allowed to form, join or organize labor unions under Article 244 of the LC. So when the FEU Alliance of Filipino Workers (FEU-AFW) filed a petition for consent and/or certification election and was denied, they assailed the constitutionality of Article 244 with the SC. But with BP70 which amended the said provision by granting even employees of non-stock, non-profit institutions the right to form, join and organize labor unions of their choice, the respondent union filed again for certification election and was granted by the Med Arbiter, and was affirmed by the respondent director Trajano. FEU now assails the validity of the order by Director Trajano.

ISSUES1. WON workers of non-stock, non-profit institutions could now form, join and organize labor unions of their choice (focus in the outline)2. WON Director Trajano gravely abused his discretion in granting the petition for certification election despite the pendency of a similar petition

HELD1. YESRatio. Article 244 of the Labor Code was already amended by BP7 which provides that rank and file employees of non-profit medical institutions were now permitted to form, organize or join labor unions of their choice for purposes of collective bargaining. Since private respondent had complied with the requisites provided by law for calling a certification election, it was incumbent upon respondent Director to conduct such certification election to ascertain the bargaining representative of petitioner's employees.2. NORatio. In order that the pendency of another action between the same parties for the same cause may be availed of as a ground to dismiss a case, there must be, between the action under consideration and the other action: (1) identity of parties, or at least such as representing the same interest in both actions; (2) identity of rights asserted and relief prayed for, the relief being founded on the same facts; and (3) the identity on the two preceding particulars should be such that any judgment which may be rendered on the other action will, regardless of which party is successful, amount to res judicata in the action under consideration. Reasoning. Any judgment which may be rendered in the petition for certiorari pending before the SC will not constitute res judicata in the petition for certification election under consideration, for while in the former, private respondent questioned the constitutionality of Article 244 of the Labor Code before its amendment, in the latter, private respondent invokes the same article as already amended.

Disposition. WHEREFORE, this petition is DISMISSED, and the decision appealed from is hereby AFFIRMED.

Members of Religious GroupVICTORIANO vs ELIZALDE ROPE WORKERS' UNION

59 SCRA 54ZALDIVAR; September 12, 1974

Page 16: 33994637 Labor Rounds 1and2

Labor Law 2 A2010 - 16 - DisiniFACTS- Victoriano was an employee of the Elizalde Rope Factory, Inc. and was a member of the Elizalde Rope Workers' Union. The Union had with the Company a CBA containing a closed shop agreement, by virtue of which, if any person, regardless of his religious beliefs, wishes to be employed or to keep his employment, he must become a member of the collective bargaining union. - In 1961, RA No. 3350 was enacted (amending the industrial peace act), providing that a closed shop agreement shall not cover members of any religious sects which prohibit affiliation of their members in any such labor organization.- Being a member of a member of the religious sect "Iglesia ni Cristo" that prohibits the affiliation of its members with any labor organization, Victoriano resigned from the union. Thereupon, the Union asked the Company to separate Victoriano from the service.

ISSUEWON RA No. 3350 infringes on the freedom of association and discriminates in favor of members of the INC (hence, unconstitutional)

HELD NO.RA No. 3350 introduced an exception to the closed shop agreement under the Industrial Peace Act. It excluded ipso jure from the application and coverage of the closed shop agreement the employees belonging to any religious sects which prohibit affiliation of their members with any labor organization. What the exception provides is that members of said religious sects cannot be compelled or coerced to join labor unions even when said unions have closed shop agreements with the employers; that in spite of any closed shop agreement, members of said religious sects cannot be refused employment or dismissed from their jobs on the sole ground that they are not members of the collective bargaining union. It is clear, therefore, that the assailed Act, far from infringing the constitutional provision on freedom of association, upholds and reinforces it. It does not prohibit the members of said religious sects from affiliating with labor unions. It still leaves to said members the liberty and the power to affiliate, or not to affiliate, with labor unions. If, notwithstanding their religious beliefs, the members of said religious sects prefer to sign up with the labor union, they can do so. If in deference and fealty to their religious faith, they refuse to sign up, they can do so; the law does not coerce them to join; neither does the law prohibit them from joining; and neither may the employer or labor union compel them to join. Republic Act No. 3350, therefore, does not violate the constitutional provision on freedom of association.The purpose sought to be achieved by RA 3350 was to insure freedom of belief and religion, and to promote the general welfare by preventing discrimination against those members of religious sects which prohibit their members from joining labor unions, confirming thereby their natural, statutory and constitutional right to work, the fruits of which work are usually the only means whereby they can maintain their own life and the life of their dependents. It cannot be gainsaid that said purpose is legitimate.The purpose of RA 3350 is secular, worldly, and temporal, not spiritual or religious or holy and eternal. It was intended to serve the secular purpose of advancing the constitutional right to the free exercise of religion, by averting that certain persons be refused work, or be dismissed from work, or be dispossessed of their right to work and of being impeded to pursue a modest means of livelihood, by reason of union security agreements. To help its citizens to find gainful employment whereby they can make a living to support themselves and their families is a valid objective of the state. The primary effects of the exemption from closed shop agreements in favor of members of religious sects that prohibit their members from affiliating with a labor organization, is the protection of said employees against the aggregate force of the collective bargaining agreement, and relieving certain citizens of a burden on their religious beliefs; and by eliminating to a certain extent economic insecurity due to unemployment, which is a serious menace to the health, morals, and welfare of the people of the State, the Act also promotes the well-being of society. It is our view that the exemption from the effects of closed shop agreement does not directly advance, or diminish, the interests of any particular religion. Although the exemption may benefit those who are members of religious sects that prohibit their members from joining labor unions, the benefit upon the religious sects is merely incidental and indirect. The "establishment clause" (of religion) does not ban regulation on conduct whose reason or effect merely happens to coincide or harmonize with the tenets of some or all religions. The free exercise clause of the Constitution has been interpreted to require that religious exercise be preferentially aided. To compel persons to join and remain members of a union to keep their jobs in violation of their religious scrupples, would hurt, rather than help, labor unions, Congress has seen it fit to exempt religious objectors lest their resistance spread to other workers, for religious objections have contagious potentialities more than political and philosophic objections.Furthermore, let it be noted that coerced unity and loyalty even to the country, and a fortiori to a labor union assuming that such unity and loyalty can be attained through coercion is not a goal that is constitutionally obtainable at the expense of religious liberty. A desirable end cannot be promoted by prohibited means.

KAPATIRAN SA MEAT AND CANNING V. CALLEJA162 SCRA 367

HELD: In Victoriano v Elizalde Rope Workers Union, the refusal of the members of the INC sect not to join a labor union for being contrary to their religious beliefs does not bar the members of the

sect from forming their own union. The recognition of the tenets of the sect should not infringe on the basic right of self-organizationgranted by the Constitution to workers, regardless of religious affiliation.

2. Government Corporation EmployeesART. 244. Right of employees in the public service. - Employees of government corporations established under the Corporation Code shall have the right to organize and to bargain collectively with their respective employers. All other employees in the civil service shall have the right to form associations for purposes not contrary to law. (As amended by Executive Order No. 111, December 24, 1986).

3. SupervisorsART. 245. Ineligibility of managerial employees to join any labor organization; right of supervisory employees. - Managerial employees are not eligible to join, assist or form any labor organization. Supervisory employees shall not be eligible for membership in a labor organization of the rank-and-file employees but may join, assist or form separate labor organizations of their own. (As amended by Section 18, Republic Act No. 6715, March 21, 1989).

RightFILOIL REFINERY CORPORATION vs.

FILOIL SUPERVISORY & CONFIDENTIAL EMPLOYEES ASSOCIATION46 SCRA 512

TEEHANKEE; Aug 18, 1972

NATUREAppeal from the orders of the Court of Industrial Relations

FACTS- Filoil Refinery Corporation executed a collective bargaining agreement with the Filoil Employees & Workers Association (FEWA), a labor association composed of the corporation's rank-and-file employees . This collective bargaining agreement expressly excluded from its coverage petitioner's supervisory and confidential employees, who in turn organized their own labor association, respondent herein.- The Corporation filed a motion to dismiss the petition for certification of the respondent association as the sole and exclusive collective bargaining agent of all petitioner’s supervisory and confidential employees working at its refinery in Rosario, Cavite. Their reason being, since they are part of the management, they do not have the right to bargain collectively although they may organize an organization of their own.

ISSUEWON supervisors 'shall have the right to self-organization, and to form, join or assist labor organizations of their own choosing for the purpose of collective bargaining

HELDYES.Reasoning As stated for the Court by the now Chief Justice in AG & P Co. of Manila, Inc. vs. C.I.R., 8 section 3 of the Industrial Peace Act "explicitly provides that 'employees' and this term includes supervisors 'shall have the right to self-organization, and to form, join or assist labor organizations of their own choosing for the purpose of collective bargaining through representations of their own choosing and to engage in concerted activities for the purpose of collective bargaining and other mutual aid or protection' and that 'individuals employed as supervisors . . . may form separate organizations of their own'. Indeed, it is well settled that 'in relation to his employer,' a foreman or supervisor 'is an employee within the meaning of the Act' . . . For this reason, supervisors are entitled to engage in union activities and any discrimination against them by reason thereof constitutes an unfair labor practice."

Supervisors and confidential employees, even though they may exercise the prerogatives of management as regards the rank and file employees are indeed employees in relation to their employer, the company which is owned by the stockholders and bondholders (capital) and should therefore be entitled under the law to bargain collectively with the top management with respect to their terms and conditions of employment.

DISPOSITIONPetition DISMISSED.

STA ROSA COCA-COLA PLANT EMPLOYEES UNION (union), ET AL V COCA-COLA BOTTLERS PHILS INC (company)

312 SCRA 437

Page 17: 33994637 Labor Rounds 1and2

Labor Law 2 A2010 - 17 - DisiniCALLEJO, SR; January 24, 2007

NATUREThis is a petition for review on certiorari of the Decision of the CA, which affirmed the ruling of the NLRC and the Labor Arbiter

FACTS- The union is the sole and exclusive bargaining representative of the regular paid workers and the manthly paid non-ccommission earning employees of the comopany. individual petitioners are union officers,directors and shop stewards. - The union and the company entered into a 3 year CBA. upon the expiration, the union told the company that they wanted to negotiate the terms. the union insisted that representatives from Alyansa ng mga Unyon ng Coca Cola be allowed to observe the CBA meetings. the company refused to allow alyansa to observe and an impasse ensued. union officers, directors and stewards filed a notice of strrice with NCMB based on deadlock on CBA and unfair labor practice arising from the company's refusal to bargain. the grounds were ammended to unfair labor practice for the company's refusal to bargain in good faith and interference with the exercise if their right to self-organization. - pending the notice to strike, the union decided to participate in a mass action by alyansa in front of the company's premises. operations would come to complete stops for insufficiency of contractual employees who would take over. after the mass strike (separate and distinct from the mass action), the company filed to declare strike illegal, to declare the officers of union and individual respondents to have lost their employment status, to declare the union, its officers and members guilty of unfair labor practice to violation of the CBA, and to award them damages.- the LA found the strike to be illegal such that the participants lost their employment status. the CA affirmed the decision.

ISSUE/S1. WON the mass action wit Alyansa is actually a strike

HELD1. YESRatio The factual findings and conclusions of tribunals, as long as based on substantial evidence, are conclusive on the SC. Reasoning The term strike2 encompasses not only concerted work stoppages, but also slowdowns, mass leaves, sit-downs, attempts to damage, destroy or sabotage plant equipment and facilities, and similar activities. Picketing involves merely the marching to and fro at the premises of the employer, usually accompanied by the display of placards and other signs making known the facts involved in a labor dispute. That there was a labor dispute between the parties is not an issue. Petitioners notified the respondent of their intention to stage a strike, and not merely to picket.

Disposition Petition is denied for lack of merit. CA decision is affirmed.

PENARADA VS BANGANGA PLYWOOD CORP. AND HUDSON CHUA

489 SCRA 95PANGANIBAN; May 3, 2006

NATUREPetition for review under rule 45 of the ROC

FACTS- Charlito Peñaranda was hired as a foreman/boiler/shift engineer of Baganga

Plywood Corporation (BPC) to take charge of the operations and maintenance of its steam plant boiler.

- He filed a complaint for illegal dismissal with money claims. He claims that he was not paid his overtime pay, premium pay for working during holidays/rest days, night shift differentials and finally claims for payment of damages and attorney’s fees having been forced to litigate the present complaint.

- BPC on the other hand claims that Penaranda’s the separation from service was due to the temporary closure due to repair and general maintenance of the company. When BPC partially reopened, Peñaranda failed to reapply. Hence, he was not terminated from employment much less illegally. He opted to severe employment when he insisted payment of his separation benefits. Furthermore, being a managerial employee he is not entitled to overtime pay and if ever he rendered services beyond the normal hours of work, [there] was no office order/or authorization for him to do so.

- LA found Penaranda to be entitled of the overtime pay and premium pay. NLRc, deleted the award because he was a managerial employee. CA dismissed Penaranda’s petition for certiorari.

ISSUE

2Article 212(o) of the Labor Code defines strike as a temporary stoppage of work by the

concerted action of employees as a result of an industrial or labor dispute.

1. WON Penaranda is a regular, common employee entitled to overtime pay and other monetary benefits.

HELD NO.

Managerial employees and members of the managerial staff are exempted from the provisions of the Labor Code on labor standards. Since petitioner belongs to this class of employees, he is not entitled to overtime pay and premium pay for working on rest days.The Court disagrees with the NLRC’s finding that the petitioner was a managerial employee. However, he was a member of the managerial staff, which also takes him out of the coverage of labor standards. Like managerial employees, officers and members of the managerial staff are not entitled to the provisions of law on labor standards.Article 82 of the Labor Code exempts managerial employees from the coverage of labor standards. Labor standards provide the working conditions of employees, including entitlement to overtime pay and premium pay for working on rest days. Under this provision, managerial employees are "those whose primary duty consists of the management of the establishment in which they are employed or of a department or subdivision." (1) Their primary duty consists of the management of the establishment in which they are employed or of a department or subdivision thereof; (2) They customarily and regularly direct the work of two or more employees therein; (3) They have the authority to hire or fire other employees of lower rank; or their suggestions and recommendations as to the hiring and firing and as to the promotion or any other change of status of other employees are given particular weight.Penaranda’s duty as a shift engineer, particularly, ((1)To supply the required and continuous steam to all consuming units at minimum cost, (2) To supervise, check and monitor manpower workmanship as well as operation of boiler and accessories, (3) To evaluate performance of machinery and manpower, (5) To train new employees for effective and safety while working) illustrates that petitioner was a member of the managerial staff. His duties and responsibilities conform to the definition of a member of a managerial staff under the Implementing Rules. He supervised the engineering section of the steam plant boiler. His work involved overseeing the operation of the machines and the performance of the workers in the engineering section. This work necessarily required the use of discretion and independent judgment to ensure the proper functioning of the steam plant boiler. As supervisor, petitioner is deemed a member of the managerial staff. On the basis of the foregoing, the Court finds no justification to award overtime pay and premium pay for rest days to petitioner.

DispositionPetition is denied.

TEST

DUNLOP SLAZENGER V SEC. OF LABOR (RUIZ)300 SCRA 120

PUNO; DECEMBER 11, 1998

NATUREPetition for certiorari

FACTS- Respondent union filed a Petition for Certification Election among the supervisory, office and technical employees of the petitioner company before the DOLE, Regional Office No. III.- Petitioner company filed a motion to dismiss based on 1) that the respondent union is comprised of supervisory and rank-and-file employees and cannot act as bargaining agent for the proposed unit; (2) that a single certification election cannot be conducted jointly among supervisory and rank-and-file employees; and (3) that the respondent union lacks legal standing since it failed to submit its books of accounts.- Respondent alleges that it is composed only of supervisory employees and that it has no obligation to attach its books of accounts since it is a legitimate labor organization.- The mediator arbiter granted the petition of the union. It said that the contention of the respondent that the petitioning union is composed of both supervisory and rank and file employees is not sufficient to dismiss the petition. It can be remedied thru the exclusion-inclusion proceedings wherein those employees who are occupying rank and file positions will be excluded from the list of eligible voters. The secretary of labor affirmed.(as regards the TEST)The TEST of supervisory status as we have repeatedly ruled is:-Whether an employee possesses authority to act in the interest of his employer,- Which authority should not be merely routinary or clerical in nature- But requires the use of independent judgment.- Corollary, what determines the nature of

Page 18: 33994637 Labor Rounds 1and2

Labor Law 2 A2010 - 18 - Disiniemployment is not the employee's title, but hisjob description.

- It is not decisive that these employees are monthly paid employees. Their mode of compensation is usually a matter of convenience and does not necessarily determine the nature and character of their job.

PAPER INDUSTRIES CORP. OF THE PHILS. V LAGUESMA330 SCRA 295

DE LEON; April 12, 2000

NATUREPetition for certiorari seeking to annul the Resolution and the Order Laguesma, acting then as Undersecretary, now the Secretary, of the DOLE

FACTS- PICOP-Bislig Supervisory and Technical Staff Employees Union (PBSTSEU) instituted a Petition for Certification Election to determine the sole and exclusive bargaining agent of the supervisory and technical staff employees of PICOP for CBA purposes.- Federation of Free Workers (FFW) and Associated Labor Union (ALU) filed their respective petitions for intervention.- Med-Arbiter granted the petitions for interventions of the FFW and ALU. Another Order set the holding of a certification election among PICOP's supervisory and technical staff employees in Tabon, Bislig, Surigao del Sur, with four (4) choices, namely: (1) PBSTSEU; (2) FFW; (3) ALU; and (4) no union.- PICOP appealed the Order which set the holding of the certification election contending that the Med-Arbiter committed grave abuse of discretion in deciding the case without giving PICOP the opportunity to file its comments/answer, and that PBSTSEU had no personality to file the petition for certification election.- DOLE Sec. Drilon upheld the Med-Arbiter's Order with modification allowing the supervising and staff employees in Cebu, Davao and Iligan City to participate in the certification election.- PICOP questioned and objected to the inclusion of some section heads and supervisors in the list of voters whose positions it averred were reclassified as managerial employees in the light of the reorganization effected by it.

ISSUEWON the positions Section Heads and Supervisors, who have been designated as Section Managers and Unit Managers who were converted to managerial employees are ineligible for union membership

HELD- NO. The mere fact that an employee is designated “manager” does not ipso facto make him one. Designation should be reconciled with the actual job description of the employee, for it is the job description that determines the nature of employment.- Managerial employees are ranked as Top Managers, Middle Managers and First Line Managers. Top and Middle Managers have the authority to devise, implement and control strategic and operational policies while the task of First-Line Managers is simply to ensure that such policies are carried out by the rank-and- file employees of an organization. - A thorough dissection of the job description of the concerned supervisory employees and section heads indisputably show that they are not actually managerial but only supervisory employees since they do not lay down company policies. Any authority they exercise is not supreme but merely advisory in character. Theirs is not a final determination of the company policies inasmuch as any action taken by them on matters relative to hiring, promotion, transfer, suspension and termination of employees is still subject to confirmation and approval by their respective superior.

Disposition WHEREFORE, the petition is hereby DISMISSED, and the Resolution and Order of public respondent Bienvenido E. Laguesma dated April 17, 1991 and August 17, 1991, respectively, finding the subject supervisors and section heads as supervisory employees eligible to vote in the certification election are AFFIRMED.

Samson vs NLRC (Schering-Plough Corp)330 SCRA 460

Kapunan, J.; April 12, 2000

NATURESpecial civil action of certiorari

FACTS- Rufino Norberto F. Samson was dismissed from the Company for uttering what was considered by the company as obscene, insulting, and offensive words and for making malicious and lewd gestures directed at the President and General Manager of the company during an informal Sales and Marketing gathering in relation to the decision of the Management Committee on a dispute with another employee. He was also accused of threatening to disrupt or create violence in a forthcoming National Sales Conference.- The Labor Arbiter found that Samson was illegally dismissed but the decision was

reversed by NLRC.

ISSUE/SWON the dismissal on the ground of loss of confidence is valid

HELDNo. As a ground for dismissal, the term “trust and confidence” is restricted to managerial employees. And before one may be properly considered a managerial employee, three conditions must be met:

a. Their primary duty consists of the management of the establishment in which they are employed or of a department or subdivision thereof:

b. They customarily and regularly direct the work of two or more employees therein;

c. They have the authority to hire or fire other employees of lower rank; or their suggestions and recommendations as to hiring an firing and as to the promotion or any other change of status of other employees are given particular weight.

-In this case the job description of Samson does not mention that petitioner possesses the power “to lay down policies nor to hire, transfer, suspend, lay off, recall. discharge, assign or discipline employees”. Absent this crucial element, petitioner cannot be considered a managerial employee despite his designation as District Sales Manager. Disposition Petition is granted. NLRC decision is reversed and set aside. Samson is reinstated to his position without loss of seniority rights and is awarded payment of his full backwages.

TAGAYTAY HIGHLANDS INTERNATIONAL GOLF CLUB INC V TAGAYTAY HIGHLANDS EMPLOYEES UNION-PGTWO

395 SCRA 699CARPIO-MORALES; January 22, 2003

NATUREPetition for Certiorari under Rule 45 THIGCI assailing CA decision denying its petition to annul the Department of Labor and Employment (DOLE) Resolutions of November 12, 1998 and December 29, 1998

FACTS- October 16, 1997 > Tagaytay Highlands Employees Union (THEU), Philippine Transport and General Workers Organization (PTGWO), Local Chapter No. 776, a legitimate labor organization said to represent majority of the rank-and-file employees of THIGCI, filed a petition for certification election before the DOLE Mediation-Arbitration Unit, Regional Branch No. IV- November 27, 1997 > opposed petition for certification election because the list of union members submitted by it was defective and fatally flawed as it included the names and signatures of supervisors, resigned, terminated and absent without leave (AWOL) employees, as well as employees of The Country Club, Inc., a corporation distinct and separate from THIGCI; and that out of the 192 signatories to the petition, only 71 were actual rank-and-file employees of THIGCI. Also, some of the signatures in the list of union members were secured through fraudulent and deceitful means, and submitted copies of the handwritten denial and withdrawal of some of its employees from participating in the petition.- THEU asserted that it complied with all the requirements for valid affiliation and inclusion in the roster of legitimate labor organizations pursuant to DOLE Department Order No. 9, series of 1997, on account of which it was duly granted a Certification of Affiliation by DOLE on October 10, 1997; and that Section 5, Rule V of said Department Order provides that the legitimacy of its registration cannot be subject to collateral attack, and for as long as there is no final order of cancellation, it continues to enjoy the rights accorded to a legitimate organization. Therefore, the Med-Arbiter should, pursuant to Article 257 of the Labor Code and Section 11, Rule XI of DOLE Department Order No. 09, automatically order the conduct of a certification election. - January 28, 1998 > DOLE Med-Arbiter Anastacio Bactin ordered the holding of a certification election - DOLE Resolution of November 12, 19981 > setting aside the June 4, 1998 Resolution dismissing the petition for certification election. MFR denied - CA - denied THIGCI’s Petition for Certiorari and affirmed the DOLE Resolution dated November 12, 1998. It held that while a petition for certification election is an exception to the innocent bystander rule, hence, the employer may pray for the dismissal of such petition on the basis of lack of mutuality of interests of the members of the union as well as lack of employer-employee relationship and petitioner failed to adduce substantial evidence to support its allegations.

ISSUES1. WON the inclusion of the supervisors should affect the result of the certification election

Page 19: 33994637 Labor Rounds 1and2

Labor Law 2 A2010 - 19 - DisiniHELD1. NOReasoning Article 245 of the Labor CodeArticle 245. Ineligibility of managerial employees to join any labor organization; right of supervisory employees. Managerial employees are not eligible to join, assist or form any labor organization. Supervisory employees shall not be eligible for membership in a labor organization of the rank-and-file employees but may join, assist or form separate labor organizations of their own.- As this Court put it in Pepsi-Cola Products Philippines, Inc. v. Secretary of Labor:Designation should be reconciled with the actual job description of subject employees x x x The mere fact that an employee is designated manager does not necessarily make him one. Otherwise, there would be an absurd situation where one can be given the title just to be deprived of the right to be a member of a union. In the case of National Steel Corporation vs. Laguesma (G. R. No. 103743, January 29, 1996), it was stressed that:What is essential is the nature of the employee’s function and not the nomenclature or title given to the job which determines whether the employee has rank-and-file or managerial status or whether he is a supervisory employee.

Disposition Petition is DENIED. Let the records of the case be remanded to the office of origin, the Mediation-Arbitration Unit, Regional Branch No. IV, for the immediate conduct of a certification election subject to the usual pre-election conference.

CAINTA CATHOLIC SCHOOL V CAINTA CATHOLIC SCHOOL EMPLOYEES’ UNION

489 SCRA 468TINGA; May 4, 2006

NATURESpecial civil action of certiorari

FACTS- the school and union entered into a CBA. Then the union in the school became inactive for several years. When it became active again, led by Llagas (Pres) and Javier (VP), the school decided to enforce the provision on the CBA re: retirement (that management can retire an employee if s/he has reached age of 60 or rendered at least 20 years of service, the last 3 years continuous). The school retired Llagas and Javier, both having rendered more than 20 years of continuous service. The union struck, alleging unfair labor practice and unlawful termination. The school countered with a petition to declare the strike illegal.- school claims that it could validly retire Llagas and Javier, at its prerogative granted to it by law and the CBA. Union claims that retirement was equivalent to union busting, as the dismissed EEs were their leaders. - NLRC ruled in favor of the school, declaring that the management prerogative on retirement of EEs was validly exercised. Also, it gave, as another justification for the retirement, the reason that as managerial employees (LLagas was the Dean of Student Affairs while Javier was the Subject Area Coordinator), when they joined and became officers of the union. They lost the trust and confidence of the school. They allowed their loyalties be divided between the union and the school. - appeal of the union to the CA was granted, saying that the retirement of the 2 union officers was a mere subterfuge to bust the union. The school then filed this petition.

ISSUE/SWON Llagas and Javier were managerial EES, thus making the strike called by the union on their behalf (as unqualified union members) illegal

HELDYESRatio The 2 employees are not rank and file employees, thus disqualified from joining a union. Thus, the strike called in their behalf is illegal. Reasoning Art.212(m) defines a managerial employee as: one who is vested with powers or prerogatives to lay down and execute management policies and/or to hire, transfer, suspend, lay-off, recall, discharge, assign or discipline employees, or to effectively recommend such managerial actions. On the other hand, Supervisory EEs are those who, in the interest of the ER, effectively recommend such managerial actions if the exercise of such authority is not merely routinary or clerical in nature but requires the use of independent judgment.- court reviewed the functions of the positions of the retired EEs found in the Faculty Manual in order to determine their classification. As the Dean of Student Affairs, Llagas was clearly a managerial employee. As a Subject Area Coordinator, Javier falls under supervisory EEs - thus, Llagas is proscribed from joining a labor union, more so being elected as union officer. In the case of Javier, a supervisory EE, she may join a labor union composed only of supervisory employees. Finding both union officers to be employees not belonging to the rank-and-file, their membership in the Union has become questionable, rendering the Union inutile to represent their cause.Disposition Petition is granted. NLRC resolution reinstated.

PENARADA VS BANGANGA PLYWOOD CORP. AND HUDSON CHUA

489 SCRA 95PANGANIBAN; May 3, 2006

NATUREPetition for review under rule 45 of the ROC

FACTS- Charlito Peñaranda was hired as a foreman/boiler/shift engineer of Baganga

Plywood Corporation (BPC) to take charge of the operations and maintenance of its steam plant boiler.

- He filed a complaint for illegal dismissal with money claims. He claims that he was not paid his overtime pay, premium pay for working during holidays/rest days, night shift differentials and finally claims for payment of damages and attorney’s fees having been forced to litigate the present complaint.

- BPC on the other hand claims that Penaranda’s the separation from service was due to the temporary closure due to repair and general maintenance of the company. When BPC partially reopened, Peñaranda failed to reapply. Hence, he was not terminated from employment much less illegally. He opted to severe employment when he insisted payment of his separation benefits. Furthermore, being a managerial employee he is not entitled to overtime pay and if ever he rendered services beyond the normal hours of work, [there] was no office order/or authorization for him to do so.

- LA found Penaranda to be entitled of the overtime pay and premium pay. NLRc, deleted the award because he was a managerial employee. CA dismissed Penaranda’s petition for certiorari.

ISSUEWON Penaranda is a regular, common employee entitled to overtime pay and other monetary benefits.

HELD NO.- Managerial employees and members of the managerial staff are exempted from

the provisions of the Labor Code on labor standards. Since petitioner belongs to this class of employees, he is not entitled to overtime pay and premium pay for working on rest days.

- The Court disagrees with the NLRC’s finding that the petitioner was a managerial employee. However, he was a member of the managerial staff, which also takes him out of the coverage of labor standards. Like managerial employees, officers and members of the managerial staff are not entitled to the provisions of law on labor standards.

- Article 82 of the Labor Code exempts managerial employees from the coverage of labor standards. Labor standards provide the working conditions of employees, including entitlement to overtime pay and premium pay for working on rest days. Under this provision, managerial employees are "those whose primary duty consists of the management of the establishment in which they are employed or of a department or subdivision." (1) Their primary duty consists of the management of the establishment in which they are employed or of a department or subdivision thereof; (2) They customarily and regularly direct the work of two or more employees therein; (3) They have the authority to hire or fire other employees of lower rank; or their suggestions and recommendations as to the hiring and firing and as to the promotion or any other change of status of other employees are given particular weight.

- Penaranda’s duty as a shift engineer, particularly, ((1)To supply the required and continuous steam to all consuming units at minimum cost, (2) To supervise, check and monitor manpower workmanship as well as operation of boiler and accessories, (3) To evaluate performance of machinery and manpower, (5) To train new employees for effective and safety while working) illustrates that petitioner was a member of the managerial staff. His duties and responsibilities conform to the definition of a member of a managerial staff under the Implementing Rules. He supervised the engineering section of the steam plant boiler. His work involved overseeing the operation of the machines and the performance of the workers in the engineering section. This work necessarily required the use of discretion and independent judgment to ensure the proper functioning of the steam plant boiler. As supervisor, petitioner is deemed a member of the managerial staff.

- On the basis of the foregoing, the Court finds no justification to award overtime pay and premium pay for rest days to petitioner.

4. AliensART. 269. Prohibition against aliens; exceptions. - All aliens, natural or juridical, as well as foreign organizations are strictly prohibited from engaging directly or indirectly in all forms of trade union activities without prejudice to normal contacts between Philippine labor unions and recognized international labor centers:

Page 20: 33994637 Labor Rounds 1and2

Labor Law 2 A2010 - 20 - DisiniProvided, however, That aliens working in the country with valid permits issued by the Department of Labor and Employment, may exercise the right to self-organization and join or assist labor organizations of their own choosing for purposes of collective bargaining: Provided, further, That said aliens are nationals of a country which grants the same or similar rights to Filipino workers. (As amended by Section 29, Republic Act No. 6715, March 21, 1989).

5. Security GuardsMANILA ELECTRIC COMPANY V SECRETARY OF LABOR.

197 SCRA 275MEDIALDEA; May 20, 1991

NATURE: Petition for review

FACTS:- The Staff and Technical Employees Association of MERALCO filed a petition for certification election to represent regular employees of MERALC0 who are: (a) non managerial employees with Pay Grades VII and above; (b) non-managerial employees in the Patrol Division, Treasury Security Services Section. Secretaries who are automatically removed from the bargaining unit; and (c) employees within the rank and file unit who are automatically disqualified from becoming union members of any organization within the same bargaining unit.MERALCO moved for dismissal of the petition on ground that some of the employees they sought to represent are security services personnel who are prohibited from joining or assisting the rank-and-file union

ISSUE:WON Security Guards may join a rank and file organization

HELD:-Pres. Corazon C. Aquino issued E.O. No. 111 which eliminated the disqualification of security guards. While therefore under the old rules, security guards were barred from joining a labor organization of the rank and file, under RA 6715, they may now freely join a labor organization of the rank and file or that of the supervisory union, depending on their rank. By accommodating supervisory employees, the Secretary of Labor must likewise apply the provisions of RA 6715 to security guards by favorably allowing them free access to a labor organization, whether rank and file or supervisory, in recognition of their constitutional right to self-organization.- The implementing rules of RA 6715, therefore, insofar as they disqualify security guards from joining a rank and file organization are null and void, for being not germane to the object and purposes of EO 111 and RA 6715 upon which such rules purportedly derive statutory moorings.

WORKERS WITH NO RIGHT OF SELF-ORGANIZATION

1. MANAGERIAL AND CONFIDENTIAL EMPLOYEES

SEC. 8. Article 245 of the Labor Code is hereby amended to read as follows: “ART. 245. Ineligibility of Managerial Employees to Join any Labor

Organization; Right of Supervisory Employees. - Managerial employees are not eligible to join, assist or form any labor organization. Supervisory employees shall not be eligible for membership in the collective bargaining unit of the rank-and-file employees but may join, assist or form separate collective bargaining units and/or legitimate labor organizations of their own. The rank and file union and the supervisors’ union operating within the same establishment may jointhe same federation or national union.”

TESTSAN MIGUEL CORPORATION SUPERVISORS AND EXEMPT

UNION, VS. LAGUESMA277 SCRA 370 (1997)

ROMERO, J.:

NATURE Petition for Certiorari

FACTS-On October 5, 1990, petitioner union filed before the Department of Labor and Employment (DOLE) a Petition for Direct Certification or Certification Election among the supervisors and exempt employees of the SMC Magnolia Poultry Products Plants of Cabuyao, San Fernando and Otis.-On December 19, 1990, Med-Arbiter Danilo L. Reynante issued an Order ordering the conduct of certification election among the supervisors and exempt employees of the SMC Magnolia Poultry Products Plants of Cabuyao, San Fernando and Otis as ONE BARGAINING UNIT.

-On January 18, 1991, respondent San Miguel Corporation appealed pointing out, among others, the Med-Arbiter's error in grouping together all three (3) separate plants, Otis, Cabuyao and San Fernando, into one bargaining unit, and in including SUPERVISORY LEVELS 3 AND ABOVE WHOSE POSITIONS ARE CONFIDENTIAL IN NATURE.-Upon petitioner-union's motion dated August 7, 1991, Undersecretary Laguesma granted the reconsideration prayed for on September 3, 1991 and directed the conduct of separate certification elections among the supervisors ranked as supervisory levels 1 to 4 (S1 to S4) and the exempt employees in each of the three plants at Cabuyao, San Fernando and Otis.

ISSUES1. Whether Supervisory employees 3 and 4 and the exempt employees of the company are considered confidential employees, hence ineligible from joining a union.2. Whether or not it was proper for Usec Laguesma to consider that the employees of the three plants constitute an appropriate single bargaining unit.

HELD1. NO. RATIO- An employee of a labor union, or of a management association, must have access to confidential labor relations information with respect to his employer, the union, or the association, to be regarded a confidential employee, and knowledge of labor relations information pertaining to the companies with which the union deals, or which the association represents, will not cause an employee to be excluded from the bargaining unit representing employees of the union or association. - Access to information which is regarded by the employer to be confidential from the business standpoint, such as financial information or technical trade secrets, will not render an employee a confidential employee."

REASONING-Confidential employees are those who (1) assist or act in a confidential capacity, (2) to persons who formulate, determine, and effectuate management policies in the field of labor relations. The two criteria are cumulative, and both must be met if an employee is to be considered a confidential employee that is, the confidential relationship must exist between the employee and his supervisor, and the supervisor must handle the prescribed responsibilities relating to labor relations. -The exclusion from bargaining units of employees who, in the normal course of their duties, become aware of management policies relating to labor relations is a principal objective sought to be accomplished by the ''confidential employee rule." THE BROAD RATIONALE BEHIND THIS RULE IS THAT EMPLOYEES SHOULD NOT BE PLACED IN A POSITION INVOLVING A POTENTIAL CONFLICT OF INTERESTS. "Management should not be required to handle labor relations matters through employees who are represented by the union with which the company is required to deal and who in the normal performance of their duties may obtain advance information of the company's position with regard to contract negotiations, the disposition of grievances, or other labor relations matters."-AN IMPORTANT ELEMENT OF THE "CONFIDENTIAL EMPLOYEE RULE" IS THE EMPLOYEE'S NEED TO USE LABOR RELATIONS INFORMATION. Thus, in determining the confidentiality of certain employees, a key question frequently considered is the employee's necessary access to confidential labor relations information. -Granting arguendo that an employee has access to confidential labor relations information but such is merely incidental to his duties and knowledge thereof is not necessary in the performance of such duties, said access does not render the employee a confidential employee. - It must be borne in mind that Section 3 of Article XIII of the 1987 Constitution mandates the State to guarantee to "all" workers the right to self-organization. Hence, confidential employees who may be excluded from bargaining unit must be strictly defined so as not to needlessly deprive many employees of their right to bargain collectively through representatives of their choosing. -In the case at bar, supervisors 3 and above may not be considered confidential employees merely because they handle "confidential data" as such must first be strictly classified as pertaining to labor relations for them to fall under said restrictions. The information they handle are properly classifiable as technical and internal business operations data which, to our mind, has no relevance to negotiations and settlement of grievances wherein the interests of a union and the management are invariably adversarial.

2. NORATIO- An appropriate bargaining unit may be defined as "a group of employees of a given employer, comprised of all or less than all of the entire body of employees, which

Page 21: 33994637 Labor Rounds 1and2

Labor Law 2 A2010 - 21 - Disinithe collective interest of all the employees, consistent with equity to the employer, indicate to be best suited to serve the reciprocal rights and duties of the parties under the collective bargaining provisions of the law."

- A unit to be appropriate must effect a grouping of employees who have SUBSTANTIAL, MUTUAL INTERESTS IN WAGES, HOURS, WORKING CONDITIONS AND OTHER SUBJECTS OF COLLECTIVE BARGAINING.

REASONING

- It is readily seen that the employees in the instant case have "community or mutuality of interests," which is the standard in determining the proper constituency of a collective bargaining unit. It is undisputed that they all belong to the Magnolia Poultry Division of San Miguel Corporation. This means that, although they belong to three different plants, they perform work of the same nature, receive the same wages and compensation, and most importantly, share a common stake in concerted activities.

SUGBUANON RURAL BANK, INC. VS LAGUESMA 325 SCRA 425

QUISUMBING: February 2, 2000

NATURE:- Special civil action for certiorari and prohibition

FACTS:- Petitioner Sugbuanon Rural Bank, Inc., (SRBI, for brevity) is a duly-registered

banking institution with principal office in Cebu City and a branch in Mandaue City. Private respondent SRBI Association of Professional, Supervisory, Office, and Technical Employees Union (APSOTEU) is a legitimate labor organization affiliated with the Trade Unions Congress of the Philippines (TUCP).

- On October 8, 1993, the DOLE Regional Office in Cebu City granted Certificate of Registration to APSOTEU-TUCP, hereafter referred to as the union.

- On October 26, 1993, the union filed a petition for certification election of the supervisory employees of SRBI. It alleged, among others, that: (1) APSOTEU-TUCP was a labor organization duly-registered with the Labor Department; (2) SRBI employed 5 or more supervisory employees; (3) a majority of these employees supported the petition: (4) there was no existing collective bargaining agreement (CBA) between any union and SRBI; and (5) no certification election had been held in SRBI during the past 12 months prior to the petition.

- On November 12, 1993, SRBI filed a motion to dismiss the union's petition. It sought to prevent the holding of a certification election on two grounds. First, that the members of APSOTEU-TUCP were in fact managerial or confidential employees. Second, the Association of Labor Unions-Trade Unions Congress of the Philippines or ALU-TUCP was representing the union.

- The union filed its opposition to the motion to dismiss on December 1, 1993. It argued that its members were not managerial employees but merely supervisory employees.

- On December 9, 1993, the Med-Arbiter denied petitioner's motion to dismiss.- SRBI appealed the Med-Arbiter's decision to the Secretary of Labor and

Employment. The appeal was denied for lack of merit. The certification election was ordered.

- On June 16, 1994, the Med-Arbiter scheduled the holding of the certification election for June 29, 1994.

- On June 17, 1994, SRBI filed with the Med-Arbiter an urgent motion to suspend proceedings. The Med-Arbiter denied the same. SRBI then filed a motion for reconsideration. Two days later, the Med-Arbiter cancelled the certification election scheduled for June 29, 1994 in order to address the motion for reconsideration.

- The Med-Arbiter later denied petitioner's motion for reconsideration- SRBI appealed the order of denial to the DOLE Secretary- Petitioner proceeded to file a petition with the DOLE Regional Office seeking

the cancellation of the respondent union's registration. It averred that the APSOTEU-TUCP members were actually managerial employees who were prohibited by law from joining or organizing unions.

- DOLE Undersecretary denied SRBI's appeal for lack of merit. He ruled that APSOTEU-TUCP was a legitimate labor organization. It was fully entitled to all the rights and privileges granted by law to a legitimate labor organization, including the right to file a petition for certification election. He also held that until and unless a final order is issued cancelling APSOTEU-TUCP's registration certificate, it had the legal right to represent its members for collective bargaining purposes.

- SRBI moved for reconsideration of the Undersecretary's decision

ISSUE/S:

(1) WON the members of the respondent union are managerial employees and/or highly-placed confidential employees, hence prohibited by law from joining labor organizations and engaging in union activities(2) WON the Med-Arbiter may validly order the holding of a certification election

HELD:1. NO

Reasoning- Article 212 (m) of the Labor Code defines the terms "managerial employee" and "supervisory employees" as follows:

Art. 212. Definitions(m) "Managerial employee" is one who is vested with powers or prerogatives to lay down and execute management policies and/or hire, transfer, suspend, lay-off, recall, discharge, assign or discipline employees. Supervisory employees are those who, in the interest of the employer, effectively recommend such managerial actions if the exercise of such authority is not merely routinary or clerical in nature but requires the use of independent judgment. All employees not falling within any of the above definitions are considered rank-and-file employees for purposes of

- Petitioner submitted detailed job descriptions to support its contention that the union members are managerial employees and/or confidential employees proscribed from engaging in labor activities. In the present case, however, petitioner failed to show that the employees in question were vested with managerial powers. At best they only had recommendatory powers subject to evaluation, review, and final decision by the bank's management. The job description forms submitted by petitioner clearly show that the union members in question may not transfer, suspend, lay-off, recall, discharge, assign, or discipline employees. Moreover, the forms also do not show that the Cashiers, Accountants, and Acting Chiefs of the Loans Department formulate and execute management policies which are normally expected of management officers.2. YES Reasoning

- One of the rights of a legitimate labor organization under Article 242(b) of the Labor Code is the right to be certified as the exclusive representative of all employees in an appropriate bargaining unit for purposes of collective bargaining. Having complied with the requirements of Art. 234, it is our view that respondent union is a legitimate labor union. Article 257 of the Labor Code mandates that a certification election shall automatically be conducted by the Med-Arbiter upon the filing of a petition by a legitimate labor organization.

DISPOSITIVE:Petition dismissed.

PAPER INDUSTRIES CORP. OF THE PHILS. V LAGUESMA330 SCRA 295

DE LEON; April 12, 2000

NATUREPetition for certiorari seeking to annul the Resolution and the Order Laguesma, acting then as Undersecretary, now the Secretary, of the DOLE

FACTS- PICOP-Bislig Supervisory and Technical Staff Employees Union (PBSTSEU) instituted a Petition for Certification Election to determine the sole and exclusive bargaining agent of the supervisory and technical staff employees of PICOP for CBA purposes.- Federation of Free Workers (FFW) and Associated Labor Union (ALU) filed their respective petitions for intervention.- Med-Arbiter granted the petitions for interventions of the FFW and ALU. Another Order set the holding of a certification election among PICOP's supervisory and technical staff employees in Tabon, Bislig, Surigao del Sur, with four (4) choices, namely: (1) PBSTSEU; (2) FFW; (3) ALU; and (4) no union.- PICOP appealed the Order which set the holding of the certification election contending that the Med-Arbiter committed grave abuse of discretion in deciding the case without giving PICOP the opportunity to file its comments/answer, and that PBSTSEU had no personality to file the petition for certification election.- DOLE Sec. Drilon upheld the Med-Arbiter's Order with modification allowing the supervising and staff employees in Cebu, Davao and Iligan City to participate in the certification election.- PICOP questioned and objected to the inclusion of some section heads and supervisors in the list of voters whose positions it averred were reclassified as managerial employees in the light of the reorganization effected by it.

ISSUE

Page 22: 33994637 Labor Rounds 1and2

Labor Law 2 A2010 - 22 - DisiniWON the positions Section Heads and Supervisors, who have been designated as Section Managers and Unit Managers who were converted to managerial employees are ineligible for union membership

HELD- NO. The mere fact that an employee is designated “manager” does not ipso facto make him one. Designation should be reconciled with the actual job description of the employee, for it is the job description that determines the nature of employment.- Managerial employees are ranked as Top Managers, Middle Managers and First Line Managers. Top and Middle Managers have the authority to devise, implement and control strategic and operational policies while the task of First-Line Managers is simply to ensure that such policies are carried out by the rank-and- file employees of an organization. - A thorough dissection of the job description of the concerned supervisory employees and section heads indisputably show that they are not actually managerial but only supervisory employees since they do not lay down company policies. Any authority they exercise is not supreme but merely advisory in character. Theirs is not a final determination of the company policies inasmuch as any action taken by them on matters relative to hiring, promotion, transfer, suspension and termination of employees is still subject to confirmation and approval by their respective superior.DispositionWHEREFORE, the petition is hereby DISMISSED, and the Resolution and Order of public respondent Bienvenido E. Laguesma dated April 17, 1991 and August 17, 1991, respectively, finding the subject supervisors and section heads as supervisory employees eligible to vote in the certification election are AFFIRMED.

SAMSON VS NLRC (SCHERING-PLOUGH CORP)330 SCRA 460

Kapunan, J.; April 12, 2000

NATURESpecial civil action of certiorari

FACTS- Rufino Norberto F. Samson was dismissed from the Company for uttering what was considered by the company as obscene, insulting, and offensive words and for making malicious and lewd gestures directed at the President and General Manager of the company during an informal Sales and Marketing gathering in relation to the decision of the Management Committee on a dispute with another employee. He was also accused of threatening to disrupt or create violence in a forthcoming National Sales Conference.- The Labor Arbiter found that Samson was illegally dismissed but the decision was reversed by NLRC.

ISSUE/SWON the dismissal on the ground of loss of confidence is valid

HELDNo. As a ground for dismissal, the term “trust and confidence” is restricted to managerial employees. And before one may be properly considered a managerial employee, three conditions must be met:

a. Their primary duty consists of the management of the establishment in which they are employed or of a department or subdivision thereof:b. they customarily and regularly direct the work of two or more employees therein;c. They have the authority to hire or fire other employees of lower rank; or their suggestions and recommendations as to hiring an firing and as to the promotion or any other change of status of other employees are given particular weight.

In this case the job description of Samson does not mention that petitioner possesses the power “to lay down policies nor to hire, transfer, suspend, lay off, recall. discharge, assign or discipline employees”. Absent this crucial element, petitioner cannot be considered a managerial employee despite his designation as District Sales Manager. Disposition Petition is granted. NLRC decision is reversed and set aside. Samson is reinstated to his position without loss of seniority rights and is awarded payment of his full backwages

PAPER INDUSTRIES CORP. OF THE PHILS. V LAGUESMA330 SCRA 295

DE LEON; April 12, 2000

NATUREPetition for certiorari seeking to annul the Resolution and the Order Laguesma, acting then as Undersecretary, now the Secretary, of the DOLE

FACTS

- PICOP-Bislig Supervisory and Technical Staff Employees Union (PBSTSEU) instituted a Petition for Certification Election to determine the sole and exclusive bargaining agent of the supervisory and technical staff employees of PICOP for CBA purposes.- Federation of Free Workers (FFW) and Associated Labor Union (ALU) filed their respective petitions for intervention.- Med-Arbiter granted the petitions for interventions of the FFW and ALU. Another Order set the holding of a certification election among PICOP's supervisory and technical staff employees in Tabon, Bislig, Surigao del Sur, with four (4) choices, namely: (1) PBSTSEU; (2) FFW; (3) ALU; and (4) no union.- PICOP appealed the Order which set the holding of the certification election contending that the Med-Arbiter committed grave abuse of discretion in deciding the case without giving PICOP the opportunity to file its comments/answer, and that PBSTSEU had no personality to file the petition for certification election.- DOLE Sec. Drilon upheld the Med-Arbiter's Order with modification allowing the supervising and staff employees in Cebu, Davao and Iligan City to participate in the certification election.- PICOP questioned and objected to the inclusion of some section heads and supervisors in the list of voters whose positions it averred were reclassified as managerial employees in the light of the reorganization effected by it.

ISSUEWON the positions Section Heads and Supervisors, who have been designated as Section Managers and Unit Managers who were converted to managerial employees are ineligible for union membership

HELD- NO. The mere fact that an employee is designated “manager” does not ipso facto make him one. Designation should be reconciled with the actual job description of the employee, for it is the job description that determines the nature of employment.- Managerial employees are ranked as Top Managers, Middle Managers and First Line Managers. Top and Middle Managers have the authority to devise, implement and control strategic and operational policies while the task of First-Line Managers is simply to ensure that such policies are carried out by the rank-and- file employees of an organization. - A thorough dissection of the job description of the concerned supervisory employees and section heads indisputably show that they are not actually managerial but only supervisory employees since they do not lay down company policies. Any authority they exercise is not supreme but merely advisory in character. Theirs is not a final determination of the company policies inasmuch as any action taken by them on matters relative to hiring, promotion, transfer, suspension and termination of employees is still subject to confirmation and approval by their respective superior.- Thus, where such power, which is in effect RECOMMENDATORY in character, is SUBJECT TO EVALUATION, REVIEW, and FINAL ACTION by department heads and other higher executives of the company. The same, although present, is not effective and not an exercise of INDEPENDENT JUDGMENT as required by law.

Disposition WHEREFORE, the petition is hereby DISMISSED, and the Resolution and Order of public respondent Bienvenido E. Laguesma dated April 17, 1991 and August 17, 1991, respectively, finding the subject supervisors and section heads as supervisory employees eligible to vote in the certification election are AFFIRMED.

CAINTA CATHOLIC SCHOOL V CAINTA CATHOLIC SCHOOL EMPLOYEES’ UNION

489 SCRA 468TINGA; May 4, 2006

NATURESpecial civil action of certiorari

FACTS- the school and union entered into a CBA. Then the union in the school became inactive for several years. When it became active again, led by Llagas (Pres) and Javier (VP), the school decided to enforce the provision on the CBA re: retirement (that management can retire an employee if s/he has reached age of 60 or rendered at least 20 years of service, the last 3 years continuous). The school retired Llagas and Javier, both having rendered more than 20 years of continuous service. The union struck, alleging unfair labor practice and unlawful termination. The school countered with a petition to declare the strike illegal.- school claims that it could validly retire Llagas and Javier, at its prerogative granted to it by law and the CBA. Union claims that retirement was equivalent to union busting, as the dismissed EEs were their leaders. - NLRC ruled in favor of the school, declaring that the management prerogative on retirement of EEs was validly exercised. Also, it gave, as another justification for the retirement, the reason that as managerial employees (LLagas was the Dean of Student Affairs while Javier was the Subject Area Coordinator), when they joined

Page 23: 33994637 Labor Rounds 1and2

Labor Law 2 A2010 - 23 - Disiniand became officers of the union. They lost the trust and confidence of the school. They allowed their loyalties be divided between the union and the school. - appeal of the union to the CA was granted, saying that the retirement of the 2 union officers was a mere subterfuge to bust the union. The school then filed this petition.

ISSUE/SWON Llagas and Javier were managerial EES, thus making the strike called by the union on their behalf (as unqualified union members) illegal

HELDYESRatio The 2 employees are not rank and file employees, thus disqualified from joining a union. Thus, the strike called in their behalf is illegal. Reasoning Art.212(m) defines a managerial employee as: one who is vested with powers or prerogatives to lay down and execute management policies and/or to hire, transfer, suspend, lay-off, recall, discharge, assign or discipline employees, or to effectively recommend such managerial actions. On the other hand, Supervisory EEs are those who, in the interest of the ER, effectively recommend such managerial actions if the exercise of such authority is not merely routinary or clerical in nature but requires the use of independent judgment.- court reviewed the functions of the positions of the retired EEs found in the Faculty Manual in order to determine their classification. As the Dean of Student Affairs, Llagas was clearly a managerial employee. As a Subject Area Coordinator, Javier falls under supervisory EEs - thus, Llagas is proscribed from joining a labor union, more so being elected as union officer. In the case of Javier, a supervisory EE, she may join a labor union composed only of supervisory employees. Finding both union officers to be employees not belonging to the rank-and-file, their membership in the Union has become questionable, rendering the Union inutile to represent their cause.Disposition Petition is granted. NLRC resolution reinstated.

PROHIBITION AND RATIONALEMETROLAB INDUSTRIES, INC VS ROLDAN-CONFESSOR

254 SCRA 182KAPUNAN; February 28, 1996

NATUREPetition for certiorari seeking the annulment of the Resolution and Omnibus Resolution of the Secretary of Labor and Employment

FACTS- On 31 December 1990, the Collective Bargaining Agreement (CBA) between Metrolab and the Private respondent Metro Drug Corporation Employees Association-Federation of Free Workers (hereinafter referred to as the Union) expired. The negotiations for a new CBA, however, ended in a deadlock. The Union filed a notice of strike against Metrolab and Metro Drug Inc. The parties failed to settle their dispute despite the conciliation efforts of the National Conciliation and Mediation Board.-To contain the escalating dispute, the then Secretary of Labor and Employment, Ruben D. Torres, issued an assumption order, the dispositive portion of which states, among others: Accordingly, any strike or lockout is hereby strictly enjoined. The Companies and the Metro Drug Corp. Employees Association ? FFW are likewise directed to cease and desist from committing any and all acts that might exacerbate the situation . - Thereafter, the union filed a motion for reconsideration. during the pendency of the abovementioned motion for reconsideration, Metrolab laid off 94 of its rank and file employees. On the same date, the Union filed a motion for a cease and desist order to enjoin Metrolab from implementing the mass layoff, alleging that such act violated the prohibition against committing acts that would exacerbate the dispute as specifically directed in the assumption order. 2 - On the other hand, Metrolab contended that the layoff was temporary and in the exercise of its management prerogative. It maintained that the company would suffer a yearly gross revenue loss of approximately sixty-six (66) million pesos due to the withdrawal of its principals in the Toll and Contract Manufacturing Department.- Thereafter, on various dates, Metrolab recalled some of the laid off workers on a temporary basis due to availability of work in the production lines.On 14 April 1992, Acting Labor Secretary Nieves Confesor issued a resolution declaring the layoff of Metrolab's 94 rank and file workers illegal and ordered their reinstatement with full backwages.- Labor Secretary Confesor issued the assailed Pending the resolution of the aforestated motions. Metrolab laid off 73 of its employees on grounds of redundancy due to lack of work which the union again promptly opposed on. Labor Secretary Confesor again issued a cease and desist order. Metrolab moved for a reconsiderations. The Omnibus Resolution contains the following orders: 1. MII's motion for partial reconsideration of our 14 April 1992 resolution specifically that portion thereof assailing our ruling that the layoff of the 94 employees is illegal, is hereby denied. MII is hereby ordered to pay such employees their full backwages computed from the time of actual layoff to the time of actual recall; Labor Secretary Confesor also ruled that executive secretaries are excluded from the closed-shop provision of the CBA, not from the bargaining unit.

ISSUE/S1. WON the Sec of Labor erred in declaring the temporary layoff illegal, and ordering the reinstatement and payment of backwages to the affected employees2. WON the Sec of Labor erred in excluding executive secretaries as part of the bargaining unit of rank and file employees

HELD1. NORatio This Court recognizes the exercise of management prerogatives and often declines to interfere with the legitimate business decisions of the employer. However, this privilege is not absolute but subject to limitations imposed by law.

Reasoning -That Metrolab's business is of national interest is not disputed. Metrolab is one of the leading manufacturers and suppliers of medical and pharmaceutical products to the country.Metrolab's management prerogatives, therefore, are not being unjustly curtailed but duly balanced with and tempered by the limitations set by law, taking into account its special character and the particular circumstances in the case at bench. - As aptly declared by public respondent Secretary of Labor in its assailed resolution: one of the substantive evils which Article 263 (g) of the Labor Code seeks to curb is the exacerbation of a labor dispute to the further detriment of the national interest. When a labor dispute has in fact occurred and a general injunction has been issued restraining the commission of disruptive acts, management prerogatives must always be exercise consistently with the statutory objective.2. NORatio Although Article 245 of the Labor Code 20 limits the ineligibility to join, form and assist any labor organization to managerial employees, jurisprudence has extended this prohibition to confidential employees or those who by reason of their positions or nature of work are required to assist or act in a fiduciary manner to managerial employees and hence, are likewise privy to sensitive and highly confidential records

Reasoning In the case at bench, the Union does not disagree with petitioner that the executive secretaries are confidential employees. (I)n the collective bargaining process, managerial employees are supposed to be on the side of the employer, to act as its representatives, and to see to it that its interest are well protected. The employer is not assured of such protection if these employees themselves are union members. Collective bargaining in such a situation can become one-sided. It is the same reason that impelled this Court to consider the position of confidential employees as included in the disqualification found in Art. 245 as if the disqualification of confidential employees were written in the provision. If confidential employees could unionize in order to bargain for advantages for themselves, then they could be governed by their own motives rather than the interest of the employers. Moreover, unionization of confidential employees for the purpose of collective bargaining would mean the extension of the law to persons or individuals who are supposed to act "in the interest of the employers. It is not farfetched that in the course of collective bargaining, they might jeopardize that interest which they are duty-bound to protect.

Disposition Petition is partly granted. The resolutions of the Sec of Labor are hereby MODIFIED to the extent that executive secretaries of petitioner Metrolab's General Manager and the executive secretaries of the members of its Management Committee are excluded from the bargaining unit of petitioner's rank and file employees.

PEPSI COLA PRODUCTS VS SEC. OF LABOR (PEPSI-COLA SUPERVISORY EMPLOYEES ORGANIZATION-UOEF)

312 SCRA 104PURISIMA; August 10, 1999

NATURE Petitions for certiorari

FACTS- the Pepsi-Cola Employees Organization-UOEF (Union) filed a petition for certification election with the Med-Arbiter seeking to be the exclusive bargaining agent of supervisors of Pepsi-Cola Philippines, Inc. (PEPSI).-the Med-Arbiter granted the Petition, with the explicit statement that it was an affiliate of Union de Obreros Estivadores de Filipinas (federation) together with two (2) rank and file unions. Pepsi-Cola Labor Unity (PCLU) and Pepsi-Cola Employees Union of the Philippines (PEUP).- PEPSI filed with the Bureau of Labor Relations a petition to Set Aside, Cancel and/or Revoke Charter Affiliation of the Union on the grounds that (a) the members of the Union were managers and (b) a supervisors' union can not affiliate with a federation whose members include the rank and file union of the same company. - the Federation argued that:

Page 24: 33994637 Labor Rounds 1and2

Labor Law 2 A2010 - 24 - DisiniThe pertinent portion of Art.245 of the Labor Code states that. "Supervisory employees shall not be eligible for membership in a labor organization of the rank and file employees but may join, assist or form separate labor organization of their own.This provision of law does not prohibit a local union composed of supervisory employees from being affiliated to a federation which has local unions with rank-and-file members as affiliates.- the Secretary of Labor and Employment, sent in a Comment, alleging inter alia, that:…Art. 245 of the New Labor Code does not preclude the supervisor's union and the rank-and-file union from being affiliated with the same federation.A federation of local union is not the labor organization referred to in Art.245 but only becomes entitled to all the rights enjoyed by the labor organization (at the company level) when it has complied with the registration requirements found in Art.234 and 237. Hence, what is prohibited by Art.245 is membership of supervisory employees in a labor union (at the company level) of the rank and file. . . .. . . In other words, the affiliation of the supervisory employee's union with the same federation with which the rank and file employees union is affiliated did not make the supervisory employees members of the rank and file employee's union and vice versa.2 . . .- PEPSI, in its Reply, asserted:It is our humble contention that a final determination of the Petition to Set-Aside, Cancel, Revoke Charter Union Affiliation should first be disposed of before granting the Petition for the Conduct of Certification Election. - Petitioner again filed a Supplemental Reply stressing:The respondent supervisory union could do indirectly what it could not do directly as the simple expedient of affiliating with UOEF would negate the manifest intent and letter of the law that supervisory employees can only "join, assist or form separate labor organizations of their own" and cannot "be eligible for membership in a labor organization of the rank and file employees."

ISSUES1. WON a supervisors' union can affiliate with the same Federation of which 2 rank and file unions are likewise members, without violating Art. 245 of the Labor Code3 2. WON the Petition to cancel/revoke registration is a prejudicial question to the petition for certification election.3. WON confidential employees can join the labor union of the rank and file.

HELD1. NORatio If the intent of the law is to avoid a situation where supervisors would merge with the rank-and-file or where the supervisors' labor organization would represent conflicting interests, then a local supervisors' union should not be allowed to affiliate with the national federation of union of rank-and-file employees where that federation actively participates in union activity in the company.Reasoning The Court emphasizes that the limitation is not confined to a case of supervisors' wanting to join a rank-and-file union. The prohibition extends to a supervisors' local union applying for membership in a national federation the members of which include local unions of rank and file employees. The intent of the law is clear especially where, as in this case at bar, the supervisors will be co-mingling with those employees whom they directly supervise in their own bargaining unit.2. NORatio The Court applies the established rule correctly followed by the public respondent that an order to hold a certification election is proper despite the pendency of the petition for cancellation of the registration certificate of the respondent union. The rationale for this is that at the time the respondent union filed its petition, it still had the legal personality to perform such act absent an order directing the cancellation.3. NORatio A confidential employee is one entrusted with confidence on delicate matters, or with the custody, handling, or care and protection of the employer's property. While Art. 245 of the Labor Code singles out managerial employee as ineligible to join, assist or form any labor organization, under the doctrine of necessary implication, confidential employees are similarly disqualified. Reasoning- if these managerial employees would belong to or be affiliated with a Union, the latter might not be assured of their loyalty to the Union in view of evident conflict of interests. The Union can also become company dominated with the presence of managerial employees in Union membership." - in the collective bargaining process, managerial employees are supposed to be on the side of the employer, to act as its representatives, and to see to it that its interest are well protected. The employer is not assured of such protection if these employees themselves are union members. Collective bargaining in such a situation can become one-sided. It is the same reason that impelled this Court to consider the position of confidential employees as included in the disqualification found in Art. 245 as if the disqualification of confidential employees were written in the provision. If confidential employees could

3 Art. 245. Ineligibility of managerial employees to join any labor organization; right of supervisory employees. Managerial employees are not eligible to join, assist or form any labor organization. Supervisory employees shall not be eligible for membership in a labor organization of the rank-and-file employees but may join, assist or form separate labor organizations of their own.

unionize in order to bargain for advantages for themselves, then they could be governed by their own motives rather than the interest of the employers. Moreover, unionization of confidential employees for the purpose of collective bargaining would mean the extension of the law to persons or individuals who are supposed to act "in the interest of" the employers. It is not farfetched that in the course of collective bargaining, they might jeopardize that interest which they are duty bound to protect.

DISPOSITIONPetitions DISMISSED but Decision of the Secretary of Labor is MODIFIED in that Credit and Collection Managers and Accounting Managers are highly confidential employees not eligible for membership in a supervisors' union.

2. WORKER/MEMBER OF COOPERATIVE

BENGUET ELECTRIC COOPERATIVE v CALLEJA180 SCRA 740

Cortes; Dec.29, 1989

NATUREPetition for certiorari to review the resolution of the Director of the Bureau of Labor Relations

FACTS- A certification election was held to determine which of 2 BENECO (Benguet Electric Cooperative) labor unions would be the sole and exclusive bargaining representative of all rank and file employees of BENECO. The 2 disputing labor unions were: 1. BWLU-ADLO (Benguet electric cooperative Workers’ Labor Union-Association of Democratic Labor Organizations) and 2. BELU (BENECO Employees Labor Union)- After BELU won, BENECO filed a protest claiming that some employees who are members and co-owners of the cooperative had voted in the election when they are ineligible to do so. BENECO claimed that these employees are not eligible to form, join or assist labor orgs. of their own choosing because they are members and joint owners of the cooperative. BENECO’s protest was dismissed by the med-arbiter. - Bureau of Labor Relations (BLR) director Calleja affirmed the med-arbiter’s order and certified BELU as the sole and exclusive bargaining agent of all the rank and file employees of BENECO.

ISSUE1. WON employees of a cooperative are qualified to form or join a labor organization for purposes of collective bargaining

HELD1. NORatio The right to collective bargaining is not available to an employee of a cooperative who at the same time is a member and co-owner thereof. But employees who are neither members nor co-owners of the cooperative are entitled to exercise the rights to self-organization, collective bargaining and negotiation as mandated by the 1987 Constitution and applicable statutes. Reasoning As members of the cooperative they are co-owners thereof. As such, they cannot invoke the right to collective bargaining for “certainly an owner cannot bargain with himself or his co-owners.” (Cooperative Rural Bank of Davao City, Inc. v. Ferrer-Calleja, et al.)- The fact that the members-employees of petitioner do not participate in the actual management of the cooperative does not make them eligible to form, assist or join a labor organization for the purpose of collective bargaining with petitioner. It is the fact of ownership of the cooperative, and not involvement in the management thereof, which disqualifies a member from joining any labor organization within the cooperative. Thus, irrespective of the degree of their participation in the actual management of the cooperative, all members thereof cannot form, assist or join a labor organization for the purpose of collective bargaining.Disposition Petition is granted. Resolution of respondent director of BLR is annulled. Certification election is set aside.

CENTRAL NEGROS ELECTRIC COOPERATIVE V DOLE SECRETARY

201 SCRA 584REGALADO; September 13, 1991

NATURESpecial civil action for certiorari

FACTS

Page 25: 33994637 Labor Rounds 1and2

Labor Law 2 A2010 - 25 - Disini- Petitioner Central Negros Electric Cooperative, Inc. (CENECO) seeks to annul the order issued by then Acting Secretary of Labor Laguesma declaring the projected certification election unnecessary and directing petitioner CENECO to continue recognizing private respondent CENECO Union of Rational Employees (CURE) as the sole and exclusive bargaining representative of all the rank-and-file employees of petitioner's electric cooperative.- Previous events: Their CBA was valid for a term of 3 years; CURE then wrote CENECO proposing that negotiations be conducted for a new agreement. CENECO denied CURE's request on the ground that, under applicable decisions of the Supreme Court, employees who at the same time are members of an electric cooperative are not entitled to form or join a union.- CURE filed a petition for direct recognition or for certification election; CENECO filed a motion to dismiss on the ground of legal restraints. - Some employees of CENECO then filed for withdrawal of membership in the cooperative but CENECO contended that this cannot be allowed.

ISSUE/S1. WON the employees were allowed to withdraw membership from the cooperative so as to entitle them to form or join CURE for purposes of the negotiations for a collective bargaining agreement

HELD1. YESRatio Membership in the cooperative is on a voluntary basis. Hence, withdrawal therefrom cannot be restricted unnecessarily. The right to join an organization necessarily includes the equivalent right not to join the same.

No Right of Self-Organization: WORKER/MEMBER OF COOPERATIVE- It was held in Batangas I Electric Cooperative Labor Union vs. Romeo A. Young that "employees who at the same time are members of an electric cooperative are not entitled to form or join unions for purposes of collective bargaining agreement, for certainly an owner cannot bargain with himself or his co-owners."- However, nowhere in said case is it stated that member-employees are prohibited from withdrawing their membership in the cooperative in order to join a labor union.

Disposition The questioned order is hereby ANNULLED and SET ASIDE. The med-arbiter is hereby ordered to conduct a certification election among the rank-and- file employees of CENECO with CURE and No Union as the choices therein.

3. NON-EMPLOYEES

REPUBLIC PLANTERS BANK GENERAL SERVICES EMPLOYEES UNION - NATU v LAGUESMA

264 SCRA 637PUNO; Nov 21, 1996

NATUREPetition for certiorari

FACTS-on January 21, 1991, Republic Planters Bank General Services Employees Union-National Association of Trade Unions (petitioner) filed a petition for certification election to determine the sole and exclusive bargaining representative of all regular employees outside the bargaining unit of Republic Planters Bank. -The proposed bargaining unit is composed of clerks, messengers, janitors, plumbers, telex operators, mailing and printing personnel, drivers, mechanics and computer personnel. Allegedly, these employees are regular employees but are considered as contractual employees by the bank and are excluded from the existing collective bargaining agreement.-The bank filed moved to dismiss the petition for certification election on the contentions that they are employed on contractual basis, that there was already an existing bargaining unit, and that the petition failed to state the number of employees in the proposed bargaining unit.- The petition was dismissed by the Med-Arbiter. On Dec 21, 1992, Undersecretary Bienvenido Laguesma reversed the Order of the Med-arbiter. Petitioner filed a Motion for Reconsideration. Usec Laguesma modified the December 21, 1992 Resolution, finding 6 employees as regular and included in the existing rank and file unit. - Both parties moved for reconsideration. Petitioner sought a ruling that the other workers in the proposed bargaining unit should also be considered regular employees. On Feb 24, 1995, Usec Laguesma issued another Order reinstating the Resolution dated December 21, 1992. ISSUEWON petitioners have the right to self-organization, and thus be allowed to file a petition for certification election.

HELD NO.- In the case of Singer Sewing Machine Company vs. Drilon, et al., it was rules that if the union members are not employees, no right to organize for purposes of bargaining, nor to be certified as bargaining agent can be recognized. Since the persons involved are not employees of the company, they are not entitled to the constitutional right to join or form a labor organization for purposes of collective bargaining.

Disposition Petition is dismissed

2.05 PARTY PROTECTEDMACTAN WORKERS UNION vs. DON RAMON ABOITIZ,

President, Cebu Shipyard & Engineering Works, Inc.45 SCRA 577

FERNANDO; June 30, 1972

NATUREAppeal from CFI Cebu by intervenor Associated Labor Union

FACTS- The employees and laborers of Cebu Shipyard and Eng’g Works belong to two rival unions: the Mactan Workers Union and Associated Labor Union.- Associated Labor Union (ALU), as exclusive bargaining representative of the workers, entered into a CBA with Cebu Shipyard where they agreed on a profit-sharing bonus scheme. It was agreed that Cebu Shipyard will release the money to ALU then ALU will deliver it to the members. Any unclaimed money will be returned to the company after 60 days.-The 72 members of Mactan Workers Union failed to get their money (bec they did not like to go to ALU’s office). Their money was returned to the company and ALU advised the company not to deliver the amount to the members of the Mactan Workers Union unless ordered by the Court, otherwise the ALU will take steps to protect the interest of its members. -So Cebu Shipyard did not pay to the plaintiffs, but instead, deposited the said amount with the Labor Administrator. For the recovery of their money, Mactan Workers filed money claims against Cebu Shipyard.-The lower court declared the Cebu Shipyard to deliver the money to ALU, and for ALU to distribute it immediately to Mactan Workers members.- ALU appealed, alleging lack of a cause of action, of jurisdiction of the City Court of Lapulapu and of personality of the Mactan Workers Union to represent its members.

ISSUESWON CFI Cebu was correct in ordering ALU to deliver Mactan Workers Union’s share

HELD YES-The lower court just required literal compliance with the terms of a collective bargaining contract-The terms and conditions of a collective bargaining contract constitute the law between the parties. Those who are entitled to its benefits can invoke its provisions. In the event that an obligation therein imposed is not fulfilled, the aggrieved party has the right to go to court for redress. - The benefits of a collective bargaining agreement extend to the laborers and employees in the collective bargaining unit, including those who do not belong to the chosen bargaining labor organization. … The labor union that gets the majority vote as the exclusive bargaining representative does not act for its members alone. It represents all the employees in such a bargaining unit.

- The raison d’etre of labor unions: it is not to be forgotten that what is entitled to constitutional protection is labor, or more specifically the working men and women, not labor organizations. The latter are merely the instrumentalities through which their welfare may be promoted and fostered. That is the raison d'etre of labor unions.

Disposition CFI Decision affirmed

2.06 NON-ABRIDGMENT OF RIGHTART. 246. Non-abridgment of right to self-organization. - It shall be unlawful for any person to restrain, coerce, discriminate against or unduly interfere with employees

and workers in their exercise of the right to self-organization. Such right shall include the right to form, join, or assist labor organizations for the purpose of collective bargaining through representatives of their own choosing and to engage in lawful concerted activities for the same purpose for their mutual aid and protection, subject to the provisions of Article 264 of this Code. (As amended by Batas Pambansa Bilang 70, May 1, 1980).

Page 26: 33994637 Labor Rounds 1and2

Labor Law 2 A2010 - 26 - DisiniART. 248. Unfair labor practices of employers. - It shall be unlawful for an employer to commit any of the following unfair labor practice: (a) To interfere with, restrain or coerce employees in the exercise of their right to self-organization; (b) To require as a condition of employment that a person or an employee shall not join a labor organization or shall withdraw from one to which he belongs; (c) To contract out services or functions being performed by union members when such will interfere with, restrain or coerce employees in the exercise of their rights to self-organization; (d) To initiate, dominate, assist or otherwise interfere with the formation or administration of any labor organization, including the giving of financial or other support to it or its organizers or supporters; (e) To discriminate in regard to wages, hours of work and other terms and conditions of employment in order to encourage or discourage membership in any labor organization. Nothing in this Code or in any other law shall stop the parties from requiring membership in a recognized collective bargaining agent as a condition for employment, except those employees who are already members of another union at the time of the signing of the collective bargaining agreement. Employees of an appropriate bargaining unit who are not members of the recognized collective bargaining agent may be assessed a reasonable fee equivalent to the dues and other fees paid by members of the recognized collective bargaining agent, if such non-union members accept the benefits under the collective bargaining agreement: Provided, that the individual authorization required under Article 242, paragraph (o) of this Code shall not apply to the non-members of the recognized collective bargaining agent; (f) To dismiss, discharge or otherwise prejudice or discriminate against an employee for having given or being about to give testimony under this Code; (g) To violate the duty to bargain collectively as prescribed by this Code; (h) To pay negotiation or attorney’s fees to the union or its officers or agents as part of the settlement of any issue in collective bargaining or any other dispute; or (i) To violate a collective bargaining agreement.

The provisions of the preceding paragraph notwithstanding, only the officers and agents of corporations, associations or partnerships who have actually participated in, authorized or ratified unfair labor practices shall be held criminally liable. (As amended by Batas Pambansa Bilang 130, August 21, 1981).

ART. 249. Unfair labor practices of labor organizations. - It shall be unfair labor practice for a labor organization, its officers, agents or representatives: (a) To restrain or coerce employees in the exercise of their right to self-organization. However, a labor organization shall have the right to prescribe its own rules with respect to the acquisition or retention of membership; (b) To cause or attempt to cause an employer to discriminate against an employee, including discrimination against an employee with respect to whom membership in such organization has been denied or to terminate an employee on any ground other than the usual terms and conditions under which membership or continuation of membership is made available to other members; (c) To violate the duty, or refuse to bargain collectively with the employer, provided it is the representative of the employees; (d) To cause or attempt to cause an employer to pay or deliver or agree to pay or deliver any money or other things of value, in the nature of an exaction, for services which are not performed or not to be performed, including the demand for fee for union negotiations; (e) To ask for or accept negotiation or attorney’s fees from employers as part of the settlement of any issue in collective bargaining or any other dispute; or (f) To violate a collective bargaining agreement.

The provisions of the preceding paragraph notwithstanding, only the officers, members of governing boards, representatives or agents or members of labor associations or organizations who have actually participated in, authorized or ratified unfair labor practices shall be held criminally liable. (As amended by Batas Pambansa Bilang 130, August 21, 1981).

III. LABOR ORGANIZATIONPOLICY

ART. 211. Declaration of Policy. - A. It is the policy of the State:(b) To promote free trade unionism as an instrument for the enhancement of democracy and the promotion of social justice and development; (c) To foster the free and voluntary organization of a strong and united labor movement;

LABOR ORGANIZATION –UNIONS

DEFINITIONAIRLINE PILOTS ASSOCIATION OF THE PHILIPPINES V. CIR

76 SCRA 274

CASTRO; April 15, 1977

Definition of Legitimate Labor Organization:Section 2(e) of R.A. 875 defines "labor organization" as any union or association of employees which exist, in whole or in part, for the purpose of the collective bargaining or dealing with employers concerning terms and conditions of employment." The emphasis of Industrial Peace Act is clearly on the purposes for which a union or association of employees established rather than that membership therein should be limited only to the employees of a particular employer. Under Section 2(h) of R.A 875 "representative" is define as including "a legitimate labor organization or any officer or agent of such organization, whether or not employed by the employer or employee whom he represents." It cannot be overemphasized likewise that labor dispute can exist "regardless of whether the disputants stand in the proximate relation of employer and employee.”

DUNLOP SLAZENGER V SEC. OF LABOR (RUIZ)300 SCRA 120

PUNO; DECEMBER 11, 1998

NATUREPetition for certiorari

FACTS- Respondent union filed a Petition for Certification Election among the supervisory, office and technical employees of the petitioner company before the DOLE, Regional Office No. III.- Petitioner company filed a motion to dismiss based on 1) that the respondent union is comprised of supervisory and rank-and-file employees and cannot act as bargaining agent for the proposed unit; (2) that a single certification election cannot be conducted jointly among supervisory and rank-and-file employees; and (3) that the respondent union lacks legal standing since it failed to submit its books of accounts.- Respondent alleges that it is composed only of supervisory employees and that it has no obligation to attach its books of accounts since it is a legitimate labor organization.- The mediator arbiter granted the petition of the union. It said that the contention of the respondent that the petitioning union is composed of both supervisory and rank and file employees is not sufficient to dismiss the petition. It can be remedied thru the exclusion-inclusion proceedings wherein those employees who are occupying rank and file positions will be excluded from the list of eligible voters. The secretary of labor affirmed.

ISSUE/SWON the union can be composed of supervisory and rank and file employees

HELDNO. Ratio Article 245 of the Labor Code clearly provides that "supervisory employees shall not be eligible for membership in a labor organization of the rank-and-file employees.”Reasoning Public respondent gravely misappreciates the basic antipathy between the interest of supervisors and the interest of rank-and-file employees. There is a irreconcilability of their interests which cannot be cured even in the exclusion-inclusion proceedings.Disposition Petition is granted.

DOLE REGISTRATION AS BASIS LEGITIMACY

TAGAYTAY HIGHLANDS INTERNATIONAL GOLF CLUB INC V TAGAYTAY HIGHLANDS EMPLOYEES UNION-PGTWO

395 SCRA 699CARPIO-MORALES; January 22, 2003

NATUREPetition for Certiorari under Rule 45 THIGCI assailing CA decision denying its petition to annul the Department of Labor and Employment (DOLE) Resolutions of November 12, 1998 and December 29, 1998

FACTS- October 16, 1997 > Tagaytay Highlands Employees Union (THEU), Philippine Transport and General Workers Organization (PTGWO), Local Chapter No. 776, a legitimate labor organization said to represent majority of the rank-and-file employees of THIGCI, filed a petition for certification election before the DOLE Mediation-Arbitration Unit, Regional Branch No. IV

Page 27: 33994637 Labor Rounds 1and2

Labor Law 2 A2010 - 27 - Disini- November 27, 1997 > opposed petition for certification election because the list of union members submitted by it was defective and fatally flawed as it included the names and signatures of supervisors, resigned, terminated and absent without leave (AWOL) employees, as well as employees of The Country Club, Inc., a corporation distinct and separate from THIGCI; and that out of the 192 signatories to the petition, only 71 were actual rank-and-file employees of THIGCI. Also, some of the signatures in the list of union members were secured through fraudulent and deceitful means, and submitted copies of the handwritten denial and withdrawal of some of its employees from participating in the petition.- THEU asserted that it complied with all the requirements for valid affiliation and inclusion in the roster of legitimate labor organizations pursuant to DOLE Department Order No. 9, series of 1997, on account of which it was duly granted a Certification of Affiliation by DOLE on October 10, 1997; and that Section 5, Rule V of said Department Order provides that the legitimacy of its registration cannot be subject to collateral attack, and for as long as there is no final order of cancellation, it continues to enjoy the rights accorded to a legitimate organization. Therefore, the Med-Arbiter should, pursuant to Article 257 of the Labor Code and Section 11, Rule XI of DOLE Department Order No. 09, automatically order the conduct of a certification election. - January 28, 1998 > DOLE Med-Arbiter Anastacio Bactin ordered the holding of a certification election - DOLE Resolution of November 12, 19981 > setting aside the June 4, 1998 Resolution dismissing the petition for certification election. MFR denied - CA - denied THIGCI’s Petition for Certiorari and affirmed the DOLE Resolution dated November 12, 1998. It held that while a petition for certification election is an exception to the innocent bystander rule, hence, the employer may pray for the dismissal of such petition on the basis of lack of mutuality of interests of the members of the union as well as lack of employer-employee relationship and petitioner failed to adduce substantial evidence to support its allegations.

ISSUE. WON DOLE registration can be the basis for legitimacyHELD NO- Continuing, petitioner argues that without resolving the status of THEU, the DOLE Undersecretary "conveniently deferred the resolution on the serious infirmity in the membership of [THEU] and ordered the holding of the certification election" which is frowned upon as the following ruling of this Court shows:We also do not agree with the ruling of the respondent Secretary of Labor that the infirmity in the membership of the respondent union can be remedied in "the pre-election conference thru the exclusion-inclusion proceedings wherein those employees who are occupying rank-and-file positions will be excluded from the list of eligible voters." Public respondent gravely misappreciated the basic antipathy between the interest of supervisors and the interest of rank-and-file employees. Due to the irreconcilability of their interest we held in Toyota Motor Philippines v. Toyota Motors Philippines Corporation Labor Union, viz:"Clearly, based on this provision [Article 245], a labor organization composed of both rank-and-file and supervisory employees is no labor organization at all. It cannot, for any guise or purpose, be a legitimate labor organization. Not being one, an organization which carries a mixture of rank-and-file and supervisory employees cannot posses any of the rights of a legitimate labor organization, including the right to file a petition for certification election for the purpose of collective bargaining. It becomes necessary, therefore, anterior to the granting of an order allowing a certification election, to inquire into the composition of any labor organization whenever the status of the labor organization is challenged on the basis of Article 245 of the Labor Code." (Emphasis by petitioner) (Dunlop Slazenger (Phils.), v. Secretary of Labor, 300 SCRA 120 [1998]; Underscoring and emphasis supplied by petitioner.)

LAGUNA AUTOPARTS V OFFICE OF THE DOLE SECRETARY 457 SCRA 730

CALLEJO; April 29, 2005 NATUREPetition to review the decision of the Court of Appeals FACTS

- May 3, 1999 - Laguna Autoparts Manufacturing Corporation Obrero Pilipino-LAMCOR Chapter filed a petition for certification election with the DOLE. In its petition, it noted its registration certificate number along with the registration certificate number of chapter affiliate. The petition alleged that the union was composed of all rank-and-file employees, that the bargaining unit is unorganized and that there had been no certification election for the past 12 months prior to the filing of the petition.- Laguna Autoparts moved to cancel the certification election because the union was not considered a legitimate labor organization for failure to show that it had complied with registration requirements such as submission of required documents to the Bureau of Labor Relations.- Med-Arbiter Bactin dismissed the certification election petition because of the union’s lack of personality. DOLE Secretary Sto. Tomas reversed Bactin’s order. This was affirmed by the CA.

- The CA stressed that a local or chapter need not be registered to become a legitimate labor organization. It pointed out that a local or chapter acquires legal personality as a labor organization from the date of filing of the complete documents. - The CA noted that it was the employer which offered the most tenacious resistance to the holding of a certification election among its regular rank-and-file employees. It opined that this must not be so for the choice of a collective bargaining agent was the sole concern of the employees, and the employer should be a mere bystander.

ISSUE/S

1.        WON the union is a legitimate labor organization2.        WON the chapter’s legal personality can be attacked collaterally in an election action

HELD1. YESRatio A local or chapter need not be independently registered to acquire legal personality. A local/chapter constituted in accordance with Section 1 of Rule 6 shall acquire legal personality from the date of filing of the complete documents enumerated. ReasoningThe task of determining whether the local or chapter has submitted the complete documentary requirements is lodged with the Regional Office or the BLR, as the case may be. The records of the case show that the respondent union submitted the said documents to the regional office and was issued a certification. 2. NORatioReasoning- It may not be subject to a collateral attack but only through a separate action instituted particularly for the purpose of assailing the chapter, as prescribed in Section 5, Rule 5 of the implementing rules of Book 5 which states: “The labor organization or workers’ association shall be deemed registered and vested with legal personality on the date of issuance of its certificate of registration. Such legal personality cannot thereafter be subject to collateral attack but may be questioned only in an independent petition for cancellation in accordance with these Rules.[”- The pronouncement of the Labor Relations Division Chief, that the respondent union acquired a legal personality with the submission of the complete documentary requirement, cannot be challenged in a petition for certification election.Disposition Petition is denied. No costs.

COASTAL SUBIC BAY TERMINAL, INC. V DOLE507 SCRA 300

QUISUMBING; November 20, 2006

NATURE For review on certiorari

FACTS- Respondents Coastal Subic Bay Terminal, Inc. Rank-and-File Union (CSBTI-RFU) and Coastal Subic Bay Terminal, Inc. Supervisory Union (CSBTI-SU) filed separate petitions for certification election - The rank-and-file union insists that it is a legitimate labor organization having been issued a charter certificate by the Associated Labor Union (ALU), and the supervisory union by the Associated Professional, Supervisory, Office and Technical Employees Union (APSOTEU).

Page 28: 33994637 Labor Rounds 1and2

Labor Law 2 A2010 - 28 - Disini- Petitioner Coastal Subic Bay Terminal, Inc. (CSBTI) opposed both petitions for certification election alleging that the rank-and-file union and supervisory union were not legitimate labor organizations- Petitioner argues that APSOTEU improperly secured its registration from the DOLE Regional Director and not from the BLR - Petitioner insists that APSOTEU lacks legal personality, and its chartered affiliate CSBTI-SU cannot attain the status of a legitimate labor organization to file a petition for certification election. - Without ruling on the legitimacy of the respondent unions, the Med-Arbiter dismissed, both petitions - Both parties appealed to the Secretary of Labor and Employment, who reversed the decision of the Med-Arbiter. The Secretary ruled that CSBTI-SU and CSBTI-RFU have separate legal personalities to file their separate petitions for certification election. The Secretary held that APSOTEU is a legitimate labor organization because it was properly registered pursuant to the 1989 Revised Rules and Regulations implementing Republic Act No. 6715, the rule applicable at the time of its registration. It further ruled that ALU and APSOTEU are separate and distinct labor unions having separate certificates of registration from the DOLE. They also have different sets of locals. - The motion for reconsideration was also denied.- On appeal, the Court of Appeals affirmed the decision of the Secretary. - The motion for reconsideration was likewise denied. Hence, the instant petition by the company anchored on the following grounds:

ISSUES1. WON the CA erred in relying on the “1989 Revised Rules and Regulations implementing RA 6715” as basis to recognize private respondent APSOTEU’s registration by the DOLE Regional Director.2. WON ALU, a rank-and-file union and APSOTEU, a supervisory union one and the same because of the commonalities between them. 3. WON the supervisory and rank-and-file unions could separately petition for certification elections.

HELD1. NO. Ratio Article 235 of LC provides that applications for registration shall be acted upon by the Bureau. “Bureau” as defined under the Labor Code means the BLR and/or the Labor Relations Division in the Regional Offices of the Department of Labor. - Section 2, Rule II, Book V of the 1989 Revised Implementing Rules of the Labor Code (Implementing Rules) provides that:Section 2. Where to file application; procedure - Any national labor organization or labor federation or local union may file an application for registration with the Bureau or the Regional Office where the applicant’s principal offices is located. The Bureau or the Regional Office shall immediately process and approve or deny the application. In case of approval, the Bureau or the Regional Office shall issue the registration certificate within thirty (30) calendar days from receipt of the application, together with all the requirements for registration as hereinafter provided. The Implementing Rules specifically Section 1, Rule III of Book V, as amended by Department Order No. 9, thus: SECTION 1. Where to file applications. - The application for registration of any federation, national or industry union or trade union center shall be filed with the Bureau. Where the application is filed with the Regional Office, the same shall be immediately forwarded to the Bureau within forty-eight (48) hours from filing thereof, together with all the documents supporting the registration.The applications for registration of an independent union shall be filed with and acted upon by the Regional Office where the applicant’s principal office is located ….- The DOLE issued Department Order No. 40-03, which took effect on March 15, 2003, further amending Book V of the above implementing rules. The new implementing rules explicitly provide that applications for registration of labor organizations shall be filed either with the Regional Office or with the BLR.- Even after the amendments, the rules did not divest the Regional Office and the BLR of their jurisdiction over applications for registration by labor organizations. The amendments to the implementing rules merely specified that when the application was filed with the Regional Office, the application would be acted upon by the BLR. Reasoning The records in this case showed that APSOTEU was

registered on March 1, 1991. Accordingly, the law applicable at that time was Section 2, Rule II, Book V of the Implementing Rules, and not Department Order No. 9 which took effect only on June 21, 1997. Thus, considering further that APSOTEU’s principal office is located in Diliman, Quezon City, and its registration was filed with the NCR Regional Office, the certificate of registration is valid.

2. NO.Ratio Once a labor union attains the status of a legitimate labor organization, it continues as such until its certificate of registration is cancelled or revoked in an independent action for cancellation. In addition, the legal personality of a labor organization cannot be collaterally attacked. Reasoning In the absence of any independent action for cancellation of registration against either APSOTEU or ALU, and unless and until their registrations are cancelled, each continues to possess a separate legal personality. The CSBTI-RFU and CSBTI-SU are therefore affiliated with distinct and separate federations, despite the commonalities of APSOTEU and ALU. - Under the rules implementing the Labor Code, a chartered local union acquires legal personality through the charter certificate issued by a duly registered federation or national union, and reported to the Regional Office in accordance with the rules implementing the Labor Code. A local union does not owe its existence to the federation with which it is affiliated. It is a separate and distinct voluntary association owing its creation to the will of its members. Mere affiliation does not divest the local union of its own personality, neither does it give the mother federation the license to act independently of the local union. It only gives rise to a contract of agency, where the former acts in representation of the latter. Hence, local unions are considered principals while the federation is deemed to be merely their agent. As such principals, the unions are entitled to exercise the rights and privileges of a legitimate labor organization, including the right to seek certification as the sole and exclusive bargaining agent in the appropriate employer unit.

3. NO. Ratio Under Article 245 of the Labor Code, supervisory employees are not eligible for membership in a labor union of rank-and-file employees. The supervisory employees are allowed to form their own union but they are not allowed to join the rank-and-file union because of potential conflicts of interest. Further, to avoid a situation where supervisors would merge with the rank-and-file or where the supervisors’ labor union would represent conflicting interests, a local supervisors’ union should not be allowed to affiliate with the national federation of unions of rank-and-file employees where that federation actively participates in the union activity within the company. Thus, the limitation is not confined to a case of supervisors wanting to join a rank-and-file union. The prohibition extends to a supervisors’ local union applying for membership in a national federation the members of which include local unions of rank-and-file employees. Reasoning In the instant case, the national federations that exist as separate entities to which the rank-and-file and supervisory unions are separately affiliated with, do have a common set of officers. In addition, APSOTEU, the supervisory federation, actively participates in the CSBTI-SU while ALU, the rank-and-file federation, actively participates in the CSBTI-RFU, giving occasion to possible conflicts of interest among the common officers of the federation of rank-and-file and the federation of supervisory unions. For as long as they are affiliated with the APSOTEU and ALU, the supervisory and rank-and-file unions both do not meet the criteria to attain the status of legitimate labor organizations, and thus could not separately petition for certification elections. The purpose of affiliation of the local unions into a common enterprise is to increase the collective bargaining power in respect of the terms and conditions of labor. When there is commingling of officers of a rank-and-file union with a supervisory union, the constitutional policy on labor is circumvented. Labor organizations should ensure the freedom of employees to organize themselves for the purpose of leveling the bargaining process but also to ensure the freedom of workingmen and to keep open the corridor of opportunity to enable them to do it for themselves.

Page 29: 33994637 Labor Rounds 1and2

Labor Law 2 A2010 - 29 - DisiniDisposition The petition was GRANTED.

CEBU SEAMEN'S ASSO., INC., V FERRER-CALLEJA212 SCRA 50

MEDIALDEA; August 4, 1992

NATUREPetition seeking the reversal of the resolution of the Bureau of Labor Relations which affirmed the decision of the Med-Arbiter holding that the set of officers of Seamen's Association of the Philippines headed by Dominica C. Nacua, as president, was the lawful set of officers entitled to the release and custody of the union dues as well as agency fees of said association.

FACTS-23Oct 1950: a group of deck officers and marine engineers on board vessels plying Cebu and other ports of the Philippines organized themselves into an association and registered the same as a non-stock corporation known as Cebu Seamen's Association, Inc. (CSAI), with the Securities and Exchange Commission (SEC). Later, on 23 June 1969, the same group registered its association with this Bureau as a labor union known as the Seamen's Association of the Philippines, Incorporated (SAPI).-SAPI has an existing CBA with the Aboitiz Shipping Corporation due to expire on 31 Dec 1988. In consonance with the CBA, said company has been remitting checked-off union dues to said union until February, 1987 when Banayoyo, et al introducing themselves to be the new set of officers, went to the company and claimed that they are entitled to the remittance and custody of such union dues. -26 May 1987: another group headed by Dominica C. Nacua, claiming as the duly elected set of union officers, filed a complaint, for and in behalf of the union, against the CSAI as represented by Manuel Gabayoyo for the security of the aforementioned CBA, seeking such relief, among others, as an order restraining the respondent from acting on behalf of the union and directing the Aboitiz Shipping Corp. to remit the checked-off union dues for the months of March and April 1987.-CSAI filed its Answer alleging that the complainant union and CSAI are one and the same union; that Dominica C. Nacua and Atty. Prospero Paradilla who represented the union had been expelled as members/officers as of November 1984 for lawful causes; and, that its set of officers headed by Manuel Gabayoyo has the lawful right to the remittance and custody of the corporate funds (otherwise known as union dues) in question pursuant to the resolution of the SEC dated 22 April 1987. The following day, CSAI filed MTD on the grounds, among others, that the SEC, not the Med-Arbiter, has jurisdiction over the dispute as provided under P.D. No. 902-A; that there can neither be a complainant nor respondent in the instant case as the parties involved are one and the same labor union, and that Mrs. Dominica C. Nacua and Atty. Prospero Paradilla have no personality to represent the union as they had already been expelled as members/officers thereof in two resolutions of the Board of Directors dated November 1984 and January 17, 1987.-19 June 1987: the Med-Arbiter issued an Order denying said motion but directing the Aboitiz Shipping Corporation to remit the already checked-off union dues to the complainant union through its officers and to continue remitting any checked-off union dues until further notice. The Med-Arbiter also set further hearing of the complaint on July 1, 1987.-The Med-Arbiter eventually held that SAPI, headed by Nacua, was the lawful set of officers entitled to the release and custody of the union dues as well as agency fees of said association. CSAI headed by Gabayoyo filed an appeal with the Bureau of Labor Relations (BLR). But the latter affirmed the Med-Arbiter’s decision. The Gabayoyo group also appealed to the Office of the SOLE, but this was also denied for lack of merit. Hence, this petition.

ISSUES1. WON the med-arbiter of Region VII has jurisdiction over the case2. WON SAPI was registered as a labor federation with the BLR

3. WON Nacua and Paradilla have the personality to represent the union

HELD1. YES- Article 226 of the Labor Code vests upon the Bureau of Labor Relations and Labor Relations Division the original and exclusive authority and jurisdiction to act on all inter-union and intra-union disputes. The controversy between the aforesaid two sets of officers (Cebu Seamen's Association headed by Gabayoyo & Seamen's Association of the Philippines headed by Nacua) is an intra-union dispute. Both sets of officers claim to be entitled to the release of the union dues collected by the company with whom it had an existing CBA.

2. YES.-As stated in the findings of fact in the questioned resolution of Director Pura Ferrer-Calleja, on October 23, 1950, a group of deck officers organized the Cebu Seamen's Association, Inc., (CSAI), a non-stock corporation and registered it with the Securities and Exchange Commission (SEC). The same group registered the organization with the Bureau of Labor Relations (BLR) as Seamen's Association of the Philippines (SAPI). It is the registration of the organization with the BLR are not with the SEC which made it a legitimate labor organization with rights and privileges granted under the Labor Code.

3. YES.-CSAI, the corporation was already inoperational before the controversy in this case arose. In fact, on August 24, 1984 the SEC ordered the CSAI to show cause why its certificate of registration should not be revoked for continuous inoperation. There is nothing in the records which would show that CSAI answered said show-cause order.-Also, before the controversy, Nacua was elected president of the labor union, SAPI. It had an existing CBA with Aboitiz Shipping Corporation. Before the end of the Nacua’s term, some union members including Gabayoyo showed signs of discontentment with the leadership of Nacua. This break-away group revived the moribund corporation and issued an undated resolution expelling Nacua from association. It later held its own election of officers supervised by the SEC and filed a case of estafa against Nacua. -The expulsion of Nacua from the corporation, of which she denied being a member, has however, not affected her membership with the labor union. In fact, in the elections of officers for 1987-1989, she was re-elected as the president of the labor union. -In this connection, We cannot agree with the contention of Gabayoyo that Nacua was already expelled from the union. Whatever acts their group had done in the corporation do not bind the labor union. Moreover, Gabayoyo cannot claim leadership of the labor group by virtue of his having been elected as a president of the dormant corporation CSAI.- Bureau of Labor Relations correctly ruled on the basis of the evidence presented by the parties that SAPI, the legitimate labor union, registered with its office, is not the same association as CSAI, the corporation, insofar as their rights under the Labor Code are concerned. Hence, the former and not the latter association is entitled to the release and custody of union fees with Aboitiz Shipping and other shipping companies with whom it had an existing CBA. -It is undisputed from the records that the election of the so-called set of officers headed by Gabayoyo was conducted under the supervision of the SEC, presumably in accordance with its constitution and by-laws as well as the articles of incorporation of respondent CSAI, and the Corporation Code. That had been so precisely on the honest belief of the participants therein that they were acting in their capacity as members of the said corporation. That being the case, the aforementioned set of officers is of the respondent corporation and not of the complainant union. It follows, then, that any proceedings, and actions taken by said set of officers can not, in any manner, affect the union and its members.-The other set of officers headed by Dominica C. Nacua is the lawful set of officers of SAPI and therefore, is entitled to the release and

Page 30: 33994637 Labor Rounds 1and2

Labor Law 2 A2010 - 30 - Disinicustody of the union dues as well as the agency fees, if any, there be. Disposition Petition dismissed.

PROGRESSIVE DEVELOPMENT CORP. V. LAGUESMA244 SCRA 173

GUTIERREZ; FEB. 4,1992

NATUREPetition for certiorari to set aside the resolution of Med-arbiter and two orders the of the Sec. of Labor.

FACTSRespondent Kilusan filed with the Department of Labor and Employment (DOLE) a petition for certification election among the rank-and-file employees of the petitioner alleging that it is a legitimate labor federation and its local chapter, Progressive Development Employees Union, was issued charter certificate No. 90-6-1-153. Kilusan claimed that there was no existing collective bargaining agreement and that no other legitimate labor organization existed in the bargaining unit.

Petitioner PDC filed its motion to dismiss contending that the local union failed to comply with Rule II, Section 3, Book V of the Rules Implementing the Labor Code, as amended, which requires the submission of: (a) the constitution and by-laws; (b) names, addresses and list of officers and/or members; and (c) books of accounts.

Kilusan claimed that it had submitted the necessary documentary requirements for registration, such as the constitution and by-laws of the local union, and the list of officers/members with their addresses. Kilusan further averred that no books of accounts could be submitted as the local union was only recently organized.

Petitioner insisted that upon verification with the Bureau of Labor Relations (BLR), it found that the alleged minutes of the organizational meeting was unauthenticated, the list of members did not bear the corresponding signatures of the purported members, and the constitution and by-laws did not bear the signatures of the members and was not duly subscribed. It argued that the private respondent therefore failed to substantially comply with the registration requirements provided by the rules.

Med-Arbiter held that there was substantial compliance with the requirements for the formation of a chapter. Undersecretary Laguesma denied PDC's motion for reconsideration. Hence, this petition.

ISSUEWhen does a branch, local or affiliate of a federation become a legitimate labor organization.

HELDOrdinarily, a labor organization acquires legitimacy only upon registration with the BLR. Under Article 234 (Requirements of Registration):

Any applicant labor organization, association or group of unions or workers shall acquire legal personality and shall be entitled to the rights and privileges granted by law to legitimate labor organizations upon issuance of the certificate of registration based on the following requirements: (a) Fifty-pesos (P50.00) registration fee; (b) The names of its officers, their addresses, the principal address of the labor organization, the minutes of the organizational meetings and the list of the workers who participated in such meetings;(c) The names of all its members comprising at least twenty 20% percent of all the employees in the bargaining unit where it seeks to operate;(d) If the applicant has been in existence for one or more years, copies of its annual financial reports; and(e) Four copies of the constitution and by-laws of the applicant union, the minutes of its adoption or ratification and the list of the members who

participated in it."

And under Article 235 (Action on Application):"The Bureau shall act on all applications for registration within thirty (30) days from filing.

All requisite documents and papers shall be certified under oath by the secretary or the treasurer of the organization, as the case may be, and attested to by its president."

Moreover, section 4 of Rule II, Book V of the Implementing Rules requires that the application should be signed by at least twenty percent (20%) of the employees in the appropriate bargaining unit and be accompanied by a sworn statement of the applicant union that there is no certified bargaining agent or, where there is an existing collective agreement duly submitted to the DOLE, that the application is filed during the last sixty (60) days of the agreement.

RATIONALE: The registration prescribed in is not a limitation to the right of assembly or association, which may be exercised with or without said registration. The latter is merely a condition sine qua non for the acquisition of legal personality by labor organizations, associations or unions and the possession of the "rights and privileges granted by law to legitimate labor organizations." The Constitution does not guarantee these rights and privileges, much less said personality, which are mere statutory creations, for the possession and exercise of which registration is required to protect both labor and the public against abuses, fraud, or impostors who pose as organizers, although not truly accredited agents of the union the purport to represent. Such requirement is a valid exercise of the police power, because the activities in which labor organizations, associations and unions of workers are engaged affect public interest, which should be protected. Furthermore, the obligation to submit financial statements, as a condition for the non-cancellation of a certificate of registration, is a reasonable regulation for the benefit of the members of the organization, considering that the same generally solicits funds or membership, as well as oftentimes collects, on behalf of its members, huge amounts of money due to them or to the organization.

But when an unregistered union becomes a branch, local or chapter of a federation, some of the aforementioned requirements for registration are no longer required. The provisions governing union affiliation are found in Rule II, Section 3, Book V of the Implementing Rules.

A local or chapter therefore becomes a legitimate labor organization only upon submission of the following to the BLR:

1) A charter certificate, within 30 days from its issuance by the labor federation or national union, and

2) The constitution and by-laws, a statement on the set of officers, and the books of accounts all of which are certified under oath by the secretary or treasurer, as the case may be, of such local or chapter, and attested to by its president.

RATIONALE: The intent of the law in imposing lesser requirements in the case of a branch or local of a registered federation or national union is to encourage the affiliation of a local union with a federation or national union in order to increase the local union's bargaining powers respecting terms and conditions of labor.

Absent compliance with these mandatory requirements, the local or chapter does not become a legitimate labor organization.

In the case at bar, the failure of the secretary of PDEU-Kilusan to certify the required documents under oath is fatal to its acquisition of a legitimate status.

DISPOSITION Petition is GRANTED. The assailed resolution and orders of respondents Med-Arbiter and Secretary of Labor and Employment,

Page 31: 33994637 Labor Rounds 1and2

Labor Law 2 A2010 - 31 - Disinirespectively, are hereby SET ASIDE.

UNION FUNCTION AND RATIONALE

UNITED SEAMEN’S UNION V DAVAO SHIPOWNERS ASSOCIATION

MAKALINTAL, J.; August 31, 1967

A labor organization is wholesome if it serves its legitimate purpose of promoting the interests of labor without unnecessary labor disputes. That is why it is given personality and recognition in concluding collective bargaining agreements. But if it is made use of as a subterfuge, or as a means to subvert valid commitments, it defeats its own purpose, for it tends to undermine the harmonious relations between management and labor.

GUIJARNO V CIR52 SCRA 307

FERNANDO; August 27, 1973

NATURESpecial civil action of certiorari

FACTS- Three unfair labor practice cases for unlawful dismissal allegedly based on legitimate union activity were filed against respondent Central Santos Lopez Co., Inc. and respondent United Sugar Workers Union-ILO- The respondent company, in its answer, alleged that the only reason for the dismissal of the complainants herein is because their said dismissal was asked by the USWU-ILO of which union respondent company has a valid and existing collective bargaining contract with a closed-shop provision to the effect that those laborers who are no longer members of good standing in the union may be dismissed by the respondent company if their dismissal is sought by the union; that respondent company has never committed acts of unfair labor practice against its employees or workers much less against the complainants herein but that it has a solemn obligation to comply with the terms and conditions of the contract.- Petitioners were employed by the company long before the collective bargaining contract.

ISSUE/S1. WON petitioners were validly dismissed

HELD1. NORatio A closed-shop provision in a collective bargaining agreement is not to be given a retroactive effect so as to preclude its being applied to employees already in the serviceReasoning In order that an employer may be deemed bound, under a collective bargaining agreement, to dismiss employees for non-union membership, the stipulation to this effect must be so clear and unequivocal as to leave no room for doubt thereon. An undertaking of this nature is so harsh that it must be strictly construed, and doubts must be resolved against the existence of "closed shop".- The function of a labor union is to assure that Constitution’s fundamental objectives for labor would be achieved. It is the instrumentality through which an individual laborer who is helpless as against a powerful employer may, through concerted effort and activity, achieve the goal of economic well-being Disposition Petition is granted.

RATIONALE: Where does that leave a labor union? Correctly understood, it is nothing but the means of assuring that such fundamental objectives would be achieved. It is the instrumentality through which an individual laborer who is helpless as against a powerful employer may, through concerted effort and activity achieve the goal of economic well-being. Workers unorganized are weak, workers organized are strong. Unions are merely instrumentalities through which their welfare may be protected and fostered. That is the raison d’être of labor unions. It is the instrumentality by which the weak laborer up

against the strong employer, may, by concerted effort, achieve economic well-being.

LABOR UNION AND GOVERNMENT REGULATION

1. Union Registration and Procedure

A. Requirements

SAN MIGUEL CORP. V MANDAUE467 SCRA 107

Tinga ; Aug. 16, 2005

Facts-CA affirmes DOLE Undersecretary for Labor Relations, Rosalinda Dimapilis-Baldoz, ordering the immediate conduct of a certification election among the petitioner’s rank-and-file employees. - Federation of Free Workers (FFW/ respondent) filed a petition for certification election with the DOLE Regional Office No. VII. It sought to be certified and to represent the permanent rank-and-file monthly paid employees of the petitioner. The following documents were attached to the petition: (1) a Charter Certificate certifying that respondent as of that date was duly certified as a local or chapter of FFW; (2) a copy of the constitution of respondent prepared by its Secretary, Noel T. Bathan and attested by its President, Wilfred V. Sagun; (3) a list of respondent’s officers and their respective addresses, again prepared by Bathan and attested by Sagun; (4) a certification signifying that respondent had just been organized and no amount had yet been collected from its members, signed by respondent’s treasurer Chita D. Rodriguez and attested by Sagun; and (5) a list of all the rank-and-file monthly paid employees of the Mandaue Packaging Products Plants and Mandaue Glass Plant prepared by Bathan and attested by Sagun. -SMC (Petitioner) filed a motion to dismiss the petition for certification election on the sole ground that herein respondent is not listed or included in the roster of legitimate labor organizations based on the certification issued by the Officer-In-Charge, Regional Director of the DOLE Regional Office No. VII, Atty. Jesus B. Gabor.-Respondent submitted to the Bureau of Labor Relations the same documents earlier attached to its petition for certification. The accompanying letter, signed by respondent’s president Sagun, stated that such documents were submitted in compliance with the requirements for the creation of a local/chapter pursuant to the Labor Code and its Implementing Rules; and it was hoped that the submissions would facilitate the listing of respondent under the roster of legitimate labor organizations.The Chief of Labor Relations Division of DOLE Regional Office No. VII issued a Certificate of Creation of Local/Chapter No. ITD. I-ARFBT-058/98, certifying that from 30 July 1998, respondent has acquired legal personality as a labor organization/worker’s association, it having submitted all the required documents.

Issues 1. When did the union acquire legal personality?2. Whether or not the inclusion of the two alleged supervisory employees in appellee union’s membership amounts to fraud, misrepresentation, or false statement within the meaning of Article 239(a) and (c) of the Labor Code.3. Whether or not subsequent developments change the disposition of the case

Held1. 15 June 1998, the date the complete documents were submitted.The issuance of the certificate of registration by the Bureau or Regional Office is not the operative act that vests legal personality upon a local/chapter under Department Order No. 9. Such legal personality is acquired from the filing of the complete documentary requirements enumerated in Section 1, Rule VI. Admittedly, the manner by which respondent was

Page 32: 33994637 Labor Rounds 1and2

Labor Law 2 A2010 - 32 - Disinideemed to have acquired legal personality by the DOLE and the Court of Appeals was not in strict conformity with the provisions of Department Order No. 9. Thus, in order to ascertain when respondent acquired legal personality, we only need to determine on what date the Regional Office or Bureau received the complete documentary requirements enumerated under Section 1, Rule VI of Department Order No. 9. There is no doubt that on 15 June 1998, or the date respondent filed its petition for certification election, attached thereto were respondent’s constitution, the names and addresses of its officers, and the charter certificate issued by the national union FFW. The first two of these documents were duly certified under oath by respondent’s secretary Bathan and attested to by president Sagun.What about the lack of documents containing the by-laws? Not needed so long as it is part of the union’s constitution.An examination of respondent’s constitution reveals it sufficiently comprehensive in establishing the necessary rules for its operation. Article IV establishes the requisites for membership in the local/chapter. Articles V and VI name the various officers and what their respective functions are. The procedure for election of these officers, including the necessary vote requirements, is provided for in Article IX, while Article XV delineates the procedure for the impeachment of these officers. Article VII establishes the standing committees of the local/chapter and how their members are appointed. Article VIII lays down the rules for meetings of the union, including the notice and quorum requirements thereof. Article X enumerates with particularity the rules for union dues, special assessments, fines, and other payments. Article XII provides the general rule for quorum in meetings of the Board of Directors and of the members of the local/chapter, and cites the applicability of the Robert’s Rules of Order[43] in its meetings. And finally, Article XVI governs and institutes the requisites for the amendment of the constitution. The Court likewise sees no impediment in deeming respondent as having acquired legal personality as of 15 June 1998, the fact that it was the local/chapter itself, and not the FFW, which submitted the documents required under Section 1, Rule VI of Department Order No. 9. The evident rationale why the rule states that it is the federation or national union that submits said documents to the Bureau or Regional Office is that the creation of the local/chapter is the sole prerogative of the federation or national union, and not of any other entity. Certainly, a putative local/chapter cannot, without the imprimatur of the federation or national union, claim affiliation with the larger unit or source its legal personality therefrom. In the ordinary course, it should have been FFW, and not respondent, which should have submitted the subject documents to the Regional Office. Nonetheless, there is no good reason to deny legal personality or defer its conferral to the local/chapter if it is evident at the onset that the federation or national union itself has already through its own means established the local/chapter. In this case, such is evidenced by the Charter Certificate dated 9 June 1998, issued by FFW, and attached to the petition for certification election. The Charter Certificate expressly states that respondent has been issued the said certificate “to operate as a local or chapter of the [FFW]”. The Charter Certificate expressly acknowledges FFW’s intent to establish respondent as of 9 June 1998. This being the case, we consider it permissible for respondent to have submitted the required documents itself to the Regional Office, and proper that respondent’s legal personality be deemed existent as of 15 June 1998, the date the complete documents were submitted.

2. No, it does not. Under the law, a managerial employee is “one who is vested with powers or prerogatives to lay down and execute management policies and/or to hire, transfer, suspend, layoff, recall, discharge, assign or discipline employees.” A supervisory employee is “one who, in the interest of the employer, effectively recommends managerial actions if the exercise of such recommendatory authority is not merely routinary or clerical in nature but requires the use of independent judgment.’” Finally, “all employees not falling within the definition of managerial or supervisory employee are considered rank-and-file employees”. It is also well-settled that the actual functions of an employee, not merely his job title, are determinative in classifying such employee as managerial, supervisory or rank and file. Good faith is presumed in all representations, an essential element of fraud, false statement and misrepresentation in

order for these to be actionable is intent to mislead by the party making the representation. In this case, there is no proof to show that Bathan, or appellee union for that matter, intended to mislead anyone. If this was appellee union’s intention, it would have refrained from using a more precise description of the organization instead of declaring that the organization is composed of ‘rank and file monthlies’. Hence, the charge of fraud, false statement or misrepresentation cannot be sustained.Even if they are supervisory employees, no action can be done that emasculates the right to self-organization and the promotion of free trade unionism. We take administrative notice of the realities in union organizing, during which the organizers must take their chances, oftentimes unaware of the fine distinctions between managerial, supervisory and rank and file employees. The grounds for cancellation of union registration are not meant to be applied automatically, but indeed with utmost discretion. Where a remedy short of cancellation is available, that remedy should be preferred. In this case, no party will be prejudiced if Bathan were to be excluded from membership in the union. The vacancy he will thus create can then be easily filled up through the succession provision of appellee union’s constitution and by-laws. What is important is that there is an unmistakeable intent of the members of appellee union to exercise their right to organize. We cannot impose rigorous restraints on such right if we are to give meaning to the protection to labor and social justice clauses of the Constitution.

3. No, it does not affect the case.The allegation that the bargaining unit that respondent sought to represent is no longer the same because of the dynamic nature of petitioner’s business, a lot of changes having occurred in the work environment, and that four of respondent’s officers are no longer connected with petitioner have no effect on the Court’s ruling that a certification election should be immediately conducted with respondent as one of the available choices. Petitioner’s bare manifestations adduce no reason why the certification election should not be conducted forthwith. If there are matters that have arisen since the filing of the petition that serve to delay or cancel the election, these can be threshed out during the pre-election conferences. Neither is the fact that some of respondent’s officers have since resigned from petitioner of any moment. The local/chapter retains a separate legal personality from that of its officers or members that remains viable notwithstanding any turnover in its officers or members.

DISPOSITION The Petition is DENIED. Costs against petitioner.

COASTAL SUBIC BAY TERMINAL, INC. V DOLE507 SCRA 300

QUISUMBING; November 20, 2006

NATURE For review on certiorari

FACTS- Respondents Coastal Subic Bay Terminal, Inc. Rank-and-File Union (CSBTI-RFU) and Coastal Subic Bay Terminal, Inc. Supervisory Union (CSBTI-SU) filed separate petitions for certification election - The rank-and-file union insists that it is a legitimate labor organization having been issued a charter certificate by the Associated Labor Union (ALU), and the supervisory union by the Associated Professional, Supervisory, Office and Technical Employees Union (APSOTEU). - Petitioner Coastal Subic Bay Terminal, Inc. (CSBTI) opposed both petitions for certification election alleging that the rank-and-file union and supervisory union were not legitimate labor organizations- Petitioner argues that APSOTEU improperly secured its registration from the DOLE Regional Director and not from the BLR - Petitioner insists that APSOTEU lacks legal personality, and its chartered affiliate CSBTI-SU cannot attain the status of a legitimate

Page 33: 33994637 Labor Rounds 1and2

Labor Law 2 A2010 - 33 - Disinilabor organization to file a petition for certification election. - Without ruling on the legitimacy of the respondent unions, the Med-Arbiter dismissed, both petitions - Both parties appealed to the Secretary of Labor and Employment, who reversed the decision of the Med-Arbiter. The Secretary ruled that CSBTI-SU and CSBTI-RFU have separate legal personalities to file their separate petitions for certification election. The Secretary held that APSOTEU is a legitimate labor organization because it was properly registered pursuant to the 1989 Revised Rules and Regulations implementing Republic Act No. 6715, the rule applicable at the time of its registration. It further ruled that ALU and APSOTEU are separate and distinct labor unions having separate certificates of registration from the DOLE. They also have different sets of locals. - The motion for reconsideration was also denied.- On appeal, the Court of Appeals affirmed the decision of the Secretary. - The motion for reconsideration was likewise denied. Hence, the instant petition by the company anchored on the following grounds:

ISSUES1. WON the CA erred in relying on the “1989 Revised Rules and Regulations implementing RA 6715” as basis to recognize private respondent APSOTEU’s registration by the DOLE Regional Director.2. WON ALU, a rank-and-file union and APSOTEU, a supervisory union one and the same because of the commonalities between them. 3. WON the supervisory and rank-and-file unions could separately petition for certification elections.

HELD1. NO. Ratio Article 235 of LC provides that applications for registration shall be acted upon by the Bureau. “Bureau” as defined under the Labor Code means the BLR and/or the Labor Relations Division in the Regional Offices of the Department of Labor. - Section 2, Rule II, Book V of the 1989 Revised Implementing Rules of the Labor Code (Implementing Rules) provides that:Section 2. Where to file application; procedure - Any national labor organization or labor federation or local union may file an application for registration with the Bureau or the Regional Office where the applicant’s principal offices is located. The Bureau or the Regional Office shall immediately process and approve or deny the application. In case of approval, the Bureau or the Regional Office shall issue the registration certificate within thirty (30) calendar days from receipt of the application, together with all the requirements for registration as hereinafter provided. The Implementing Rules specifically Section 1, Rule III of Book V, as amended by Department Order No. 9, thus: SECTION 1. Where to file applications. - The application for registration of any federation, national or industry union or trade union center shall be filed with the Bureau. Where the application is filed with the Regional Office, the same shall be immediately forwarded to the Bureau within forty-eight (48) hours from filing thereof, together with all the documents supporting the registration.The applications for registration of an independent union shall be filed with and acted upon by the Regional Office where the applicant’s principal office is located ….- The DOLE issued Department Order No. 40-03, which took effect on March 15, 2003, further amending Book V of the above implementing rules. The new implementing rules explicitly provide that applications for registration of labor organizations shall be filed either with the Regional Office or with the BLR.- Even after the amendments, the rules did not divest the Regional Office and the BLR of their jurisdiction over applications for registration by labor organizations. The amendments to the implementing rules merely specified that when the application was filed with the Regional Office, the application would be acted upon by the BLR. Reasoning The records in this case showed that APSOTEU was registered on March 1, 1991. Accordingly, the law applicable at that time was Section 2, Rule II, Book V of the Implementing Rules, and not Department Order No. 9 which took effect only on June 21, 1997. Thus, considering further that APSOTEU’s principal office is located in Diliman, Quezon City, and its registration was filed with the NCR Regional Office, the certificate of registration is valid.

2. NO.Ratio Once a labor union attains the status of a legitimate labor organization, it continues as such until its certificate of registration is cancelled or revoked in an independent action for cancellation. In addition, the legal personality of a labor organization cannot be collaterally attacked. Reasoning In the absence of any independent action for cancellation of registration against either APSOTEU or ALU, and unless and until their registrations are cancelled, each continues to possess a separate legal personality. The CSBTI-RFU and CSBTI-SU are therefore affiliated with distinct and separate federations, despite the commonalities of APSOTEU and ALU. - Under the rules implementing the Labor Code, a chartered local union acquires legal personality through the charter certificate issued by a duly registered federation or national union, and reported to the Regional Office in accordance with the rules implementing the Labor Code. A local union does not owe its existence to the federation with which it is affiliated. It is a separate and distinct voluntary association owing its creation to the will of its members. Mere affiliation does not divest the local union of its own personality, neither does it give the mother federation the license to act independently of the local union. It only gives rise to a contract of agency, where the former acts in representation of the latter. Hence, local unions are considered principals while the federation is deemed to be merely their agent. As such principals, the unions are entitled to exercise the rights and privileges of a legitimate labor organization, including the right to seek certification as the sole and exclusive bargaining agent in the appropriate employer unit.

3. NO. Ratio Under Article 245 of the Labor Code, supervisory employees are not eligible for membership in a labor union of rank-and-file employees. The supervisory employees are allowed to form their own union but they are not allowed to join the rank-and-file union because of potential conflicts of interest. Further, to avoid a situation where supervisors would merge with the rank-and-file or where the supervisors’ labor union would represent conflicting interests, a local supervisors’ union should not be allowed to affiliate with the national federation of unions of rank-and-file employees where that federation actively participates in the union activity within the company. Thus, the limitation is not confined to a case of supervisors wanting to join a rank-and-file union. The prohibition extends to a supervisors’ local union applying for membership in a national federation the members of which include local unions of rank-and-file employees. Reasoning In the instant case, the national federations that exist as separate entities to which the rank-and-file and supervisory unions are separately affiliated with, do have a common set of officers. In addition, APSOTEU, the supervisory federation, actively participates in the CSBTI-SU while ALU, the rank-and-file federation, actively participates in the CSBTI-RFU, giving occasion to possible conflicts of interest among the common officers of the federation of rank-and-file and the federation of supervisory unions. For as long as they are affiliated with the APSOTEU and ALU, the supervisory and rank-and-file unions both do not meet the criteria to attain the status of legitimate labor organizations, and thus could not separately petition for certification elections. The purpose of affiliation of the local unions into a common enterprise is to increase the collective bargaining power in respect of the terms and conditions of labor. When there is commingling of officers of a rank-and-file union with a supervisory union, the constitutional policy on labor is circumvented. Labor organizations should ensure the freedom of employees to organize themselves for the purpose of leveling the bargaining process but also to ensure the freedom of workingmen and to keep open the corridor of opportunity to enable them to do it for themselves.

Disposition The petition was GRANTED.

Page 34: 33994637 Labor Rounds 1and2

Labor Law 2 A2010 - 34 - DisiniB. Requirements and Rationale

PROGRESSIVE DEVELOPMENT CORP. V LAGUESMA205 SCRA 802

GUTIERREZ; FEB. 4,1992

NATUREPetition for certiorari to set aside the resolution of Med-arbiter and two orders the of the Sec. of Labor.

FACTSRespondent Kilusan filed with the Department of Labor and Employment (DOLE) a petition for certification election among the rank-and-file employees of the petitioner alleging that it is a legitimate labor federation and its local chapter, Progressive Development Employees Union, was issued charter certificate No. 90-6-1-153. Kilusan claimed that there was no existing collective bargaining agreement and that no other legitimate labor organization existed in the bargaining unit.

Petitioner PDC filed its motion to dismiss contending that the local union failed to comply with Rule II, Section 3, Book V of the Rules Implementing the Labor Code, as amended, which requires the submission of: (a) the constitution and by-laws; (b) names, addresses and list of officers and/or members; and (c) books of accounts.

Kilusan claimed that it had submitted the necessary documentary requirements for registration, such as the constitution and by-laws of the local union, and the list of officers/members with their addresses. Kilusan further averred that no books of accounts could be submitted as the local union was only recently organized.

Petitioner insisted that upon verification with the Bureau of Labor Relations (BLR), it found that the alleged minutes of the organizational meeting was unauthenticated, the list of members did not bear the corresponding signatures of the purported members, and the constitution and by-laws did not bear the signatures of the members and was not duly subscribed. It argued that the private respondent therefore failed to substantially comply with the registration requirements provided by the rules.

Med-Arbiter held that there was substantial compliance with the requirements for the formation of a chapter. Undersecretary Laguesma denied PDC's motion for reconsideration. Hence, this petition.

ISSUEWhen does a branch, local or affiliate of a federation become a legitimate labor organization.

HELDOrdinarily, a labor organization acquires legitimacy only upon registration with the BLR. Under Article 234 (Requirements of Registration):

Any applicant labor organization, association or group of unions or workers shall acquire legal personality and shall be entitled to the rights and privileges granted by law to legitimate labor organizations upon issuance of the certificate of registration based on the following requirements: (a) Fifty-pesos (P50.00) registration fee;

(b) The names of its officers, their addresses, the principal address of the labor organization, the minutes of the organizational meetings and the list of the workers who participated in such meetings;(c) The names of all its members comprising at least twenty 20% percent of all the employees in the bargaining unit where it seeks to operate;(d) If the applicant has been in existence for one or more years, copies of its annual financial reports; and(e) Four copies of the constitution and by-laws of the applicant union, the minutes of its adoption or ratification and the list of the members who participated in it."

And under Article 235 (Action on Application):"The Bureau shall act on all applications for registration within thirty (30) days from filing.

All requisite documents and papers shall be certified under oath by the secretary or the treasurer of the organization, as the case may be, and attested to by its president."

Moreover, section 4 of Rule II, Book V of the Implementing Rules requires that the application should be signed by at least twenty percent (20%) of the employees in the appropriate bargaining unit and be accompanied by a sworn statement of the applicant union that there is no certified bargaining agent or, where there is an existing collective agreement duly submitted to the DOLE, that the application is filed during the last sixty (60) days of the agreement.

RATIONALE: The registration prescribed in is not a limitation to the right of assembly or association, which may be exercised with or without said registration. The latter is merely a condition sine qua non for the acquisition of legal personality by labor organizations, associations or unions and the possession of the "rights and privileges granted by law to legitimate labor organizations." The Constitution does not guarantee these rights and privileges, much less said personality, which are mere statutory creations, for the possession and exercise of which registration is required to protect both labor and the public against abuses, fraud, or impostors who pose as organizers, although not truly accredited agents of the union the purport to represent. Such requirement is a valid exercise of the police power, because the activities in which labor organizations, associations and unions of workers are engaged affect public interest, which should be protected. Furthermore, the obligation to submit financial statements, as a condition for the non-cancellation of a certificate of registration, is a reasonable regulation for the benefit of the members of the organization, considering that the same generally solicits funds or membership, as well as oftentimes collects, on behalf of its members, huge amounts of money due to them or to the organization.

But when an unregistered union becomes a branch, local or chapter of a federation, some of the aforementioned requirements for registration are no longer required. The provisions governing union affiliation are found in Rule II, Section 3, Book V of the Implementing Rules.

A local or chapter therefore becomes a legitimate labor organization only upon submission of the following to the BLR:

1) A charter certificate, within 30 days from its issuance by the labor federation or national union, and

Page 35: 33994637 Labor Rounds 1and2

Labor Law 2 A2010 - 35 - Disini

2) The constitution and by-laws, a statement on the set of officers, and the books of accounts all of which are certified under oath by the secretary or treasurer, as the case may be, of such local or chapter, and attested to by its president.

RATIONALE: The intent of the law in imposing lesser requirements in the case of a branch or local of a registered federation or national union is to encourage the affiliation of a local union with a federation or national union in order to increase the local union's bargaining powers respecting terms and conditions of labor.

Absent compliance with these mandatory requirements, the local or chapter does not become a legitimate labor organization.

In the case at bar, the failure of the secretary of PDEU-Kilusan to certify the required documents under oath is fatal to its acquisition of a legitimate status.

DISPOSITION Petition is GRANTED. The assailed resolution and orders of respondents Med-Arbiter and Secretary of Labor and Employment, respectively, are hereby SET ASIDE.

PHOENIX IRON & STEEL CORP V SEC. OF LABOR244 SCRA 173

BELLOSILLO; May 16, 1995

NATUREPetition for review

FACTS- Private respondent PISCOR-ANGLO (Union)asserting to be legitimate labor organization filed a petition for certification election with the Med-Arbiter. Petitioner Phoenix Iron and Steel Corporation (Phoenix) sought clarification of the legal personality of PISCOR-ANGLO.- Med-Arbiter’s ruling: Dismissed petition, holding that PISCOR-ANGLO is not a duly registered labor organization. Record shows that for purposes of registering Piscor Workers Union as an affiliate of ANGLO no books of account was filed before the Burueau of Labor Relations (BLR). The constitution and by-laws and the list of members who ratified the same were not attested to by the union president. The constitution and by-laws was not likewise verified under oath. So, the authority to file petition for certification election is without force and effect. Thus, in the absence of legal personality Piscor Workers Union may not validly authorize ANGLO to file the present petition.- Sec. of Labor reversed Med-Arbiter’s ruling: On appeal, USec. Laguesma, acting by authority of the Sec. of Labor, ruled that PISCOR has complied with the requirements of the law on organization of a local after it was shown that it has submitted duly certified copies of its constitution and by-laws, list of officers and charter certificate and called for the immediate conduct of a certification election

ISSUEWON PISCOR-ANGLO has complied with the requirements to be legitimate labor organization

HELDNORatio A local or chapter becomes a legitimate labor organization only upon submission of the following to the BLR: 1) A charter certificate, within 30 days from its issuance by the labor federation or national union, and 2) The constitution and by-laws, a statement on the set of officers, and the books of accounts all of which are certified under oath by the secretary or treasurer, as the case may be, of such local or chapter, and attested to by its president. Failure of a labor union to submit and certify under oath the required documents submitted with the BLR is fatal to the

attainment of a legitimate status. (Progressive Dev’t Corp v. Sec. of Labor)Reasoning The rationale for the requirements enunciated above: The certification and attestation requirements are preventive measures against the commission of fraud. They likewise afford a measure of protection to unsuspecting employees who may be lured into joining unscrupulous or fly-by-night unions whose sole purpose is to control union funds or to use the union for dubious ends.

Disposition Petition is GRANTED.

PAGAPALAIN HAULERS, INC. V TRAJANO310 SCRA 354

ROMERO; July 15, 1999

NATUREPetition for certiorari

FACTS- Integrated Labor Organization-Pagpalain Haulers Worker’s Union (ILO-PHILS), in a bid to represent the rank-and-file drivers and helpers of petitioner Pagpalain Haulers, Inc. (Pagpalain), filed a petition for certification election with the Department of Labor and Employment. ILO-PHILS attached to the petition copies of its charter certificate, its constitution and by-laws, its books of account, and a list of its officers and their addresses.- Pagpalain filed a motion to dismiss the petition, alleging that ILO-PHILS was not a legitimate labor organization due to its failure to comply with the requirements for registration under the Labor Code. Specifically, it claimed that the books of account submitted by ILO-PHILS were not verified under oath by its treasurer and attested to by its president, a required by Rule II, Book V of the Omnibus Rules Implementing the Labor Code.- ILO-PHILS dismissed Pagpalain’s claims, saying that Department Order No. 9, Series of 1997 had dispensed with the requirement that a local or chapter of a national union submit books of account in order to be registered with the Department of Labor and Employment.- Med-Arbiter decided in favor of ILO-PHILS. Pagpalain appealed. Secretary of Labor dismissed appeal. Hence, this petition. Pagpalain maintains that Department Order No. 9 is null and void because it is illegal for contravening rulings of this Court which, according to Article 8 of the Civil Code, form a part of the legal system of the Philippines; and that dispensing with the books of accounts contravenes public policy.

ISSUEWON Department Order No. 09 is null and void.

HELDNORatio Article 234 of the Labor Code does not require the submission of books of account in order for a labor organization to be registered as a legitimate labor organization. The requirement that books of account be submitted as a requisite for a registration can be found only in Book V of the Omnibus Rules Implementing the Labor Code. Specifically, the old Section 3(e), Rule II, of Book V provided that ‘[t]he local or chapter of a labor federation or national union shall have and maintain a constitution and by-laws, set of officers and books of accounts. For reporting purposes, the procedure governing the reporting of independently registered unions, federations or national unions shall be observed.”- By virtue of Department Order No. 9, Series of 1997, however, the submission of books of account as a requisite for registration has been done away with, and the documents needed to be submitted by a local or chapter have been reduced to the following:

(a) A charter certificate issued by the federation or national union indicating the creation or establishment of the local/chapter;

Page 36: 33994637 Labor Rounds 1and2

Labor Law 2 A2010 - 36 - Disini(b) The names of the local/chapter’s officers, their addresses, and the principal office of the local/chapter;(c) The local/chapter’s constitution and by-laws; provided that where the local/chapter’s constitution and by-laws is the same as that of the federation or national union, this fact shall be indicated accordingly.

Reasoning Department Order No. 9 has been issued on authority of law. Under the law, the Secretary is authorized to promulgate rules and regulations to implement the Labor Code. Specifically, Article 5 of the Labor Code provides that “[t]he Department of Labor and other government agencies charged with the administration and enforcement of this Code or any of its parts shall promulgate the necessary implementing rules and regulations.” Consonant with this article, the Secretary of Labor and Employment promulgated the Omnibus Rules Implementing the Labor Code. By virtue of this self-same authority, the Secretary amended the above-mentioned omnibus rules by issuing Department Order No. 9, Series of 1997.- Cases cited by Pagpalain are not to be deemed as laws on the registration of unions. They merely interpret and apply the implementing rules of the Labor Code as to registration of unions. It is this interpretation that forms part of the legal system of the Philippines, for the interpretation placed upon the written law by a competent court has the force of law. The cited cases, however, applied and interpreted the then existing Book V of the Omnibus Rules Implementing the Labor Code. Since Book V of the Omnibus Rules, as amended by Department Order No. 9, no longer requires a local or chapter to submit books of accounts as a prerequisite for registration, the doctrines enunciated in the cases, with respect to books of account, are already passe and therefore, no longer applicable. Hence, Pagpalain cannot insist that ILO-PHILS comply with the requirements prescribed in said rulings, for the current implementing rules have deleted the same.- Pagpalain cannot also allege that Department Order No. 9 is violative of public policy. As adverted to earlier, the sole function of our courts is to apply or interpret the laws. It does not formulate public policy, which is the province of the legislative and executive branches of government. - Furthermore, the controlling intention in requiring the submission of books of account is the protection of labor through the minimization of the risk of fraud and diversion in the handling of union funds. As correctly pointed out by the Solicitor General, this intention can still be realized through other provisions of the Labor Code. (Article 241 and 274)

Disposition Petition is dismissed.

C. Constitution—by law

SAN MIGUEL CORP. V. MANDAUETinga ; Aug. 16, 2005

467 SCRA 107

Facts-CA affirmes DOLE Undersecretary for Labor Relations, Rosalinda Dimapilis-Baldoz, ordering the immediate conduct of a certification election among the petitioner’s rank-and-file employees. - Federation of Free Workers (FFW/ respondent) filed a petition for certification election with the DOLE Regional Office No. VII. It sought to be certified and to represent the permanent rank-and-file monthly paid employees of the petitioner. The following documents were attached to the petition: (1) a Charter Certificate certifying that respondent as of that date was duly certified as a local or chapter of FFW; (2) a copy of the constitution of respondent prepared by its Secretary, Noel T. Bathan and attested by its President, Wilfred V. Sagun; (3) a list of respondent’s officers and their respective addresses, again prepared by Bathan and attested by Sagun; (4) a certification signifying that respondent had just been organized and no amount had yet been collected from its members, signed by respondent’s treasurer Chita D. Rodriguez and attested by Sagun; and (5) a list of all the rank-and-file monthly paid employees of the Mandaue Packaging Products Plants and Mandaue Glass Plant prepared by Bathan and attested by Sagun. -SMC (Petitioner) filed a motion to dismiss the petition for certification election on the sole ground that herein respondent is not listed or included in the roster of legitimate labor organizations based on the certification

issued by the Officer-In-Charge, Regional Director of the DOLE Regional Office No. VII, Atty. Jesus B. Gabor.-Respondent submitted to the Bureau of Labor Relations the same documents earlier attached to its petition for certification. The accompanying letter, signed by respondent’s president Sagun, stated that such documents were submitted in compliance with the requirements for the creation of a local/chapter pursuant to the Labor Code and its Implementing Rules; and it was hoped that the submissions would facilitate the listing of respondent under the roster of legitimate labor organizations.The Chief of Labor Relations Division of DOLE Regional Office No. VII issued a Certificate of Creation of Local/Chapter No. ITD. I-ARFBT-058/98, certifying that from 30 July 1998, respondent has acquired legal personality as a labor organization/worker’s association, it having submitted all the required documents.

Issues 1. When did the union acquire legal personality?2. Whether or not the inclusion of the two alleged supervisory employees in appellee union’s membership amounts to fraud, misrepresentation, or false statement within the meaning of Article 239(a) and (c) of the Labor Code.3. Whether or not subsequent developments change the disposition of the case

Held1. 15 June 1998, the date the complete documents were submitted.The issuance of the certificate of registration by the Bureau or Regional Office is not the operative act that vests legal personality upon a local/chapter under Department Order No. 9. Such legal personality is acquired from the filing of the complete documentary requirements enumerated in Section 1, Rule VI. Admittedly, the manner by which respondent was deemed to have acquired legal personality by the DOLE and the Court of Appeals was not in strict conformity with the provisions of Department Order No. 9. Thus, in order to ascertain when respondent acquired legal personality, we only need to determine on what date the Regional Office or Bureau received the complete documentary requirements enumerated under Section 1, Rule VI of Department Order No. 9. There is no doubt that on 15 June 1998, or the date respondent filed its petition for certification election, attached thereto were respondent’s constitution, the names and addresses of its officers, and the charter certificate issued by the national union FFW. The first two of these documents were duly certified under oath by respondent’s secretary Bathan and attested to by president Sagun.What about the lack of documents containing the by-laws? Not needed so long as it is part of the union’s constitution.An examination of respondent’s constitution reveals it sufficiently comprehensive in establishing the necessary rules for its operation. Article IV establishes the requisites for membership in the local/chapter. Articles V and VI name the various officers and what their respective functions are. The procedure for election of these officers, including the necessary vote requirements, is provided for in Article IX, while Article XV delineates the procedure for the impeachment of these officers. Article VII establishes the standing committees of the local/chapter and how their members are appointed. Article VIII lays down the rules for meetings of the union, including the notice and quorum requirements thereof. Article X enumerates with particularity the rules for union dues, special assessments, fines, and other payments. Article XII provides the general rule for quorum in meetings of the Board of Directors and of the members of the local/chapter, and cites the applicability of the Robert’s Rules of Order[43] in its meetings. And finally, Article XVI governs and institutes the requisites for the amendment of the constitution. The Court likewise sees no impediment in deeming respondent as having acquired legal personality as of 15 June 1998, the fact that it was the local/chapter itself, and not the FFW, which submitted the documents required under Section 1, Rule VI of Department Order No. 9. The evident rationale why the rule states that it is the federation or national union that

Page 37: 33994637 Labor Rounds 1and2

Labor Law 2 A2010 - 37 - Disinisubmits said documents to the Bureau or Regional Office is that the creation of the local/chapter is the sole prerogative of the federation or national union, and not of any other entity. Certainly, a putative local/chapter cannot, without the imprimatur of the federation or national union, claim affiliation with the larger unit or source its legal personality therefrom. In the ordinary course, it should have been FFW, and not respondent, which should have submitted the subject documents to the Regional Office. Nonetheless, there is no good reason to deny legal personality or defer its conferral to the local/chapter if it is evident at the onset that the federation or national union itself has already through its own means established the local/chapter. In this case, such is evidenced by the Charter Certificate dated 9 June 1998, issued by FFW, and attached to the petition for certification election. The Charter Certificate expressly states that respondent has been issued the said certificate “to operate as a local or chapter of the [FFW]”. The Charter Certificate expressly acknowledges FFW’s intent to establish respondent as of 9 June 1998. This being the case, we consider it permissible for respondent to have submitted the required documents itself to the Regional Office, and proper that respondent’s legal personality be deemed existent as of 15 June 1998, the date the complete documents were submitted.

2. No, it does not. Under the law, a managerial employee is “one who is vested with powers or prerogatives to lay down and execute management policies and/or to hire, transfer, suspend, layoff, recall, discharge, assign or discipline employees.” A supervisory employee is “one who, in the interest of the employer, effectively recommends managerial actions if the exercise of such recommendatory authority is not merely routinary or clerical in nature but requires the use of independent judgment.’” Finally, “all employees not falling within the definition of managerial or supervisory employee are considered rank-and-file employees”. It is also well-settled that the actual functions of an employee, not merely his job title, are determinative in classifying such employee as managerial, supervisory or rank and file. Good faith is presumed in all representations, an essential element of fraud, false statement and misrepresentation in order for these to be actionable is intent to mislead by the party making the representation. In this case, there is no proof to show that Bathan, or appellee union for that matter, intended to mislead anyone. If this was appellee union’s intention, it would have refrained from using a more precise description of the organization instead of declaring that the organization is composed of ‘rank and file monthlies’. Hence, the charge of fraud, false statement or misrepresentation cannot be sustained.Even if they are supervisory employees, no action can be done that emasculates the right to self-organization and the promotion of free trade unionism. We take administrative notice of the realities in union organizing, during which the organizers must take their chances, oftentimes unaware of the fine distinctions between managerial, supervisory and rank and file employees. The grounds for cancellation of union registration are not meant to be applied automatically, but indeed with utmost discretion. Where a remedy short of cancellation is available, that remedy should be preferred. In this case, no party will be prejudiced if Bathan were to be excluded from membership in the union. The vacancy he will thus create can then be easily filled up through the succession provision of appellee union’s constitution and by-laws. What is important is that there is an unmistakeable intent of the members of appellee union to exercise their right to organize. We cannot impose rigorous restraints on such right if we are to give meaning to the protection to labor and social justice clauses of the Constitution.

3. No, it does not affect the case.The allegation that the bargaining unit that respondent sought to represent is no longer the same because of the dynamic nature of petitioner’s business, a lot of changes having occurred in the work environment, and that four of respondent’s officers are no longer connected with petitioner have no effect on the Court’s ruling that a certification election should be immediately conducted with respondent as one of the available choices. Petitioner’s bare manifestations adduce no reason why the certification election should not be conducted forthwith. If there are matters that have arisen since the filing of the petition that serve to delay or cancel the election, these can be threshed out during the pre-

election conferences. Neither is the fact that some of respondent’s officers have since resigned from petitioner of any moment. The local/chapter retains a separate legal personality from that of its officers or members that remains viable notwithstanding any turnover in its officers or members.

DISPOSITIONWHEREFORE, the Petition is DENIED. Costs against petitioner.SO ORDERED.

D. Effect on Constitution – Statutory Guarantee Freedom of Association

PHILIPPINE ASSOCIATION OF FREE LABOR UNIONS V SEC. OF LABOR

27 SCRA 40The requirement of registration does NOT curtail the freedoms of assembly and association. Said freedoms may still be exercised with or without registration. The latter is merely a condition sine qua non for the acquisition of legal personality by labor unions and the possessions of rights and privileges granted by law. The constitution does not guarantee these rights and privileges, much less legal personality, which are mere statutory creations.

E. Question of Legitimacy

PHILIPPINE DIAMOND HOTEL V MANILA DIAMOND HOTEL EMPLOYEES ASSOCIATION

494 SCRA 195PHILIPPINE DIAMOND HOTEL AND RESORT INC (MANILA

DIAMOND HOTEL V MANILA DIAMOND HOTEL EMPLOYEES UNION

494 SCRA 195CARPIO MORALES; June 30, 2006

FACTS-Union filed a petition for certification election to be declared the exclusive bargaining representative of the Hotel’s employees. This petition was dismissed by DOLE for lack of legal requirements.-after a few months, Union sent a letter to Hotel informing it of its desire to negotiate for a collective bargaining agreement. This was rejected by the Hotel stating that the Union was not the employee’s bargaining agent as their petition for certification election was denied.-Union filed a Notice of Strike with the NCMB alleging the Hotel’ refusal to bargain and for acts of unfair labor practices. NCMB summoned both parties and held series of dialogues. Union however suddenly went on strike-Secretary of DOLE assumed jurisdiction and ordered compulsory arbitration pursuant to art. 263 (g) of LC. And Union members were directed to return to work and for Hotel to accept them back. Hotel refused to accept the employees return. The order was modified (by a different Secretary) such that reinstatement was to be done only in the payroll.-Union filed for certiorari alleging grave abuse of discretion. Case was referred to the CA. CA affirmed that the “payroll reinstatement” was not a grave abuse of discretion. On appeal, it modified NLRC decision ordering reinstatement with back wages of union members.

ISSUE1) WON the Union can bargain only in behalf of its members and not for all the employees of the Hotel.2) WON the strike conducted by the Union was illegal.3) WON those employees who participated in the strike should be given back wages

HELD1) No.-As provided by art 255 of the LC only the labor organization designated or selected by the majority of the employees in an appropriate collective bargaining unit is the exclusive representative of the employees in such unit for the purpose of collective bargaining.

Page 38: 33994637 Labor Rounds 1and2

Labor Law 2 A2010 - 38 - Disini-The Union’s petition for certificate election was denied by the DOLE. The union thus is admittedly not the exclusive representative of the majority of the employees of petitioner, hence, it could not demand from petitioner the right to bargain collectively in their behalf-Respondent insists, however, that it could validly bargain in behalf of "its members," relying on Article 242 of the Labor Code.-the CA ruled that “what [respondent] will be achieving is to divide the employees, more particularly, the rank-and-file employees of [petitioner] . . . the other workers who are not members are at a serious disadvantage, because if the same shall be allowed, employees who are non-union members will be economically impaired and will not be able to negotiate their terms and conditions of work, thus defeating the very essence and reason of collective bargaining, which is an effective safeguard against the evil schemes of employers in terms and conditions of work”- Petitioner’s refusal to bargain then with respondent can not be considered a ULP to justify the staging of the strike.TOPICS: Union registration and procedure, factors, majority union

2) yes.-as was mentioned, the first ground mentioned by the Union- the Hotel’s refusal to bargain- was not a valid ground to stage the strike.-The second ground – that petitioner prevented or intimidated some workers from joining the union before, during or after the strike – was correctly discredited by the appellate court.. Since it is the union who alleges that unfair labor practices were committed by the Hotel, the burden of proof is on the union to prove its allegations by substantial evidence. “the facts and the evidence did not establish events [sic] least a rational basis why the union would [wield] a strike based on alleged unfair labor practices it did not even bother to substantiate”.-It is doctrinal that the exercise of the right of private sector employees to strike is not absolute. Thus Section 3 of Article XIII of the Constitution, provides:SECTION 3. x x xIt shall guarantee the rights of all workers to self-organization, collective bargaining and negotiations and peaceful concerted activities, including the right to strike in accordance with law…”-Even if the purpose of a strike is valid, the strike may still be held illegal where the means employed are illegal. Thus, the employment of violence, intimidation, restraint or coercion in carrying out concerted activities which are injurious to the rights to property renders a strike illegal. Evidence show s that some of the workers-strikers who joined the strike indeed committed illegal acts – blocking the free ingress to and egress from the Hotel, holding noise barrage, threatening guests, and the like.TOPICS: right to engage in concerted activities-limitations

3) No.-The general rule is that backwages shall not be awarded in an economic strike on the principle that "a fair day’s wage" accrues only for a "fair day’s labor. If there is no work performed by the employee there can be no wage or pay, unless of course, the laborer was able, willing and ready to work but was illegally locked out, dismissed or suspended. "when employees voluntarily go on strike, even if in protest against unfair labor practices," no back wages during the strike is awarded.”-The Court ruled that “only those members of the union who did not commit illegal acts during the course of the illegal strike should be reinstated but without back wages”TOPIC: illegal strikes- burden of economic loss

SAN MIGUEL CORP. V MANDAUE467 SCRA 107

Tinga ; Aug. 16, 2005

Facts-CA affirmes DOLE Undersecretary for Labor Relations, Rosalinda Dimapilis-Baldoz, ordering the immediate conduct of a certification election among the petitioner’s rank-and-file employees. - Federation of Free Workers (FFW/ respondent) filed a petition for certification election with the DOLE Regional Office No. VII. It sought to be certified and to represent the permanent rank-and-file monthly paid employees of the petitioner. The following documents were attached to the petition: (1) a Charter Certificate certifying that respondent as of that date was duly certified as a local or chapter of FFW; (2) a copy of the constitution of respondent prepared by its Secretary, Noel T. Bathan and attested by its President, Wilfred V. Sagun; (3) a list of respondent’s officers and their respective addresses, again prepared by Bathan and attested by Sagun; (4) a certification signifying that respondent had just been organized and no amount had yet been collected from its members, signed by respondent’s treasurer Chita D. Rodriguez and attested by Sagun; and (5) a list of all the rank-and-file monthly paid employees of the Mandaue Packaging Products Plants and Mandaue Glass Plant prepared by Bathan and attested by Sagun. -SMC (Petitioner) filed a motion to dismiss the petition for certification election on the sole ground that herein respondent is not listed or included

in the roster of legitimate labor organizations based on the certification issued by the Officer-In-Charge, Regional Director of the DOLE Regional Office No. VII, Atty. Jesus B. Gabor.-Respondent submitted to the Bureau of Labor Relations the same documents earlier attached to its petition for certification. The accompanying letter, signed by respondent’s president Sagun, stated that such documents were submitted in compliance with the requirements for the creation of a local/chapter pursuant to the Labor Code and its Implementing Rules; and it was hoped that the submissions would facilitate the listing of respondent under the roster of legitimate labor organizations.The Chief of Labor Relations Division of DOLE Regional Office No. VII issued a Certificate of Creation of Local/Chapter No. ITD. I-ARFBT-058/98, certifying that from 30 July 1998, respondent has acquired legal personality as a labor organization/worker’s association, it having submitted all the required documents.

Issues 1. When did the union acquire legal personality?2. Whether or not the inclusion of the two alleged supervisory employees in appellee union’s membership amounts to fraud, misrepresentation, or false statement within the meaning of Article 239(a) and (c) of the Labor Code.3. Whether or not subsequent developments change the disposition of the case

Held1. 15 June 1998, the date the complete documents were submitted.The issuance of the certificate of registration by the Bureau or Regional Office is not the operative act that vests legal personality upon a local/chapter under Department Order No. 9. Such legal personality is acquired from the filing of the complete documentary requirements enumerated in Section 1, Rule VI. Admittedly, the manner by which respondent was deemed to have acquired legal personality by the DOLE and the Court of Appeals was not in strict conformity with the provisions of Department Order No. 9. Thus, in order to ascertain when respondent acquired legal personality, we only need to determine on what date the Regional Office or Bureau received the complete documentary requirements enumerated under Section 1, Rule VI of Department Order No. 9. There is no doubt that on 15 June 1998, or the date respondent filed its petition for certification election, attached thereto were respondent’s constitution, the names and addresses of its officers, and the charter certificate issued by the national union FFW. The first two of these documents were duly certified under oath by respondent’s secretary Bathan and attested to by president Sagun.What about the lack of documents containing the by-laws? Not needed so long as it is part of the union’s constitution.An examination of respondent’s constitution reveals it sufficiently comprehensive in establishing the necessary rules for its operation. Article IV establishes the requisites for membership in the local/chapter. Articles V and VI name the various officers and what their respective functions are. The procedure for election of these officers, including the necessary vote requirements, is provided for in Article IX, while Article XV delineates the procedure for the impeachment of these officers. Article VII establishes the standing committees of the local/chapter and how their members are appointed. Article VIII lays down the rules for meetings of the union, including the notice and quorum requirements thereof. Article X enumerates with particularity the rules for union dues, special assessments, fines, and other payments. Article XII provides the general rule for quorum in meetings of the Board of Directors and of the members of the local/chapter, and cites the applicability of the Robert’s Rules of Order[43] in its meetings. And finally, Article XVI governs and institutes the requisites for the amendment of the constitution. The Court likewise sees no impediment in deeming respondent as having acquired legal personality as of 15 June 1998, the fact that it was the local/chapter itself, and not the FFW, which submitted the documents required under Section 1, Rule VI of Department Order No. 9. The evident rationale why

Page 39: 33994637 Labor Rounds 1and2

Labor Law 2 A2010 - 39 - Disinithe rule states that it is the federation or national union that submits said documents to the Bureau or Regional Office is that the creation of the local/chapter is the sole prerogative of the federation or national union, and not of any other entity. Certainly, a putative local/chapter cannot, without the imprimatur of the federation or national union, claim affiliation with the larger unit or source its legal personality therefrom. In the ordinary course, it should have been FFW, and not respondent, which should have submitted the subject documents to the Regional Office. Nonetheless, there is no good reason to deny legal personality or defer its conferral to the local/chapter if it is evident at the onset that the federation or national union itself has already through its own means established the local/chapter. In this case, such is evidenced by the Charter Certificate dated 9 June 1998, issued by FFW, and attached to the petition for certification election. The Charter Certificate expressly states that respondent has been issued the said certificate “to operate as a local or chapter of the [FFW]”. The Charter Certificate expressly acknowledges FFW’s intent to establish respondent as of 9 June 1998. This being the case, we consider it permissible for respondent to have submitted the required documents itself to the Regional Office, and proper that respondent’s legal personality be deemed existent as of 15 June 1998, the date the complete documents were submitted.

2. No, it does not. Under the law, a managerial employee is “one who is vested with powers or prerogatives to lay down and execute management policies and/or to hire, transfer, suspend, layoff, recall, discharge, assign or discipline employees.” A supervisory employee is “one who, in the interest of the employer, effectively recommends managerial actions if the exercise of such recommendatory authority is not merely routinary or clerical in nature but requires the use of independent judgment.’” Finally, “all employees not falling within the definition of managerial or supervisory employee are considered rank-and-file employees”. It is also well-settled that the actual functions of an employee, not merely his job title, are determinative in classifying such employee as managerial, supervisory or rank and file. Good faith is presumed in all representations, an essential element of fraud, false statement and misrepresentation in order for these to be actionable is intent to mislead by the party making the representation. In this case, there is no proof to show that Bathan, or appellee union for that matter, intended to mislead anyone. If this was appellee union’s intention, it would have refrained from using a more precise description of the organization instead of declaring that the organization is composed of ‘rank and file monthlies’. Hence, the charge of fraud, false statement or misrepresentation cannot be sustained.Even if they are supervisory employees, no action can be done that emasculates the right to self-organization and the promotion of free trade unionism. We take administrative notice of the realities in union organizing, during which the organizers must take their chances, oftentimes unaware of the fine distinctions between managerial, supervisory and rank and file employees. The grounds for cancellation of union registration are not meant to be applied automatically, but indeed with utmost discretion. Where a remedy short of cancellation is available, that remedy should be preferred. In this case, no party will be prejudiced if Bathan were to be excluded from membership in the union. The vacancy he will thus create can then be easily filled up through the succession provision of appellee union’s constitution and by-laws. What is important is that there is an unmistakeable intent of the members of appellee union to exercise their right to organize. We cannot impose rigorous restraints on such right if we are to give meaning to the protection to labor and social justice clauses of the Constitution.

3. No, it does not affect the case.The allegation that the bargaining unit that respondent sought to represent is no longer the same because of the dynamic nature of petitioner’s business, a lot of changes having occurred in the work environment, and that four of respondent’s officers are no longer connected with petitioner have no effect on the Court’s ruling that a certification election should be immediately conducted with respondent as one of the available choices. Petitioner’s bare manifestations adduce no reason why the certification election should not be conducted forthwith. If there are matters that have arisen since the filing of the petition that serve

to delay or cancel the election, these can be threshed out during the pre-election conferences. Neither is the fact that some of respondent’s officers have since resigned from petitioner of any moment. The local/chapter retains a separate legal personality from that of its officers or members that remains viable notwithstanding any turnover in its officers or members.

DISPOSITIONWHEREFORE, the Petition is DENIED. Costs against petitioner.SO ORDERED.

LAGUNA V OFFICE OF THE SECRETARY457 SCRA 730

CALLEJO; April 29, 2005 NATUREPetition to review the decision of the Court of Appeals FACTS- May 3, 1999 - Laguna Autoparts Manufacturing Corporation Obrero Pilipino-LAMCOR Chapter filed a petition for certification election with the DOLE.  In its petition, it noted its registration certificate number along with the registration certificate number of chapter affiliate.  The petition alleged that the union was composed of all rank-and-file employees, that the bargaining unit is unorganized and that there had been no certification election for the past 12 months prior to the filing of the petition.- Laguna Autoparts moved to cancel the certification election because the union was not considered a legitimate labor organization for failure to show that it had complied with registration requirements such as submission of required documents to the Bureau of Labor Relations.- Med-Arbiter Bactin dismissed the certification election petition because of the union’s lack of personality.  DOLE Secretary Sto. Tomas reversed Bactin’s order.  This was affirmed by the CA.

- The CA stressed that a local or chapter need not be registered to become a legitimate labor organization. It pointed out that a local or chapter acquires legal personality as a labor organization from the date of filing of the complete documents.  - The CA noted that it was the employer which offered the most tenacious resistance to the holding of a certification election among its regular rank-and-file employees. It opined that this must not be so for the choice of a collective bargaining agent was the sole concern of the employees, and the employer should be a mere bystander.

 ISSUE/S

1.        WON the union is a legitimate labor organization2.        WON the chapter’s legal personality can be attacked collaterally in an election action

 HELD1. YESRatio A local or chapter need not be independently registered to acquire legal personality.  A local/chapter constituted in accordance with Section 1 of Rule 6 shall acquire legal personality from the date of filing of the complete documents enumerated.  ReasoningThe task of determining whether the local or chapter has submitted the complete documentary requirements is lodged with the Regional Office or the BLR, as the case may be. The records of the case show that the respondent union submitted the said documents to the regional office and was issued a certification.   2. NORatio

Page 40: 33994637 Labor Rounds 1and2

Labor Law 2 A2010 - 40 - DisiniReasoning- It may not be subject to a collateral attack but only through a separate action instituted particularly for the purpose of assailing the chapter, as prescribed in Section 5, Rule 5 of the implementing rules of Book 5 which states: “The labor organization or workers’ association shall be deemed registered and vested with legal personality on the date of issuance of its certificate of registration. Such legal personality cannot thereafter be subject to collateral attack but may be questioned only in an independent petition for cancellation in accordance with these Rules.[”- The pronouncement of the Labor Relations Division Chief, that the respondent union acquired a legal personality with the submission of the complete documentary requirement, cannot be challenged in a petition for certification election.Disposition Petition is denied. No costs.

TAGAYTARY HIGHLANDS INTERNATIONAL GOLF CLUB, INC. V TAGAYTAY HIGHLANDS EMPLOYEES

UNION395 SCRA 699

CARPIO-MORALES; January 22, 2003

NATUREPetition for Certiorari under Rule 45 THIGCI assailing CA decision denying its petition to annul the Department of Labor and Employment (DOLE) Resolutions of November 12, 1998 and December 29, 1998

FACTS- October 16, 1997 > Tagaytay Highlands Employees Union (THEU), Philippine Transport and General Workers Organization (PTGWO), Local Chapter No. 776, a legitimate labor organization said to represent majority of the rank-and-file employees of THIGCI, filed a petition for certification election before the DOLE Mediation-Arbitration Unit, Regional Branch No. IV- November 27, 1997 > opposed petition for certification election because the list of union members submitted by it was defective and fatally flawed as it included the names and signatures of supervisors, resigned, terminated and absent without leave (AWOL) employees, as well as employees of The Country Club, Inc., a corporation distinct and separate from THIGCI; and that out of the 192 signatories to the petition, only 71 were actual rank-and-file employees of THIGCI. Also, some of the signatures in the list of union members were secured through fraudulent and deceitful means, and submitted copies of the handwritten denial and withdrawal of some of its employees from participating in the petition.- THEU asserted that it complied with all the requirements for valid affiliation and inclusion in the roster of legitimate labor organizations pursuant to DOLE Department Order No. 9, series of 1997, on account of which it was duly granted a Certification of Affiliation by DOLE on October 10, 1997; and that Section 5, Rule V of said Department Order provides that the legitimacy of its registration cannot be subject to collateral attack, and for as long as there is no final order of cancellation, it continues to enjoy the rights accorded to a legitimate organization. Therefore, the Med-Arbiter should, pursuant to Article 257 of the Labor Code and Section 11, Rule XI of DOLE Department Order No. 09, automatically order the conduct of a certification election. - January 28, 1998 > DOLE Med-Arbiter Anastacio Bactin ordered the holding of a certification election - DOLE Resolution of November 12, 19981 > setting aside the June 4, 1998 Resolution dismissing the petition for certification election. MFR denied - CA - denied THIGCI’s Petition for Certiorari and affirmed the DOLE Resolution dated November 12, 1998. It held that while a petition for certification election is an exception to the innocent bystander rule, hence, the employer may pray for the dismissal of such petition on the basis of lack of mutuality of interests of the members of the union as well as lack of employer-employee relationship and petitioner failed to adduce substantial evidence to support its allegations.Issue: WON question of legitimacy would affect the certification election

Held: YES- Pizza Hut v. Ledesma which held that:

"The Labor Code requires that in organized and unorganized establishments, a petition for certification election must be filed by a legitimate labor organization. The acquisition of rights by any union or labor organization, particularly the right to file a petition for certification election, first and foremost, depends on whether or not the labor organization has attained the status of a legitimate labor organization.In the case before us, the Med-Arbiter summarily disregarded the petitioner’s prayer that the former look into the legitimacy of the respondent Union by a sweeping declaration that the union was in the

possession of a charter certificate so that for all intents and purposes, Sumasaklaw sa Manggagawa sa Pizza Hut (was) a legitimate organization"

2. Action or Denial of Application and Remedy

SAN MIGUEL CORP. V. MANDAUETinga ; Aug. 16, 2005

467 SCRA 107

Facts-CA affirmes DOLE Undersecretary for Labor Relations, Rosalinda Dimapilis-Baldoz, ordering the immediate conduct of a certification election among the petitioner’s rank-and-file employees. - Federation of Free Workers (FFW/ respondent) filed a petition for certification election with the DOLE Regional Office No. VII. It sought to be certified and to represent the permanent rank-and-file monthly paid employees of the petitioner. The following documents were attached to the petition: (1) a Charter Certificate certifying that respondent as of that date was duly certified as a local or chapter of FFW; (2) a copy of the constitution of respondent prepared by its Secretary, Noel T. Bathan and attested by its President, Wilfred V. Sagun; (3) a list of respondent’s officers and their respective addresses, again prepared by Bathan and attested by Sagun; (4) a certification signifying that respondent had just been organized and no amount had yet been collected from its members, signed by respondent’s treasurer Chita D. Rodriguez and attested by Sagun; and (5) a list of all the rank-and-file monthly paid employees of the Mandaue Packaging Products Plants and Mandaue Glass Plant prepared by Bathan and attested by Sagun. -SMC (Petitioner) filed a motion to dismiss the petition for certification election on the sole ground that herein respondent is not listed or included in the roster of legitimate labor organizations based on the certification issued by the Officer-In-Charge, Regional Director of the DOLE Regional Office No. VII, Atty. Jesus B. Gabor.-Respondent submitted to the Bureau of Labor Relations the same documents earlier attached to its petition for certification. The accompanying letter, signed by respondent’s president Sagun, stated that such documents were submitted in compliance with the requirements for the creation of a local/chapter pursuant to the Labor Code and its Implementing Rules; and it was hoped that the submissions would facilitate the listing of respondent under the roster of legitimate labor organizations.The Chief of Labor Relations Division of DOLE Regional Office No. VII issued a Certificate of Creation of Local/Chapter No. ITD. I-ARFBT-058/98, certifying that from 30 July 1998, respondent has acquired legal personality as a labor organization/worker’s association, it having submitted all the required documents.

Issues 1. When did the union acquire legal personality?2. Whether or not the inclusion of the two alleged supervisory employees in appellee union’s membership amounts to fraud, misrepresentation, or false statement within the meaning of Article 239(a) and (c) of the Labor Code.3. Whether or not subsequent developments change the disposition of the case

Held1. 15 June 1998, the date the complete documents were submitted.The issuance of the certificate of registration by the Bureau or Regional Office is not the operative act that vests legal personality upon a local/chapter under Department Order No. 9. Such legal personality is acquired from the filing of the complete documentary requirements enumerated in Section 1, Rule VI. Admittedly, the manner by which respondent was deemed to have acquired legal personality by the DOLE and the Court of Appeals was not in strict conformity with the provisions of Department Order No. 9.

Page 41: 33994637 Labor Rounds 1and2

Labor Law 2 A2010 - 41 - DisiniThus, in order to ascertain when respondent acquired legal personality, we only need to determine on what date the Regional Office or Bureau received the complete documentary requirements enumerated under Section 1, Rule VI of Department Order No. 9. There is no doubt that on 15 June 1998, or the date respondent filed its petition for certification election, attached thereto were respondent’s constitution, the names and addresses of its officers, and the charter certificate issued by the national union FFW. The first two of these documents were duly certified under oath by respondent’s secretary Bathan and attested to by president Sagun.What about the lack of documents containing the by-laws? Not needed so long as it is part of the union’s constitution.An examination of respondent’s constitution reveals it sufficiently comprehensive in establishing the necessary rules for its operation. Article IV establishes the requisites for membership in the local/chapter. Articles V and VI name the various officers and what their respective functions are. The procedure for election of these officers, including the necessary vote requirements, is provided for in Article IX, while Article XV delineates the procedure for the impeachment of these officers. Article VII establishes the standing committees of the local/chapter and how their members are appointed. Article VIII lays down the rules for meetings of the union, including the notice and quorum requirements thereof. Article X enumerates with particularity the rules for union dues, special assessments, fines, and other payments. Article XII provides the general rule for quorum in meetings of the Board of Directors and of the members of the local/chapter, and cites the applicability of the Robert’s Rules of Order[43] in its meetings. And finally, Article XVI governs and institutes the requisites for the amendment of the constitution. The Court likewise sees no impediment in deeming respondent as having acquired legal personality as of 15 June 1998, the fact that it was the local/chapter itself, and not the FFW, which submitted the documents required under Section 1, Rule VI of Department Order No. 9. The evident rationale why the rule states that it is the federation or national union that submits said documents to the Bureau or Regional Office is that the creation of the local/chapter is the sole prerogative of the federation or national union, and not of any other entity. Certainly, a putative local/chapter cannot, without the imprimatur of the federation or national union, claim affiliation with the larger unit or source its legal personality therefrom. In the ordinary course, it should have been FFW, and not respondent, which should have submitted the subject documents to the Regional Office. Nonetheless, there is no good reason to deny legal personality or defer its conferral to the local/chapter if it is evident at the onset that the federation or national union itself has already through its own means established the local/chapter. In this case, such is evidenced by the Charter Certificate dated 9 June 1998, issued by FFW, and attached to the petition for certification election. The Charter Certificate expressly states that respondent has been issued the said certificate “to operate as a local or chapter of the [FFW]”. The Charter Certificate expressly acknowledges FFW’s intent to establish respondent as of 9 June 1998. This being the case, we consider it permissible for respondent to have submitted the required documents itself to the Regional Office, and proper that respondent’s legal personality be deemed existent as of 15 June 1998, the date the complete documents were submitted.

2. No, it does not. Under the law, a managerial employee is “one who is vested with powers or prerogatives to lay down and execute management policies and/or to hire, transfer, suspend, layoff, recall, discharge, assign or discipline employees.” A supervisory employee is “one who, in the interest of the employer, effectively recommends managerial actions if the exercise of such recommendatory authority is not merely routinary or clerical in nature but requires the use of independent judgment.’” Finally, “all employees not falling within the definition of managerial or supervisory employee are considered rank-and-file employees”. It is also well-settled that the actual functions of an employee, not merely his job title, are determinative in classifying such employee as managerial, supervisory or rank and file. Good faith is presumed in all representations, an essential element of fraud, false statement and misrepresentation in order for these to be actionable is intent to mislead by the party making the representation. In this case, there is no proof to show that Bathan, or appellee union for that matter, intended to mislead

anyone. If this was appellee union’s intention, it would have refrained from using a more precise description of the organization instead of declaring that the organization is composed of ‘rank and file monthlies’. Hence, the charge of fraud, false statement or misrepresentation cannot be sustained.Even if they are supervisory employees, no action can be done that emasculates the right to self-organization and the promotion of free trade unionism. We take administrative notice of the realities in union organizing, during which the organizers must take their chances, oftentimes unaware of the fine distinctions between managerial, supervisory and rank and file employees. The grounds for cancellation of union registration are not meant to be applied automatically, but indeed with utmost discretion. Where a remedy short of cancellation is available, that remedy should be preferred. In this case, no party will be prejudiced if Bathan were to be excluded from membership in the union. The vacancy he will thus create can then be easily filled up through the succession provision of appellee union’s constitution and by-laws. What is important is that there is an unmistakeable intent of the members of appellee union to exercise their right to organize. We cannot impose rigorous restraints on such right if we are to give meaning to the protection to labor and social justice clauses of the Constitution.

3. No, it does not affect the case.The allegation that the bargaining unit that respondent sought to represent is no longer the same because of the dynamic nature of petitioner’s business, a lot of changes having occurred in the work environment, and that four of respondent’s officers are no longer connected with petitioner have no effect on the Court’s ruling that a certification election should be immediately conducted with respondent as one of the available choices. Petitioner’s bare manifestations adduce no reason why the certification election should not be conducted forthwith. If there are matters that have arisen since the filing of the petition that serve to delay or cancel the election, these can be threshed out during the pre-election conferences. Neither is the fact that some of respondent’s officers have since resigned from petitioner of any moment. The local/chapter retains a separate legal personality from that of its officers or members that remains viable notwithstanding any turnover in its officers or members.

DISPOSITIONWHEREFORE, the Petition is DENIED. Costs against petitioner.SO ORDERED.

UMALI V LOVINA86 Phil 313

PADILLA; April 29, 1950

NATUREPetition for writ of mandamus

FACTS- 201 employees of the Jai Alai Corporation of the Philippines organized themselves into an association as the "Independent Employees Union". They filed with the Department of Labor a copy of its constitution and by-laws. - Its President wrote a letter to the Secretary of Labor requesting immediate and favorable action on the pending application for registration of their labor organization. On the same date, the Secretary of Labor wrote a letter to the President of the Jai Alaistas Union of Employees, a registered and licensed union, granting it an extension of time to hold an election under the supervision of the Department of Labor, and warning the President of the union that should the election be not held within the time granted, he would be compelled to register and issue a permit to the Independent Employees Union. -Despite the failure of the Jai Alaistas Union of Employees to hold a general election and the promise of the Secretary of Labor that should such an election be not held, he would register the Independent Employees Union the following day, the said secretary

Page 42: 33994637 Labor Rounds 1and2

Labor Law 2 A2010 - 42 - Disinihas refused and still refuses to register the application and to issue a permit to the Independent Employees Union to operate as a legitimate labor organization.- respondent justifies his failure to register and to issue permit applied for upon the following grounds: (1) that he has not investigated the activities, real aims, and purposes of the Independent Employees Union, as required of him by section 3 of Commonwealth Act 213, due to the failure of the petitioner's union or its officials to accomplish and submit to him the mimeographed questionnaire required to be filled out by the labor union applicant; (2) that there is no fixed period provided for in Commonwealth Act 213, within which he as Secretary of Labor must complete his investigation and act upon the application; and (3) that he is clothed the discretion to issue or not the permit applied for.

ISSUE/S1. WON the Secretary’s refusal is justified

HELD1. No.Disposition There being no lawful reason for the respondent to refuse the registration of the application for the petitioner's union and permission to operate as a legitimate labor organization; it being the duty of the respondent to register the application and issue the permit upon payment of the required fee, as provided for in section 3 of Commonwealth Act No. 213, the investigation to be conducted by him, as required by law, having been conducted and completed, as may be inferred from his official statements in connection therewith, the conclusion in connection therewith, the conclusion is inescapable that he has neglected the performance of an act which the law specifically enjoins him to perform as a duty resulting from his office, and that such neglect unlawfully excludes the petitioner's union from the use and enjoyment of a right to which it is entitled. It appearing further that there is no other plain, speedy, and adequate remedy in the ordinary course of law, the writ prayed for should be, as is hereby, granted, without costs.

VASSAR INDUSTRIES EMPLOYEES UNION V ESTRELLA

82 SCRA 280FERNANDO; March 31, 1978

NATURE Petition for certiorari

FACTS- there was a CBA between ALU and Vassar Industries which expired. Before it expired, 111 of the 150 employees of the firm disaffiliated from the labor org and formed their own union. They filed an application for registration of their union. - respondent Estrella, then Acting Director of Bureau of Labor Relations, refused to register petitioner Vassar Industries Employees Union “on the ground that there is already a registered collective bargaining agent in the company.”

ISSUES1. What is the appropriate remedy for the applicant union?

HELD1. The union should be registered.Reasoning As long as the applicant union complies with all the legal requirements for registration, it becomes the BLR’s ministerial duty to register the union. Therefore the petitioner Union should be registered in this case. Then it can be part of a certification election to decide who would be the exclusive bargaining representative of the workers. (Philippine Labor Alliance Council v BLR: Once the fact of disaffiliation has been demonstrated beyond doubt, a certification election is the most expeditious way of determining which labor organization is to be the exclusive bargaining representative.)

Until the appropriate bargaining representative is chosen and a new CBA is concluded, the interim CBA which was entered into earlier and has favorable terms for the workers of Vassar Industries shall continue in full force and effect. This is to assure that both social justice and the protection to labor provisions would be effectively implemented without sanctioning an attempt to frustrate the exercise of this court’s

jurisdiction in a pending case.Disposition writ of certiorari is granted. Bureau of Labor Relations is ordered to conduct a certification election with petitioner labor union, Vassar Industries Labor Union, and private respondent labor union ALU, participating therein to determine the exclusive bargaining representative of the workers employed in Vassar Industries. Immediately executory.

PROGRESSIVE DEVELOPMENT CORP. V LAGUESMA (Nagkakaisang-Lakas ng

Manggagawa)271 SCRA 593

KAPUNAN; April 18, 1997

NATURESpecial civil action of certiorari

FACTS- July 9, 1993: Nagkakaisang Lakas ng Manggagawa (NLM)-Katipunan (respondent Union) filed a petition for certification election with the Department of Labor in behalf of the rank and file employees of the Progressive Development Corporation (Pizza Hut).- August 20, 1993: Progressive (petitioner) filed a verified Motion to Dismiss the petition alleging fraud, falsification and misrepresentation in the respondent Union's registration making it void and invalid. Among the bases of fraud was the fact that while the application for registration of the charter was supposed to have been approved in the organizational meeting held on June 27, 1993, the charter certification issued by the federation KATIPUNAN was dated June 26, 1993 or one (1) day prior to the formation of the chapter, and that the Constitution and by Laws submitted in support of its petition were not properly acknowledged and notarized.- August 30, 1993: Progressive filed a Petition seeking the cancellation of the Union's registration on the grounds of fraud and falsificatio. Motion was likewise filed by petitioner with the Med-Arbiter requesting suspension of proceedings in the certification election case until after the prejudicial question of the Union's legal personality is determined in the proceedings for cancellation of registration.- In an Order dated September 29, 1993, Med-Arbiter Rasidali C. Abdullah directed the holding of a certification election among petitioner's rank and file employees.- On appeal to the office of the Secretary of Labor, Labor Undersecretary Bienvenido E. Laguesma in a Resolution dated December 29, 1993 denied the same.

ISSUE/S1. WON after the necessary papers and documents have been filed by a labor organization, recognition by the Bureau of Labor Relations merely becomes a ministerial function. (Action and Denial of Application and Remedy)

HELD1. NORatio Registration requirements specifically afford a measure of protection to unsuspecting employees who may be lured into joining unscrupulous or fly-by-night unions whose sole purpose is to control union funds or use the labor organization for illegitimate ends. Such requirements are a valid exercise of the police power, because the activities in which labor organizations, associations and unions of workers are engaged directly affect the public interest and should be protected. ReasoningArt. 234. Requirements of registration. - Any applicant labor

organization, association or group of unions or workers shall acquire legal personality and shall be entitled to the rights and privileges granted by law to legitimate labor organizations upon issuance of the certificate of registration based on the following requirements xxx. (financial reports, names of officers, names of members, P50.00 filing fee)

- The Med-Arbiter summarily disregarded the petitioner's prayer that the former look into the legitimacy of the respondent Union by a

Page 43: 33994637 Labor Rounds 1and2

Labor Law 2 A2010 - 43 - Disinisweeping declaration that the union was in the possession of a charter certificate so that "for all intents and purposes, Sumasaklaw sa Manggagawa sa Pizza Hut (was) a legitimate labor organization."- The requirements embodied in Art 234 are intended as preventive measures against the commission of fraud. After a labor organization has filed the necessary papers and documents for registration, it becomes mandatory for the Bureau of Labor Relations to check if the requirements under Article 234 have been sedulously complied with.- Any action taken by the Bureau of Labor Relations is made in consonance with the mandate of the Labor Code, which, it bears emphasis, specifically requires that the basis for the issuance of a certificate of registration should be compliance with the requirements for recognition under Article 234.- The Court's conclusion should not be misconstrued as impairing the local union's right to be certified as the employees' bargaining agent in the petitioner's establishment. The Court is merely saying that the local union must first comply with the statutory requirements in order to exercise this right.

REMEDY: It would have been more prudent for the Med-Arbiter and public respondent to have granted petitioner's request for the suspension of proceedings in the certification election case, until the issue of the legality of the Union's registration shall have been resolved. Failure of the Med-Arbiter and public respondent to heed the request constituted a grave abuse of discretion.- The grounds ventilated in cancellation proceedings in accordance with Article 239 of the Labor Code constitute a grave challenge to the right of respondent Union to ask for certification election. The Med-Arbiter should have looked into the merits of the petition for cancellation before issuing an order calling for certification election

EFFECT, PENDING PETITION, CANCELLATION TRADE UNION REGISTRATION (P.24)- Once a labor organization attains the status of a legitimate labor organization it begins to possess all of the rights and privileges granted by law to such organizations.- Registration based on false and fraudulent statements and documents confer no legitimacy upon a labor organization irregularly recognized. Under such circumstances, the labor organization, not being a legitimate labor organization, acquires no rights, particularly the right to ask for certification election in a bargaining unit. The invalidity of respondent Union's registration would negate its legal personality to participate in certification election.

Disposition Petition is granted and remanded to Med-Arbiter.

3. Collateral Attack

TAGAYTAY HIGHLANDS V TAGAYTAY GOLF CLUB EMPLOYEES UNION

395 SCRA 699CARPIO-MORALES; January 22, 2003

NATUREPetition for Certiorari under Rule 45 THIGCI assailing CA decision denying its petition to annul the Department of Labor and Employment (DOLE) Resolutions of November 12, 1998 and December 29, 1998

FACTS- October 16, 1997 > Tagaytay Highlands Employees Union (THEU), Philippine Transport and General Workers Organization (PTGWO), Local Chapter No. 776, a legitimate labor organization said to represent majority of the rank-and-file employees of THIGCI, filed a petition for certification election before the DOLE Mediation-Arbitration Unit, Regional Branch No. IV- November 27, 1997 > opposed petition for certification election because the list of union members submitted by it was defective and fatally flawed as it included the names and signatures of supervisors, resigned, terminated and absent without leave (AWOL) employees, as well as employees of The Country Club, Inc., a corporation distinct and separate

from THIGCI; and that out of the 192 signatories to the petition, only 71 were actual rank-and-file employees of THIGCI. Also, some of the signatures in the list of union members were secured through fraudulent and deceitful means, and submitted copies of the handwritten denial and withdrawal of some of its employees from participating in the petition.- THEU asserted that it complied with all the requirements for valid affiliation and inclusion in the roster of legitimate labor organizations pursuant to DOLE Department Order No. 9, series of 1997, on account of which it was duly granted a Certification of Affiliation by DOLE on October 10, 1997; and that Section 5, Rule V of said Department Order provides that the legitimacy of its registration cannot be subject to collateral attack, and for as long as there is no final order of cancellation, it continues to enjoy the rights accorded to a legitimate organization. Therefore, the Med-Arbiter should, pursuant to Article 257 of the Labor Code and Section 11, Rule XI of DOLE Department Order No. 09, automatically order the conduct of a certification election. - January 28, 1998 > DOLE Med-Arbiter Anastacio Bactin ordered the holding of a certification election - DOLE Resolution of November 12, 19981 > setting aside the June 4, 1998 Resolution dismissing the petition for certification election. MFR denied - CA - denied THIGCI’s Petition for Certiorari and affirmed the DOLE Resolution dated November 12, 1998. It held that while a petition for certification election is an exception to the innocent bystander rule, hence, the employer may pray for the dismissal of such petition on the basis of lack of mutuality of interests of the members of the union as well as lack of employer-employee relationship and petitioner failed to adduce substantial evidence to support its allegations.

Issue: WON the composition of the union can be subject to collateral attack

Held:NO- After a certificate of registration is issued to a union, its legal personality cannot be subject to collateral attack. It may be questioned only in an independent petition for cancellation in accordance with Section 5 of Rule V, Book IV of the "Rules to Implement the Labor Code" (Implementing Rules) which section reads:

Sec. 5. Effect of registration. The labor organization or workers’ association shall be deemed registered and vested with legal personality on the date of issuance of its certificate of registration. Such legal personality cannot thereafter be subject to collateral attack, but may be questioned only in an independent petition for cancellation in accordance with these Rules.

PROGRESSIVE DEVELOPMENT CORP. V LAGUESMA (Nagkakaisang-Lakas ng

Manggagawa)271 SCRA 593

KAPUNAN; April 18, 1997

NATURESpecial civil action of certiorari

FACTS- July 9, 1993: Nagkakaisang Lakas ng Manggagawa (NLM)-Katipunan (respondent Union) filed a petition for certification election with the Department of Labor in behalf of the rank and file employees of the Progressive Development Corporation (Pizza Hut).- August 20, 1993: Progressive (petitioner) filed a verified Motion to Dismiss the petition alleging fraud, falsification and misrepresentation in the respondent Union's registration making it void and invalid. Among the bases of fraud was the fact that while the application for registration of the charter was supposed to have been approved in the organizational meeting held on June 27, 1993, the charter certification issued by the federation KATIPUNAN was dated June 26, 1993 or one (1) day prior to the formation of the chapter, and that

Page 44: 33994637 Labor Rounds 1and2

Labor Law 2 A2010 - 44 - Disinithe Constitution and by Laws submitted in support of its petition were not properly acknowledged and notarized.- August 30, 1993: Progressive filed a Petition seeking the cancellation of the Union's registration on the grounds of fraud and falsificatio. Motion was likewise filed by petitioner with the Med-Arbiter requesting suspension of proceedings in the certification election case until after the prejudicial question of the Union's legal personality is determined in the proceedings for cancellation of registration.- In an Order dated September 29, 1993, Med-Arbiter Rasidali C. Abdullah directed the holding of a certification election among petitioner's rank and file employees.- On appeal to the office of the Secretary of Labor, Labor Undersecretary Bienvenido E. Laguesma in a Resolution dated December 29, 1993 denied the same.

ISSUE/S1. WON after the necessary papers and documents have been filed by a labor organization, recognition by the Bureau of Labor Relations merely becomes a ministerial function. (Action and Denial of Application and Remedy)

HELD1. NORatio Registration requirements specifically afford a measure of protection to unsuspecting employees who may be lured into joining unscrupulous or fly-by-night unions whose sole purpose is to control union funds or use the labor organization for illegitimate ends. Such requirements are a valid exercise of the police power, because the activities in which labor organizations, associations and unions of workers are engaged directly affect the public interest and should be protected. ReasoningArt. 234. Requirements of registration. - Any applicant labor organization,

association or group of unions or workers shall acquire legal personality and shall be entitled to the rights and privileges granted by law to legitimate labor organizations upon issuance of the certificate of registration based on the following requirements xxx. (financial reports, names of officers, names of members, P50.00 filing fee)

- The Med-Arbiter summarily disregarded the petitioner's prayer that the former look into the legitimacy of the respondent Union by a sweeping declaration that the union was in the possession of a charter certificate so that "for all intents and purposes, Sumasaklaw sa Manggagawa sa Pizza Hut (was) a legitimate labor organization."- The requirements embodied in Art 234 are intended as preventive measures against the commission of fraud. After a labor organization has filed the necessary papers and documents for registration, it becomes mandatory for the Bureau of Labor Relations to check if the requirements under Article 234 have been sedulously complied with.- Any action taken by the Bureau of Labor Relations is made in consonance with the mandate of the Labor Code, which, it bears emphasis, specifically requires that the basis for the issuance of a certificate of registration should be compliance with the requirements for recognition under Article 234.- The Court's conclusion should not be misconstrued as impairing the local union's right to be certified as the employees' bargaining agent in the petitioner's establishment. The Court is merely saying that the local union must first comply with the statutory requirements in order to exercise this right.

REMEDY: It would have been more prudent for the Med-Arbiter and public respondent to have granted petitioner's request for the suspension of proceedings in the certification election case, until the issue of the legality of the Union's registration shall have been resolved. Failure of the Med-Arbiter and public respondent to heed the request constituted a grave abuse of discretion.- The grounds ventilated in cancellation proceedings in accordance with Article 239 of the Labor Code constitute a grave challenge to the right of respondent Union to ask for certification election. The Med-Arbiter should have looked into the merits of the petition for cancellation before issuing an order calling for certification election

EFFECT, PENDING PETITION, CANCELLATION TRADE UNION REGISTRATION (P.24)

- Once a labor organization attains the status of a legitimate labor organization it begins to possess all of the rights and privileges granted by law to such organizations.- Registration based on false and fraudulent statements and documents confer no legitimacy upon a labor organization irregularly recognized. Under such circumstances, the labor organization, not being a legitimate labor organization, acquires no rights, particularly the right to ask for certification election in a bargaining unit. The invalidity of respondent Union's registration would negate its legal personality to participate in certification election.

Disposition Petition is granted and remanded to Med-Arbiter.

4. Rights of Legitimate Labor Organization

ACEDERA V INTL. CONTAINER TERMINAL SERVICES

395 SCRA 103CARPIO-MORALES; January 13, 2003

NATUREPetition for certiorari

FACTS- Petitioners Jerry Acedera, et al. are employees of International Container Terminal Services, Inc. (ICTSI) and are officers/members of Associated Port Checkers & Workers Union-International Container Terminal Services, Inc. Local Chapter (APCWU-ICTSI), a labor organization duly registered as a local affiliate of the Associated Port Checkers & Workers Union (APCWU).- In early 1997, ICTSI went on a retrenchment program and laid off its on-call employees. This prompted the APCWU-ICTSI to file a notice of strike. The dispute respecting the retrenchment was resolved by a compromise settlement while another issue with regard to the computation of wages was referred to the LA.- Acedera, et al. filed with the LA a Complaint-in-Intervention with Motion to Intervene. The LA denied their Complaint-in-Intervention with Motion for Intervention upon a finding that they are already well represented by APCWU. NLRC affirmed. CA dismissed their petition for certiorari.

ISSUE1. WON the CA erred in ruling that the petitioners have no legal right to intervene and that their intervention is a superfluity.

HELD1. NORatio For a member of a class to be permitted to intervene in a representative action, fraud or collusion or lack of good faith on the part of the representative must be proven. It must be based on facts borne on record. Mere assertions do not suffice. Reasoning Petitioners-appellants anchor their right to intervene on Rule 19 of the 1997 Rules of Civil Procedure, Section 1 of which reads:Section 1. Who may intervene.- A person who has legal interest in the matter in litigation, or in the success of either of the parties, or an interest against both, or is so situated to be adversely affected by a distribution or other disposition of property in the custody of the court or of an officer thereof may, with leave of court, be allowed to intervene in the action. The court shall consider whether or not the intervention will unduly delay or prejudice the adjudication of the rights of the original parties, and whether or not the intervenor’s right may be fully protected in a separate proceeding.- They, however, failed to consider, in addition to the rule on intervention, the rule on representation, thusly: Sec. 3. Representatives as parties.- Where the action is allowed to be prosecuted or defended by a representative or someone acting in a fiduciary capacity, the beneficiary shall be included in the title of the case and shall be deemed to be the real party in interest. A

Page 45: 33994637 Labor Rounds 1and2

Labor Law 2 A2010 - 45 - Disinirepresentative may be a trustee of an express trust, a guardian, an executor or administrator, or a party authorized by law or these Rules. . .- A labor union is one such party authorized to represent its members under Art 242(a) of the Labor Code which provides that a union may act as the representative of its members for the purpose of collective bargaining. This authority includes the power to represent its members for the purpose of enforcing the provisions of the CBA. That APCWU acted in a representative capacity “for and in behalf of its Union members and other employees similarly situated,” the title of the case filed by it at the LA’s Office so expressly states.- While a party acting in a representative capacity, such as a union, may be permitted to intervene in a case, ordinarily, a person whose interests are already represented will not be permitted to do the same except when there is a suggestion of fraud or collusion or that the representative will not act in good faith for the protection of all interests represented by him.- Petitioners cite the dismissal of the case first by the LA, and later by the CA. The dismissal of the case does not, however, by itself show the existence of fraud or collusion or a lack of good faith on the part of APCWU. There must be clear and convincing evidence of fraud or collusion or lack of good faith independently of the dismissal.- Petitioners likewise express their fear that APCWU would not prosecute the case diligently because of its “sweetheart relationship” with ICTSI. There is nothing on record, however, to support this alleged relationship.Disposition Petition is hereby DENIED.

PHILIPPINE DIAMOND HOTEL AND RESORT, INC. V MANILA DIAMOND HOTEL EMPLOYEES UNION

493 SCRA 195CARPIO MORALES; June 30, 2006

FACTS-Union filed a petition for certification election to be declared the exclusive bargaining representative of the Hotel’s employees. This petition was dismissed by DOLE for lack of legal requirements.-after a few months, Union sent a letter to Hotel informing it of its desire to negotiate for a collective bargaining agreement. This was rejected by the Hotel stating that the Union was not the employee’s bargaining agent as their petition for certification election was denied.-Union filed a Notice of Strike with the NCMB alleging the Hotel’ refusal to bargain and for acts of unfair labor practices. NCMB summoned both parties and held series of dialogues. Union however suddenly went on strike-Secretary of DOLE assumed jurisdiction and ordered compulsory arbitration pursuant to art. 263 (g) of LC. And Union members were directed to return to work and for Hotel to accept them back. Hotel refused to accept the employees return. The order was modified (by a different Secretary) such that reinstatement was to be done only in the payroll.-Union filed for certiorari alleging grave abuse of discretion. Case was referred to the CA. CA affirmed that the “payroll reinstatement” was not a grave abuse of discretion. On appeal, it modified NLRC decision ordering reinstatement with back wages of union members.

ISSUEWON the strike conducted by the Union was illegal.

HELD Yes.-as was mentioned, the first ground mentioned by the Union- the Hotel’s refusal to bargain- was not a valid ground to stage the strike.-The second ground – that petitioner prevented or intimidated some workers from joining the union before, during or after the strike – was correctly discredited by the appellate court.. Since it is the union who alleges that unfair labor practices were committed by the Hotel, the burden of proof is on the union to prove its allegations by substantial evidence. “the facts and the evidence did not establish events [sic] least a rational basis why the union would [wield] a strike based on alleged unfair labor practices it did not even bother to substantiate”.-It is doctrinal that the exercise of the right of private sector employees to strike is not absolute. Thus Section 3 of Article XIII of the Constitution, provides:SECTION 3. x x x

It shall guarantee the rights of all workers to self-organization, collective bargaining and negotiations and peaceful concerted activities, including the right to strike in accordance with law…”-Even if the purpose of a strike is valid, the strike may still be held illegal where the means employed are illegal. Thus, the employment of violence, intimidation, restraint or coercion in carrying out concerted activities which are injurious to the rights to property renders a strike illegal. Evidence show s that some of the workers-strikers who joined the strike indeed committed illegal acts – blocking the free ingress to and egress from the Hotel, holding noise barrage, threatening guests, and the like.TOPICS: right to engage in concerted activities-limitations

CORNISTA V NLRC (CARPIO).504 SCRA 659

GARCIA; October 17, 2006

NATURE Review on certiorari of the consolidated decisions of the CA

FACTS - In 1983, the Philippine Veterans Bank was placed under receivership. The liquidator terminated all the employees and commenced payment of separation pay and other benefits to the terminated employees. - In 1992, Congress authorized the reopening of the Bank. A Rehabilitation Committee was created to select and organize manpower. The Union charged the Bank with unfair labor practices and prayed for the immediate reinstatement of the Bank's former employees.-Labor Arbiter Carpio dismissed the case. NLRC reversed and ordered the immediate reinstatement of all Union members.- Bank appealed to the SC. While appeal was pending, the Bank and the Union, through its officers, entered into a compromise agreement, which was ratified by a substantial majority of the Union’s members. Carpio approved the agreement. Parties moved to dismiss cases pending in SC. As other employees opposed motion to dismiss, SC denied said motion.

ISSUES1. WON the employees have the right to be reinstated to their former employment with the Bank upon its rehabilitation2. WON the Compromise Agreement was validly entered into by the Bank and the Union

HELD 1. NO- The forcible closure of the Bank by operation of law permanently severed the employer-employee relationship between it and its employees. The Bank's subsequent rehabilitation did not, by any test of reason, "revive" what was already a dead relationship between the petitioners and the Bank.- Had Congress intended that separated employees be rehired and given priority in the hiring of new employees, it would have clearly stated this in R.A. No. 7169. The fact that it did not only shows its clear legislative intent to give the new bank a free hand in the selection and hiring of its new staff.- Giving in to petitioners' demand of wholesale reinstatement with back wages, bonuses, holiday pay, vacation and sick leave benefits would be a fatal blow to the very intention of R.A. No. 7169 to rehabilitate the Bank. The payment of such substantial amounts would definitely further dissipate the remaining assets of the Bank and cripple its finances even as, at this point, the Bank is barely making a profit under the weight of its present liabilities, and ultimately make impossible its desired rehabilitation. This clearly contravenes the intent and spirit of R.A. No. 71692. NORatio A compromise agreement, once approved, has the effect of res judicata between the parties and should not be disturbed except for vices of consent, forgery, fraud, misrepresentation and coercion, none of which exists in this case.Reasoning

Page 46: 33994637 Labor Rounds 1and2

Labor Law 2 A2010 - 46 - Disini- A labor union's function is to represent its members. It can file an action or enter into compromise agreements on behalf of its members- Union is a closed shop union. For this reason, it was the only one with legal authority to negotiate, transact, and enter into any agreement with the Bank. The Compromise Agreement was ratified by 282 Union members representing a majority of its entire 529 membership. The ratification of the Compromise Agreement by the majority of the Union members necessarily binds the minority.- The submission of the Compromise Agreement on joint motion of the parties for approval by the Labor Arbiter cured whatever defect the signing of the agreement in the absence of the Labor Arbiter would have caused.- Furthermore, petitioners cannot belatedly reject or repudiate their acts of accepting the monetary consideration under the compromise agreement, to the prejudice of the Bank. They are now estopped from questioning the validity of the Compromise Agreement.Dispositive Petition DENIED.

5. Effect of Non-registration

PROTECTION TECHNOLOGY INC. V. SECRETARY, DEPARTMENT OF LABOR AND EMPLOYMENT

242 SCRA 99FELICIANO; March 1, 1995

NATURESpecial civil action in the Supreme Court. Certiorari.

FACTS- Union, which is newly organized and affiliated w/ a federation, filed Petition for direct certification or for certification election. Company said that the Union is not legitimate bec it failed to submit books of account w/ BLR at time it was registered.- Med Arbiter dismissed Union’s petition. DOLE Undersecretary set aside Med Arbiter’s order, ordering the holding of a certification election. Petition is now before SC.- SC issued TRO, but this notwithstanding, certification election was conducted.

ISSUE/SWON DOLE Undersecretary committed grave abuse of discretion

HELD1. YESRatio The requirements of registration of legitimate labor organizations are an exercise of the police power of the State, designed for the protection of workers against potential abuse by unions.Registration is a condition sine qua non for acquisition of legal personality by a labor organization and the exercise of rights and privileges granted by law.Reasoning It is immaterial that the Union was organized for less than a year before its application for registration w/ BLR. Accounting books must be submitted even if they contain no detailed entries.The Union must submit its books of account before it may demand recognition by the Company as exclusive bargaining agent of members.Disposition Petition is dismissed for being moot and academic.

SUGBUANON RURAL BANK, INC V LAGUESMA324 SCRA 425

QUISUMBING: February 2, 2000

NATURE:- Special civil action for certiorari and prohibition

FACTS:- Petitioner Sugbuanon Rural Bank, Inc., (SRBI, for brevity) is a

duly-registered banking institution with principal office in Cebu

City and a branch in Mandaue City. Private respondent SRBI Association of Professional, Supervisory, Office, and Technical Employees Union (APSOTEU) is a legitimate labor organization affiliated with the Trade Unions Congress of the Philippines (TUCP).

- On October 8, 1993, the DOLE Regional Office in Cebu City granted Certificate of Registration to APSOTEU-TUCP, hereafter referred to as the union.

- On October 26, 1993, the union filed a petition for certification election of the supervisory employees of SRBI. It alleged, among others, that: (1) APSOTEU-TUCP was a labor organization duly-registered with the Labor Department; (2) SRBI employed 5 or more supervisory employees; (3) a majority of these employees supported the petition: (4) there was no existing collective bargaining agreement (CBA) between any union and SRBI; and (5) no certification election had been held in SRBI during the past 12 months prior to the petition.

- On November 12, 1993, SRBI filed a motion to dismiss the union's petition. It sought to prevent the holding of a certification election on two grounds. First, that the members of APSOTEU-TUCP were in fact managerial or confidential employees. Second, the Association of Labor Unions-Trade Unions Congress of the Philippines or ALU-TUCP was representing the union.

- The union filed its opposition to the motion to dismiss on December 1, 1993. It argued that its members were not managerial employees but merely supervisory employees.

- On December 9, 1993, the Med-Arbiter denied petitioner's motion to dismiss.

- SRBI appealed the Med-Arbiter's decision to the Secretary of Labor and Employment. The appeal was denied for lack of merit. The certification election was ordered.

- On June 16, 1994, the Med-Arbiter scheduled the holding of the certification election for June 29, 1994.

- On June 17, 1994, SRBI filed with the Med-Arbiter an urgent motion to suspend proceedings. The Med-Arbiter denied the same. SRBI then filed a motion for reconsideration. Two days later, the Med-Arbiter cancelled the certification election scheduled for June 29, 1994 in order to address the motion for reconsideration.

- The Med-Arbiter later denied petitioner's motion for reconsideration

- SRBI appealed the order of denial to the DOLE Secretary- Petitioner proceeded to file a petition with the DOLE

Regional Office seeking the cancellation of the respondent union's registration. It averred that the APSOTEU-TUCP members were actually managerial employees who were prohibited by law from joining or organizing unions.

- DOLE Undersecretary denied SRBI's appeal for lack of merit. He ruled that APSOTEU-TUCP was a legitimate labor organization. It was fully entitled to all the rights and privileges granted by law to a legitimate labor organization, including the right to file a petition for certification election. He also held that until and unless a final order is issued cancelling APSOTEU-TUCP's registration certificate, it had the legal right to represent its members for collective bargaining purposes.

- SRBI moved for reconsideration of the Undersecretary's decision

ISSUE/S:

(1) WON the members of the respondent union are managerial employees and/or highly-placed confidential employees, hence prohibited by law from joining labor organizations and engaging in union activities(2) WON the Med-Arbiter may validly order the holding of a certification election

HELD:2. NO

Reasoning

Page 47: 33994637 Labor Rounds 1and2

Labor Law 2 A2010 - 47 - Disini- Article 212 (m) of the Labor Code defines the terms "managerial employee" and "supervisory employees" as follows:

Art. 212. Definitions(m) "Managerial employee" is one who is vested with powers or prerogatives to lay down and execute management policies and/or hire, transfer, suspend, lay-off, recall, discharge, assign or discipline employees. Supervisory employees are those who, in the interest of the employer, effectively recommend such managerial actions if the exercise of such authority is not merely routinary or clerical in nature but requires the use of independent judgment. All employees not falling within any of the above definitions are considered rank-and-file employees for purposes of

- Petitioner submitted detailed job descriptions to support its contention that the union members are managerial employees and/or confidential employees proscribed from engaging in labor activities. In the present case, however, petitioner failed to show that the employees in question were vested with managerial powers. At best they only had recommendatory powers subject to evaluation, review, and final decision by the bank's management. The job description forms submitted by petitioner clearly show that the union members in question may not transfer, suspend, lay-off, recall, discharge, assign, or discipline employees. Moreover, the forms also do not show that the Cashiers, Accountants, and Acting Chiefs of the Loans Department formulate and execute management policies which are normally expected of management officers.2. YES Reasoning

- One of the rights of a legitimate labor organization under Article 242(b) of the Labor Code is the right to be certified as the exclusive representative of all employees in an appropriate bargaining unit for purposes of collective bargaining. Having complied with the requirements of Art. 234, it is our view that respondent union is a legitimate labor union. Article 257 of the Labor Code mandates that a certification election shall automatically be conducted by the Med-Arbiter upon the filing of a petition by a legitimate labor organization.

DISPOSITIVE:Petition dismissed.

6. Cancellation of Union Certificate Registration

IN RE: PETITION FOR CANCELLATION OF THE UNION REGISTRATION OF AIR PHILIPPINES FLIGHT

ATTENDANTS ASSOCIATIONAnd

AIR PHILIPPINES CORPORATION v. BLR [AIR PHILS FLIGHT ATTENDANTS ASSOCIATION]

TINGA; June 22, 2006

Nature: Certiorari under Rule 45

FACTS:

- The case initially centered on the union registration of respondent Air Philippines Flight Attendants Association (APFLAA), which was issued a Certificate of Registration by the DOLE. After the Med-Arbiter rendered a ruling ordering the holding of a certification election, such election was held, with majority of the votes cast in favor of APFLAA.- APC filed a Petition for De-Certification and Cancellation of Union Registration against APFLAA with the DOLE alleging that APFLAA could not be registered as a labor organization, as its composition consisted of “a mixture of supervisory and rank-and-file flight attendants.” Particularly, APC alleged that flight attendants holding the position of “Lead Cabin Attendant,” which according to it is supervisory in character, were among those who comprised APFLAA. - The DOLE-NCR dismissed said petition and held that A245 of the LC, which states that supervisory employees are not eligible for membership in labor organizations of rank-and-file employees, does not provide a

ground for cancellation of union registration, which is instead governed by Article 239 of the Labor Code. - BLR denied the appeal, affirming the rationale of the DOLE-NCR. - CA also dismissed because APC had “failed to avail of the remedy of a prior MFR” before the filing of the certiorari petition, which step, it stressed, is a “condition sine qua non to the filing of a petition for certiorari.” Denied the MR as well for being defective.

ISSUE: WON the Union’ registration may be cancelled as it is composed of a mixture of supervisory and rank-and-file employees(and also, WON Lead Cabin Attendants are indeed supervisory employees)

HELD: NO. APC did not impute on APFLAA such misrepresentation of the character necessitated under A239 (a) and (c) of the Labor Code.

Reasoning: - APC merely argued that APFLAA was not qualified to become a legitimate labor organization by reason of its mixed composition of rank-and-file and supervisory employees; and that APFLAA committed misrepresentation by making it appear that its composition was composed purely of rank-and-file employees.

- SPI Technologies Incorporated v. DOLE: the Court observed that A245 of the LC, the legal basis for the petition for cancellation, merely prescribed the requirements for eligibility in joining a union and did not prescribe the grounds for cancellation of union registration.[- Tagaytay Highlands International Golf Club v. Tagaytay Highlands Employees Union: “[t]he inclusion in a union of disqualified employees is not among the grounds for cancellation, unless such inclusion is due to misrepresentation, false statement or fraud under the circumstances enumerated in Sec (a) and (c) of A239 LC.”

- Clearly then, for the purpose of de-certifying a union, it is not enough to establish that the rank-and-file union includes ineligible employees in its membership. Pursuant to Article 239 (a) and (c) of the Labor Code, it must be shown that there was misrepresentation, false statement or fraud in connection with the adoption or ratification of the constitution and by-laws or amendments thereto, the minutes of ratification, or in connection with the election of officers, minutes of the election of officers, the list of voters, or failure to submit these documents together with the list of the newly elected-appointed officers and their postal addresses to the BLR.

-Consistent with jurisprudence, the rule under A245 barring supervisory employees from joining the union of rank-and-file employees is not a ground for cancellation of union registration.

Dispositive: Petition is DENIED.

LAGUNA AUTO PARTS MANUFACTURING CORP. V SECRETARY, DOLE

457 SCRA 730CALLEJO; April 29, 2005

NATUREPetition to review the decision of the Court of Appeals FACTS- May 3, 1999 - Laguna Autoparts Manufacturing Corporation Obrero Pilipino-LAMCOR Chapter filed a petition for certification election with the DOLE.  In its petition, it noted its registration certificate number along with the registration certificate number of chapter affiliate.  The petition alleged that the union was composed of all rank-and-file employees, that the bargaining unit is unorganized and that there had been no certification election for the past 12 months prior to the filing of the petition.

Page 48: 33994637 Labor Rounds 1and2

Labor Law 2 A2010 - 48 - Disini- Laguna Autoparts moved to cancel the certification election because the union was not considered a legitimate labor organization for failure to show that it had complied with registration requirements such as submission of required documents to the Bureau of Labor Relations.- Med-Arbiter Bactin dismissed the certification election petition because of the union’s lack of personality.  DOLE Secretary Sto. Tomas reversed Bactin’s order.  This was affirmed by the CA.

- The CA stressed that a local or chapter need not be registered to become a legitimate labor organization. It pointed out that a local or chapter acquires legal personality as a labor organization from the date of filing of the complete documents.  - The CA noted that it was the employer which offered the most tenacious resistance to the holding of a certification election among its regular rank-and-file employees. It opined that this must not be so for the choice of a collective bargaining agent was the sole concern of the employees, and the employer should be a mere bystander.

 ISSUE/S

1.        WON the union is a legitimate labor organization2.        WON the chapter’s legal personality can be attacked collaterally in an election action

 HELD1. YESRatio A local or chapter need not be independently registered to acquire legal personality.  A local/chapter constituted in accordance with Section 1 of Rule 6 shall acquire legal personality from the date of filing of the complete documents enumerated.  ReasoningThe task of determining whether the local or chapter has submitted the complete documentary requirements is lodged with the Regional Office or the BLR, as the case may be. The records of the case show that the respondent union submitted the said documents to the regional office and was issued a certification.   2. NORatioReasoning- It may not be subject to a collateral attack but only through a separate action instituted particularly for the purpose of assailing the chapter, as prescribed in Section 5, Rule 5 of the implementing rules of Book 5 which states: “The labor organization or workers’ association shall be deemed registered and vested with legal personality on the date of issuance of its certificate of registration. Such legal personality cannot thereafter be subject to collateral attack but may be questioned only in an independent petition for cancellation in accordance with these Rules.[”- The pronouncement of the Labor Relations Division Chief, that the respondent union acquired a legal personality with the submission of the complete documentary requirement, cannot be challenged in a petition for certification election.Disposition Petition is denied. No costs.

TABLANTE-TUNGOL V NORIEL FERNANDO; AUG 23 1978

NATUREPetition for certiorari

FACTSPetitioner, after two unsuccessful attempts to prevent collective bargaining with respondent union, filed this certiorari proceeding alleging that public respondents should have cancelled the registration and permit of respondent labor organization as it had engaged in an illegal strike. It based its contention on P.D. No. 823 and Article 239 of the New Labor Code that cancellation of registration follows from "any activity prohibited by law."

The Solicitor General, in his Comment claimed that the argument proferred was false and misleading as the law should not be interpreted to

include an illegal strike engaged into by any union. The phrase 'or otherwise engaging in any activity prohibited by law' should be construed to mean such activity engaged into by a union that par takes of the nature of a labor contractor or 'cabo' system and respondent union is not engaged in any such activity. Subsequently, petitioner and private respondent filed a joint motion to dismiss alleging that they have threshed-out their respective disputes.

ISSUEWON public respondents should have cancelled the registration and permit of private respondent labor organization as private respondent labor union had engaged in an illegal strike

HELDNO. For expediency, we quote in entirety the aforesaid Article relied upon by the Petitioner for cancellation of the registration and permit of the union: 'Article 239. Ground for cancellation of union registration. The following shall constitute grounds for cancellation of union registration: . . . (e) Acting as a labor contractor or engaging in the "cabo" system, or otherwise engaging in any activity prohibited by law. Suppletory to the above provision is Section 6 (c) of Rule II, Book V of the Rules and Regulations implementing the Labor Code of the Philippines, as amended, which reads as follows: 'Section 6. Denial of Registration of local unions - The Regional Office may deny the application for registration on any of the following grounds: . . . (c) Engaging in the "cabo" system or other illegal practices.'

It is a fact that [Association of Democratic Labor Organization] is not a labor contractor or is it engaged in the 'cabo' system or is it otherwise engaged in any activity of such nature which is prohibited by law. The above-quoted article should not be interpreted or construed to include an illegal strike engaged into by any union. This is so because the phrase 'or otherwise engaging in any activity prohibited by law' should be construed to mean such activity engaged into by a union that par takes of the nature of a labor contractor or 'cabo' system. The law does not intend to include in the said phrase illegally declared strike simply because strike per se is legal. Also, if the law intends to include illegally declared strike, the same could have been expressly placed therein as had been previously done in Presidential Decree No. 823." Clearly, an awareness of the relevance of the maxims noscitur a sociis and ejusdem generis ought to have cautioned counsel for petitioner to shy away from this approach.

Dispositive Petition dismissed

ALLIANCE OF DEMOCRATIC LABOR ORGANIZATION V. LAGUESMA

254 SCRA 565PANGANIBAN; March 11, 1996

NATUREPetition for certiorari and prohibition under Rule 65

FACTS -The Alliance of Democratic Free Labor Organization (ADFLO) filed an application for registration as a national federation alleging, among others that it has twelve (12) affiliates-After proper evaluation of its application and finding ADFLO to have complied with the requirements for registration pursuant to Articles 234 and 237 of the Labor Code, the Bureau (of Labor Relations) issued a Certificate of Registration No. 11399-FED-LC to the federation.-The Confederation of labor and Allied Social Services (CLASS) filed a petition for the cancellation of the Registration Certificate issued to ADFLO.-The Bureau of Labor Relations (BLR), through Director Pura Ferrer-Calleja, rendered a Decision cancelling the registration of ADFLO. -BLR Director Pura Ferrer-Calleja, without first ruling on the admissibility of the exhibits of CLASS and without any further hearing, rendered an order affirming the cancellation

Page 49: 33994637 Labor Rounds 1and2

Labor Law 2 A2010 - 49 - DisiniISSUE1. WON decision cancelling the registration of petitioner was rendered

in violation of the due process clause

HELD1. YES. Ratio Under Section 1, Article II of our Constitution, "(n)o person shall be deprived of life, liberty or property without due process of law . . ." and under Article 238 of the Labor Code,"(t)he certificate of registration of any legitimate labor organization, whether national or local, shall be canceled by the Bureau if it has reason to believe, after due hearing, that the said labor organization no longer meets one or more of the requirements herein prescribed." Reasoning The cancellation of a certificate of registration is the equivalent of snuffing out the life of a labor organization. For without such registration, it loses - as a rule - its rights under the Labor Code. Under the circumstances, petitioner was indisputably entitled to be heard before a judgment could be rendered cancelling its certificate of registration. In David vs. Aguilizan, it was held that a decision rendered without any hearing is null and void.

Disposition Petition GRANTED

PROGRESSIVE DEVELOPMENT CORP. V LAGUESMA (Nagkakaisang-Lakas ng Manggagawa)

271 SCRA 593KAPUNAN; April 18, 1997

NATURESpecial civil action of certiorari

FACTS- July 9, 1993: Nagkakaisang Lakas ng Manggagawa (NLM)-Katipunan (respondent Union) filed a petition for certification election with the Department of Labor in behalf of the rank and file employees of the Progressive Development Corporation (Pizza Hut).- August 20, 1993: Progressive (petitioner) filed a verified Motion to Dismiss the petition alleging fraud, falsification and misrepresentation in the respondent Union's registration making it void and invalid. Among the bases of fraud was the fact that while the application for registration of the charter was supposed to have been approved in the organizational meeting held on June 27, 1993, the charter certification issued by the federation KATIPUNAN was dated June 26, 1993 or one (1) day prior to the formation of the chapter, and that the Constitution and by Laws submitted in support of its petition were not properly acknowledged and notarized.- August 30, 1993: Progressive filed a Petition seeking the cancellation of the Union's registration on the grounds of fraud and falsificatio. Motion was likewise filed by petitioner with the Med-Arbiter requesting suspension of proceedings in the certification election case until after the prejudicial question of the Union's legal personality is determined in the proceedings for cancellation of registration.- In an Order dated September 29, 1993, Med-Arbiter Rasidali C. Abdullah directed the holding of a certification election among petitioner's rank and file employees.- On appeal to the office of the Secretary of Labor, Labor Undersecretary Bienvenido E. Laguesma in a Resolution dated December 29, 1993 denied the same.

ISSUE/S1. WON after the necessary papers and documents have been filed by a labor organization, recognition by the Bureau of Labor Relations merely becomes a ministerial function. (Action and Denial of Application and Remedy)

HELD1. NORatio Registration requirements specifically afford a measure of protection to unsuspecting employees who may be lured into joining unscrupulous or fly-by-night unions whose sole purpose is to control union funds or use the labor organization for illegitimate ends. Such requirements are a valid exercise of the police power, because the

activities in which labor organizations, associations and unions of workers are engaged directly affect the public interest and should be protected. ReasoningArt. 234. Requirements of registration. - Any applicant labor

organization, association or group of unions or workers shall acquire legal personality and shall be entitled to the rights and privileges granted by law to legitimate labor organizations upon issuance of the certificate of registration based on the following requirements xxx. (financial reports, names of officers, names of members, P50.00 filing fee)

- The Med-Arbiter summarily disregarded the petitioner's prayer that the former look into the legitimacy of the respondent Union by a sweeping declaration that the union was in the possession of a charter certificate so that "for all intents and purposes, Sumasaklaw sa Manggagawa sa Pizza Hut (was) a legitimate labor organization."- The requirements embodied in Art 234 are intended as preventive measures against the commission of fraud. After a labor organization has filed the necessary papers and documents for registration, it becomes mandatory for the Bureau of Labor Relations to check if the requirements under Article 234 have been sedulously complied with.- Any action taken by the Bureau of Labor Relations is made in consonance with the mandate of the Labor Code, which, it bears emphasis, specifically requires that the basis for the issuance of a certificate of registration should be compliance with the requirements for recognition under Article 234.- The Court's conclusion should not be misconstrued as impairing the local union's right to be certified as the employees' bargaining agent in the petitioner's establishment. The Court is merely saying that the local union must first comply with the statutory requirements in order to exercise this right.

REMEDY: It would have been more prudent for the Med-Arbiter and public respondent to have granted petitioner's request for the suspension of proceedings in the certification election case, until the issue of the legality of the Union's registration shall have been resolved. Failure of the Med-Arbiter and public respondent to heed the request constituted a grave abuse of discretion.- The grounds ventilated in cancellation proceedings in accordance with Article 239 of the Labor Code constitute a grave challenge to the right of respondent Union to ask for certification election. The Med-Arbiter should have looked into the merits of the petition for cancellation before issuing an order calling for certification election

EFFECT, PENDING PETITION, CANCELLATION TRADE UNION REGISTRATION (P.24)- Once a labor organization attains the status of a legitimate labor organization it begins to possess all of the rights and privileges granted by law to such organizations.- Registration based on false and fraudulent statements and documents confer no legitimacy upon a labor organization irregularly recognized. Under such circumstances, the labor organization, not being a legitimate labor organization, acquires no rights, particularly the right to ask for certification election in a bargaining unit. The invalidity of respondent Union's registration would negate its legal personality to participate in certification election.

Disposition Petition is granted and remanded to Med-Arbiter.

TAGAYTAY HIGHLANDS V TAGAYTAY GOLF CLUB EMPLOYEES UNION

395 SCRA 699CARPIO-MORALES; January 22, 2003

NATUREPetition for Certiorari under Rule 45 THIGCI assailing CA decision denying its petition to annul the Department of Labor and Employment (DOLE) Resolutions of November 12, 1998 and December 29, 1998

Page 50: 33994637 Labor Rounds 1and2

Labor Law 2 A2010 - 50 - DisiniFACTS- October 16, 1997 > Tagaytay Highlands Employees Union (THEU), Philippine Transport and General Workers Organization (PTGWO), Local Chapter No. 776, a legitimate labor organization said to represent majority of the rank-and-file employees of THIGCI, filed a petition for certification election before the DOLE Mediation-Arbitration Unit, Regional Branch No. IV- November 27, 1997 > opposed petition for certification election because the list of union members submitted by it was defective and fatally flawed as it included the names and signatures of supervisors, resigned, terminated and absent without leave (AWOL) employees, as well as employees of The Country Club, Inc., a corporation distinct and separate from THIGCI; and that out of the 192 signatories to the petition, only 71 were actual rank-and-file employees of THIGCI. Also, some of the signatures in the list of union members were secured through fraudulent and deceitful means, and submitted copies of the handwritten denial and withdrawal of some of its employees from participating in the petition.- THEU asserted that it complied with all the requirements for valid affiliation and inclusion in the roster of legitimate labor organizations pursuant to DOLE Department Order No. 9, series of 1997, on account of which it was duly granted a Certification of Affiliation by DOLE on October 10, 1997; and that Section 5, Rule V of said Department Order provides that the legitimacy of its registration cannot be subject to collateral attack, and for as long as there is no final order of cancellation, it continues to enjoy the rights accorded to a legitimate organization. Therefore, the Med-Arbiter should, pursuant to Article 257 of the Labor Code and Section 11, Rule XI of DOLE Department Order No. 09, automatically order the conduct of a certification election. - January 28, 1998 > DOLE Med-Arbiter Anastacio Bactin ordered the holding of a certification election - DOLE Resolution of November 12, 19981 > setting aside the June 4, 1998 Resolution dismissing the petition for certification election. MFR denied - CA - denied THIGCI’s Petition for Certiorari and affirmed the DOLE Resolution dated November 12, 1998. It held that while a petition for certification election is an exception to the innocent bystander rule, hence, the employer may pray for the dismissal of such petition on the basis of lack of mutuality of interests of the members of the union as well as lack of employer-employee relationship and petitioner failed to adduce substantial evidence to support its allegations.

Issue:WON the composition of the union can be subject to collateral attack

Held: . NO- After a certificate of registration is issued to a union, its legal personality cannot be subject to collateral attack. It may be questioned only in an independent petition for cancellation in accordance with Section 5 of Rule V, Book IV of the "Rules to Implement the Labor Code" (Implementing Rules) which section reads:

Sec. 5. Effect of registration. The labor organization or workers’ association shall be deemed registered and vested with legal personality on the date of issuance of its certificate of registration. Such legal personality cannot thereafter be subject to collateral attack, but may be questioned only in an independent petition for cancellation in accordance with these Rules.

INTERNATIONAL ACTIVITIES OF UNION—PROHIBITION AND REGUULATION – 269-271

UNION-MEMBER RELATIONS

1. Nature of Relationship

HEIRS OF CRUZ V. CIR (SANTIAGO RICE MILL)30 SCRA 917

TEEHANKEE; Dec. 27, 1969

NATURESpecial civil action for certiorari

FACTS-This is a case involving union officers who entered into a compromise concerning backwages of member-employees with the employer. -Petitioners are the retained lawyers of the Santiago Labor Union who question respondent Court's approval of respondent firm's settlement of the union members' judgment claims with the union board of directors, without their knowledge and consent, notwithstanding their duly recorded attorneys' lien, and over the objection of a board member that the union board had no authority to compromise or quit-claim the judgment rights of the union members. -Petitioners are forty-nine (49) claimants-members of the Santiago Labor Union who assail respondent Court's approval of the questioned settlement, without their authority as the real parties in interest, and who denounce the settlement as unconscionable and having been entered into by the majority of the union board "under circumstances of fraud, deceit, misrepresentation and/or concealment, especially where a member of the Court has actively used his official and personal influence to effect the settlement which is manifestly unjust to laborers who by reason of their financial disadvantages in a conflict with their employers need all the aid of the Court for their protection, consonant with law, justice and equity."

ISSUE2. WON the real parties in interest are the union members who

have initiated and instituted this petition as against the limited and formal personality of the respondent labor union to represent them when so authorized by their collective will

HELD2. YES. Ratio The union is but an agent of the individual workers and it has the duty to inform the members of the labor matters entrusted to it.Reasoning The employer may rely on the authority of the union to bring the union members especially in collective bargaining where the matters to be discussed are still to be observed but this case is an exception. What is sought here are backwages and other benefits already earned. Authority for the union to waive this right to backwages must be express. In a compromise or settlement, the individual union members are the real judgment creditors and are the real parties in interest.

Disposition CIR order SET ASIDE..

2. Constitution and By-Laws

SAN MIGUEL CORP. V. MANDAUETinga ; Aug. 16, 2005

467 SCRA 107

Facts-CA affirmes DOLE Undersecretary for Labor Relations, Rosalinda Dimapilis-Baldoz, ordering the immediate conduct of a certification election among the petitioner’s rank-and-file employees. - Federation of Free Workers (FFW/ respondent) filed a petition for certification election with the DOLE Regional Office No. VII. It sought to be certified and to represent the permanent rank-and-file monthly paid employees of the petitioner. The following documents were attached to the petition: (1) a Charter Certificate certifying that respondent as of that date was duly certified as a local or chapter of FFW; (2) a copy of the constitution of respondent prepared by its Secretary, Noel T. Bathan and attested by its President, Wilfred V. Sagun; (3) a list of respondent’s officers and their respective

Page 51: 33994637 Labor Rounds 1and2

Labor Law 2 A2010 - 51 - Disiniaddresses, again prepared by Bathan and attested by Sagun; (4) a certification signifying that respondent had just been organized and no amount had yet been collected from its members, signed by respondent’s treasurer Chita D. Rodriguez and attested by Sagun; and (5) a list of all the rank-and-file monthly paid employees of the Mandaue Packaging Products Plants and Mandaue Glass Plant prepared by Bathan and attested by Sagun. -SMC (Petitioner) filed a motion to dismiss the petition for certification election on the sole ground that herein respondent is not listed or included in the roster of legitimate labor organizations based on the certification issued by the Officer-In-Charge, Regional Director of the DOLE Regional Office No. VII, Atty. Jesus B. Gabor.-Respondent submitted to the Bureau of Labor Relations the same documents earlier attached to its petition for certification. The accompanying letter, signed by respondent’s president Sagun, stated that such documents were submitted in compliance with the requirements for the creation of a local/chapter pursuant to the Labor Code and its Implementing Rules; and it was hoped that the submissions would facilitate the listing of respondent under the roster of legitimate labor organizations.The Chief of Labor Relations Division of DOLE Regional Office No. VII issued a Certificate of Creation of Local/Chapter No. ITD. I-ARFBT-058/98, certifying that from 30 July 1998, respondent has acquired legal personality as a labor organization/worker’s association, it having submitted all the required documents.

Issues 1. When did the union acquire legal personality?2. Whether or not the inclusion of the two alleged supervisory employees in appellee union’s membership amounts to fraud, misrepresentation, or false statement within the meaning of Article 239(a) and (c) of the Labor Code.3. Whether or not subsequent developments change the disposition of the case

Held1. 15 June 1998, the date the complete documents were submitted.The issuance of the certificate of registration by the Bureau or Regional Office is not the operative act that vests legal personality upon a local/chapter under Department Order No. 9. Such legal personality is acquired from the filing of the complete documentary requirements enumerated in Section 1, Rule VI. Admittedly, the manner by which respondent was deemed to have acquired legal personality by the DOLE and the Court of Appeals was not in strict conformity with the provisions of Department Order No. 9. Thus, in order to ascertain when respondent acquired legal personality, we only need to determine on what date the Regional Office or Bureau received the complete documentary requirements enumerated under Section 1, Rule VI of Department Order No. 9. There is no doubt that on 15 June 1998, or the date respondent filed its petition for certification election, attached thereto were respondent’s constitution, the names and addresses of its officers, and the charter certificate issued by the national union FFW. The first two of these documents were duly certified under oath by respondent’s secretary Bathan and attested to by president Sagun.What about the lack of documents containing the by-laws? Not needed so long as it is part of the union’s constitution.An examination of respondent’s constitution reveals it sufficiently comprehensive in establishing the necessary rules for its operation. Article IV establishes the requisites for membership in the local/chapter. Articles V and VI name the various officers and what their respective functions are. The procedure for election of these officers, including the necessary vote requirements, is provided for in Article IX, while Article XV delineates the procedure for the impeachment of these officers. Article VII establishes the standing committees of the local/chapter and how their members are appointed. Article VIII lays down the rules for meetings of the union, including the notice and quorum requirements thereof. Article X enumerates with particularity the rules for union dues, special assessments, fines, and other payments. Article XII provides the general rule for quorum in meetings of the Board of Directors and of the members of the local/chapter, and cites the applicability of the Robert’s Rules of Order[43] in its meetings. And finally, Article XVI governs and institutes the requisites for the amendment of the constitution. The Court likewise sees no impediment in deeming respondent

as having acquired legal personality as of 15 June 1998, the fact that it was the local/chapter itself, and not the FFW, which submitted the documents required under Section 1, Rule VI of Department Order No. 9. The evident rationale why the rule states that it is the federation or national union that submits said documents to the Bureau or Regional Office is that the creation of the local/chapter is the sole prerogative of the federation or national union, and not of any other entity. Certainly, a putative local/chapter cannot, without the imprimatur of the federation or national union, claim affiliation with the larger unit or source its legal personality therefrom. In the ordinary course, it should have been FFW, and not respondent, which should have submitted the subject documents to the Regional Office. Nonetheless, there is no good reason to deny legal personality or defer its conferral to the local/chapter if it is evident at the onset that the federation or national union itself has already through its own means established the local/chapter. In this case, such is evidenced by the Charter Certificate dated 9 June 1998, issued by FFW, and attached to the petition for certification election. The Charter Certificate expressly states that respondent has been issued the said certificate “to operate as a local or chapter of the [FFW]”. The Charter Certificate expressly acknowledges FFW’s intent to establish respondent as of 9 June 1998. This being the case, we consider it permissible for respondent to have submitted the required documents itself to the Regional Office, and proper that respondent’s legal personality be deemed existent as of 15 June 1998, the date the complete documents were submitted.

2. No, it does not. Under the law, a managerial employee is “one who is vested with powers or prerogatives to lay down and execute management policies and/or to hire, transfer, suspend, layoff, recall, discharge, assign or discipline employees.” A supervisory employee is “one who, in the interest of the employer, effectively recommends managerial actions if the exercise of such recommendatory authority is not merely routinary or clerical in nature but requires the use of independent judgment.’” Finally, “all employees not falling within the definition of managerial or supervisory employee are considered rank-and-file employees”. It is also well-settled that the actual functions of an employee, not merely his job title, are determinative in classifying such employee as managerial, supervisory or rank and file. Good faith is presumed in all representations, an essential element of fraud, false statement and misrepresentation in order for these to be actionable is intent to mislead by the party making the representation. In this case, there is no proof to show that Bathan, or appellee union for that matter, intended to mislead anyone. If this was appellee union’s intention, it would have refrained from using a more precise description of the organization instead of declaring that the organization is composed of ‘rank and file monthlies’. Hence, the charge of fraud, false statement or misrepresentation cannot be sustained.Even if they are supervisory employees, no action can be done that emasculates the right to self-organization and the promotion of free trade unionism. We take administrative notice of the realities in union organizing, during which the organizers must take their chances, oftentimes unaware of the fine distinctions between managerial, supervisory and rank and file employees. The grounds for cancellation of union registration are not meant to be applied automatically, but indeed with utmost discretion. Where a remedy short of cancellation is available, that remedy should be preferred. In this case, no party will be prejudiced if Bathan were to be excluded from membership in the union. The vacancy he will thus create can then be easily filled up through the succession provision of appellee union’s constitution and by-laws. What is important is that there is an unmistakeable intent of the members of appellee union to exercise their right to organize. We cannot impose rigorous restraints on such right if we are to give meaning to the protection to labor and social justice clauses of the Constitution.

3. No, it does not affect the case.The allegation that the bargaining unit that respondent

Page 52: 33994637 Labor Rounds 1and2

Labor Law 2 A2010 - 52 - Disinisought to represent is no longer the same because of the dynamic nature of petitioner’s business, a lot of changes having occurred in the work environment, and that four of respondent’s officers are no longer connected with petitioner have no effect on the Court’s ruling that a certification election should be immediately conducted with respondent as one of the available choices. Petitioner’s bare manifestations adduce no reason why the certification election should not be conducted forthwith. If there are matters that have arisen since the filing of the petition that serve to delay or cancel the election, these can be threshed out during the pre-election conferences. Neither is the fact that some of respondent’s officers have since resigned from petitioner of any moment. The local/chapter retains a separate legal personality from that of its officers or members that remains viable notwithstanding any turnover in its officers or members.

DISPOSITIONWHEREFORE, the Petition is DENIED. Costs against petitioner.SO ORDERED.

3. IssuesA. Admission and Discipline of

Members

UST FACULTY UNION V BITONIO318 SCRA 185

PANGANIBAN; November 16, 1999

NATURESpecial civil action in the Supreme Court. Certiorari.

FACTS- Union announced a general assembly to elect next union officers. - TRO was issued by med-arbiter enjoining them from conducting election.- UST held a general faculty assembly, attended by both union members and non-members. Here, appellants were elected as new union officers by acclamation and clapping of hands.- Appellees filed instant petition to seek injunctive relief and to nullify results of election.- Bitonio upheld med-arbiter and said election was void. He rejected contention that it was a legitimate exercise of right to self organization

ISSUE/S1. Basis of right to self-organization (p5 of outline) / Workers with right of self-organization (p6 of outline) 2. Admission and discipline of members (p10 of outline)3. Voters list (p11 of outline)

HELD1. Ratio Self-organization is a fundamental right to form, join or assist labor organizations for collective bargaining, mutual aid and protection. Whether employed for a definite period or not, employee shall be considered as such, beginning on 1 st day of service, for purposes of membership in a labor union. Corollary to this right is the prerogative not to join. Reasoning The election can’t be considered as exercise of right to self-organization because the petitioners’ frustration over the performance of the respondents could not justify the method they chose to impose their will on the union.2. Ratio The union’s constitution and by-laws embody a covenant between a union and its members and constitute the fundamental law governing the members’ rights and obligations.Reasoning The election was tainted by irregularities. The general faculty assembly was not the proper forum for the election. Also, the grievances of the petitioners could have been brought up and resolved in accordance with the procedure laid down by the union’s CBL and by the Labor Code.3.

Ratio The union election is pursuant to union’s constitution and by-laws and right to vote is enjoyed only by union members. On the other hand, a certification election is the process of determining, through secret ballot, the sole and executive bargaining agent of the employees and all employees belonging to the appropriate bargaining unit can vote. Verily, union affairs cannot be decided in a non-union activity.Reasoning The election held was not a union election because the procedure in the CBL was not followed. It was also not a certification election because representation was not the issue. The participation of non-union members in the election aggravated its irregularity.

SALUNGA V CIR (NATIONAL BREWERY & ALLIED INDUSTRIES LABOR UNION OF THE PHILIPPINES,

ET AL)21 SCRA 216

CONCEPCION; September 27, 1967NATUREAppeal from a resolution of the Court of Industrial Relations

FACTS-Petitioner is an employee of San Miguel Brewery Inc., which, on October 2, 1959, entered with the Union, into a collective bargaining agreement. -Section 3 of the CBA reads: The company agrees to require as a condition of employment of those workers covered by this agreement who either are members of the UNION on the date of the signing of this agreement, or may join the UNION during the effectivity of this agreement, that they shall not voluntarily resign from the UNION earlier than thirty (30) days before the expiry date of this agreement as provided in Article XIII hereof, provided, however, that nothing herein contained shall be construed to require the company to enforce any sanction whatsoever against any employee or worker who fails to retain his membership in the UNION as hereinbefore stated, for any cause other than voluntary resignation or non-payment of regular union dues on the part of said employee or worker. -On August 18, 1961, petitioner tendered his resignation from the Union, which accepted it and transmitted it to the Company, with a request for the immediate implementation of said section 3. -The Company having informed him that his aforementioned resignation would result in the termination of his employment, in view of said section, petitioner wrote to the Union, on August 31, 1961, a letter withdrawing or revoking his resignation -On September 8, 1961, the Union told the Company that petitioner's membership could not be reinstated and insisted on his separation from the service, conformably with the stipulation above-quoted. -On September 20, 1961, the Union reiterated its request for implementation of said section 3, for which reason, on September 22, 1961, the Company notified petitioner that, in view of said letter and the aforementioned section, he had to be terminated-Petitioner sought help from the Union’s national president, but to no avail-On October 11, 1961, petitioner notified the PAFLU that he was appealing to its supreme authority, the PAFLU National Convention and requested that action on his case be deferred until such time as the Convention shall have acted on his appeal. A letter of the same date and tenor was sent, also, by the petitioner to the Union. Furthermore, he asked the Company to maintain the status quo, in the meantime. This notwithstanding, at the close of the business hours, on October 15, 1961, petitioner was discharged from the employment of the Company-Petitioner filed with the CIR a complaint for unfair labor practice-CIR found for petitioner but reversed itself upon MFR

ISSUEWON petitioner was dismissed due to an unfair labor practice on the part of the Union

HELDYes.

Page 53: 33994637 Labor Rounds 1and2

Labor Law 2 A2010 - 53 - DisiniAlthough, generally, a state may not compel ordinary voluntary associations to admit thereto any given individual, because membership therein may be accorded or withheld as a matter of privilege, the rule is qualified in respect of labor unions holding a monopoly in the supply of labor, either in a given locality, or as regards a particular employer with which it has a closed-shop agreement. -The reason is that the closed shop and the union shop cause the admission requirements of trade union to become affected with the public interest. Likewise, a closed shop, a union shop, or maintenance of membership clauses cause the administration of discipline by unions to be affected with the public interest.-Consequently, it is well settled that such unions are not entitled to arbitrarily exclude qualified applicants for membership, and a closed-shop provision would not justify the employer in discharging, or a union in insisting upon the discharge of, an employee whom the union thus refuses to admit to membership, without any reasonable ground therefor. Needless to say, if said unions may be compelled to admit new members, who have the requisite qualifications, with more reason may the law and the courts exercise the coercive power when the employee involved is a long standing union member, who, owing to provocations of union officers, was impelled to tender his resignation, which he forthwith withdrew or revoked. Surely, he may, at least, invoke the rights of those who seek admission for the first time, and can not arbitrarily he denied readmission.-Having been denied readmission into the Union and having been dismissed from the service owing to an unfair labor practice on the part of the Union, petitioner is entitled to reinstatement as member of the Union and to his former or substantially equivalent position in the Company, without prejudice to his seniority and/or rights and privileges, and with back pay, which back pay shall be borne exclusively by the Union. DispositionThe appealed resolution of the CIR is reversed

B. Retention of MembershipC. Discipline

VILLAR V INCIONG121 SCRA 44

GUERRERO; April 20, 1983

NATUREPetition for review by certiorari

FACTS- Petitioners were members of the Amigo Employees Union-PAFLU, a duly registered labor organization which was the existing bargaining agent of the employees in private respondent Amigo Manufacturing, Inc. They then signed a joint resolution which stated that they were disaffiliating themselves as members of the PAFLU, and were not further authorizing PAFLU to represent them in any CBA.- During a special meeting of the Union, a Resolution was approved which called for the investigation of all the petitioners for "continuously maligning, libelling and slandering not only the incumbent officers but even the union itself and the federation" and for causing divisiveness.- The union security clause was reincorporated in the new CBA: “any members who shall resign, be expelled, or shall in any manner cease to be a member of the UNION, shall be dismissed from his employment upon written request of the UNION to the Company.”- Petitioners contend that their acts do not constitute disloyalty as these are in the exercise of their constitutional right to self-organization.- The PAFLU President then rendered a decision finding them guilty of the charges and expelling them from the Union and as a consequence the Management of the employer is hereby requested to terminate them from their employment in conformity with the security clause.

ISSUE/S1. WON PAFLU had the authority to investigate the petitioners and, thereafter, expel them from the roll of membership of the Amigo Employees Union-PAFLU2. WON the petitioners were entitled to disaffiliate from the Union

HELD

1.YESRatio That PAFLU had the authority to investigate petitioners on the charges filed by their co-employees in the local union and after finding them guilty as charged, to expel them from the roll of membership of the Amigo Employees Union-PAFLU is clear under the constitution of the PAFLU to which the local union was affiliated. And pursuant to the security clause of the new CBA, reiterating the same clause in the old CBA, PAFLU was justified in applying said security clause.UNION-MEMBER RELATIONS: Discipline IssuesInherent in every labor union, or any organization for that matter, is the right of self-preservation. When members of a labor union, therefore, sow the seeds of dissension and strife within the union; when they seek the disintegration and destruction of the very union to which they belong, they thereby forfeit their rights to remain as members of the union which they seek to destroy. Prudence and equity, as well as the dictates of law and justice, therefore, compelling mandate the adoption by the labor union of such corrective and remedial measures in keeping with its laws and regulations, for its preservation and continued existence; lest by its folly and inaction, the labor union crumble and fall. 2. YESRatio Although they are entitled to disaffiliate from their union and form a new organization of their own, they must, however, suffer the consequences of their separation from the union under the security clause of the CBA.Reasoning Disaffiliation from a labor union is not open to legal objection. It is implicit in the freedom of association ordained by the Constitution. But this Court has laid down the ruling that a closed shop is a valid form of union security, and such provision in a collective bargaining agreement is not a restriction of the right of freedom of association guaranteed by the Constitution.Local Union Disaffiliation: Rule - Legality act - DisaffiliationThere are two hundred thirty four (234) union members in the Amigo Employees Union-PAFLU, and only 96 signed the "Sama-Samang Kapasiyahan." They constituted a small minority for which reason they could not have successfully disaffiliated the local union from PAFLU. It can be inferred that the majority wanted the union to remain an affiliate of PAFLU. The action of the majority must, therefore, prevail over that of the minority members.

Disposition The Order appealed from affirming the joint decision of the OIC granting clearance to terminate petitioners as well as dismissing their complaint with application for preliminary injunction, is hereby AFFIRMED.

D. Due Process Rules

BUGAY V KAPISANAN NG MGA MANGGAGAWA SA MRR

4 SCRA 487BAUTISTA ANGELO, February 23, 1962

NATUREAppeal from decision of lower court

FACTS-Bugay was the former auditor of the union. He delivered documents to the management, as ordered by the union secretary without the approval of the union, which the management used to charge the union president with falsification of documents. Charges for disloyalty and conduct unbecoming a union member were filed against Bugay, the union held investigations even without Bugay’s presence (thus, no chance to defend himself), and his expulsion was not submitted to the different chapters of the union as required by its constitution and by-laws. -Bugay filed a charge for unfair labor practice against the union before the CIR which ordered his reinstatement as union member and restoration of all his rights and privileges. SC affirmed. Bugay

Page 54: 33994637 Labor Rounds 1and2

Labor Law 2 A2010 - 54 - Disinialso filed claim for moral damages which the lower court dismissed based on lack of cause of action.

ISSUES1. WON Bugay had a cause of action (WON it can be shown that the union officers acted in bad faith in expelling him from the union to grant MD)2. WON Bugay was afforded due process

HELD1. YESReasoning. The main basis of appellant's action is his claim that because of the unfair labor practice committed by the officers of defendant union as found by the Court of Industrial Relations and the Supreme Court he has suffered moral damages because of the mental anguish, anxiety, social humiliation and besmirched reputation he has been subjected among the thousands of employees of the Manila Railroad Company. The CIR and the SC have found that his expulsion was illegal because of the irregularities committed in his investigation.

2. NOReasoning. In the investigation held and in the board meeting where the committee’s report recommending Bugay’s expulsion was approved, Bugay was not present. The committee assigned to summon him failed to serve notice upon him because he was in Lucena. However, proceedings still continued anent the absence of Bugay. Also, the union by-laws provided that majority of the chapters should act on the expulsion within 10days but only 2 did so, therefore, his expulsion was irregular and he is still a member of the union.

Disposition. WHEREFORE, the order appealed from is set aside. The case is remanded to the lower court for further proceedings, With costs against defendant.

E. Election Officers1. Voters List

TANCINCO V CALLEJA157 SCRA 203

UST FACULTY UNION V BITONIO318 SCRA 185

PANGANIBAN; November 16, 1999

NATURESpecial civil action in the Supreme Court. Certiorari.

FACTS- Union announced a general assembly to elect next union officers. - TRO was issued by med-arbiter enjoining them from conducting election.- UST held a general faculty assembly, attended by both union members and non-members. Here, appellants were elected as new union officers by acclamation and clapping of hands.- Appellees filed instant petition to seek injunctive relief and to nullify results of election.- Bitonio upheld med-arbiter and said election was void. He rejected contention that it was a legitimate exercise of right to self organization

ISSUE/SAdmission and discipline of members (p10 of outline)

HELDRatio The union’s constitution and by-laws embody a covenant between a union and its members and constitute the fundamental law governing the members’ rights and obligations.

Reasoning The election was tainted by irregularities. The general faculty assembly was not the proper forum for the election. Also, the grievances of the petitioners could have been brought up and resolved in accordance with the procedure laid down by the union’s CBL and by the Labor Code.

2. Disqualification Candidate

MANALAD V TRAJANO174 SCRA 322

Regalado, J.: June 28, 1989

FACTS-Petitioners were disqualified from running in the election of officers in their union; the order was set aside on appeal by Trajano, Director of the Bureau of Labor Relations (BLR). Petitioners won over private respondents; the latter filed a petition assailing the decision on appeal. -The Court promulgated a resolution: (1) to dismiss the petition (2) vacate all offices, including acting president private respondent Babula and turn over union management to BLR (3) order the holding of a special election supervised by the NCR Labor Office, which private respondents later won. -A petition was filed asking to hold private respondents in contempt for failure to turn over management of the union, disqualify them and invalidate votes cast in their favor, electing those with the 2nd highest votes, or hold a new election. Meanwhile, the terms of the offices in question had expired.

ISSUE (disqualification of candidate)WON private respondent candidates should be disqualified

HELDNO. As observed by respondent Trajano, the submission that Babula failed to completely turn over management of the union to the undersigned is within the competence and authority of the Supreme Court to pass upon considering that the mandate for such a turn-over came from the Court. Before the alleged disobedient party may be cited for contempt, the allegations must be clearly established, which petitioners failed to do. On the contrary, respondents have satisfactorily answered the averments thereon. -In any case, it would be pointless to annul expired terms. Even if the disqualification is justified, petitioners cannot be declared the winners simply because they received the second highest number of votes. Absent overriding considerations to the contrary, the will of the majority in the latter elections should be respected; where people have elected a man to office, it must be assumed that they did this with knowledge of his life and character, and that they disregarded of forgave his faults or misconduct, if he had been guilty of any.This case is DISMISSED for being moot and academic

3. Expulsion Remedy

KAPISANAN NG MANGGAGAWANG PINAGYAKAP (KMP) vs. TRAJANO

134 SCRA 236RELOVA; Jan 21 1985

NATUREPetition to review

FACTS- A petition was filed for the expulsion of the union officers of KMP on the ground of alleged falsification and misrepresentation.- Public respondent Director Trajano of the Bureau of Labor Relations directed the said Union to hold and conduct, pursuant to its constitution and by-laws and under the supervision of the Bureau of Labor Relations, a general membership meeting, to vote for or against the expulsion or suspension of the herein petitioner union

Page 55: 33994637 Labor Rounds 1and2

Labor Law 2 A2010 - 55 - Disiniofficers.

ISSUEWON the proper remedy against erring union officers is not referendum but union expulsion

HELDYES.Ratio The remedy against erring union officers is not referendum but union expulsion. Reasoning If herein union officers (also petitioners) were guilty of the alleged acts imputed against them, said public respondent pursuant to Article 242 of the New Labor Code and in the light of Our ruling in Duyag vs. Inciong, should have meted out the appropriate penalty on them, i.e., to expel them from the Union, as prayed for, and not call for a referendum to decide the issue.

However, the alleged falsification and misrepresentation of herein union officers were not supported by substantial evidence. The fact that they disbursed the amount of P1,278.00 from Union funds and later on was disallowed for failure to attach supporting papers thereon did not of itself constitute falsification and/or misrepresentation. The expenditures appeared to have been made in good faith and the amount spent for the purpose mentioned in the report, if concurred in or accepted by the members, are reasonable.

DISPOSITIONResolution SET ASIDE and the petition for expulsion of herein union officers is DISMISSED for having been rendered moot and academic by the election of herein union officers in the general membership meeting/election.

4. Election Valid

RODRIGUEZ V DIRECTOR, BUREAU OF LABOR RELATIONS

165 SCRA 239NARVASA; August 31, 1988

NATURESpecial civil action of certiorari (consolidated).

FACTS- The disputes concern the validity of the general elections for union officers and increase in union dues. - In PLDT, the fees to submit candidacy drastically increases. although some still paid the frees, complaints were filed with the Bureau of Labor Relations for its invalidation as excessive, prohibitive and arbitrary. - Elections for provinces of Visayas and Mindanao were held on dates different from those specified by the Legislative Council. The validity of the elections were challenged on the grounds of lack of due notice and adequate ground rules. - Med-Arbiter abdullah heard the cases and rendered judgment denying petitions to nullify the elections as well as the motion for contempt, but invalidated the increaseof filing fees for for certificates of candidacies. the OIC of Labor Relations overturned the judgment. The elections were nullifies for lack of notice to candidates and voters, failure to disseminate the election ground rules and disregard of the temporary restraining order of the Med-Arbiter. - Due to the proceedings assailing the validity of the elections pending before the Bureau of Labor Relations, the old officers continued to exercise the functions of their respective offices. the legislative council increased the union dues from P21-P50 a month. - Dir of Labor Relations rendered a decision revering that of the med-arbiter by ordering te cessation of the collection of the P29 increase aned return the amounts already collected.

ISSUE/S1. WON the general elections for union officers is valid2. WON the increase of the union dues is valid

HELD1. YESReasoning The decision is sustained for failure of the record to disclose any grave abuse of discretion on the part of the Director in finding that the elections was tainted with irregularities and therefore invalid. Undue haste, lack of adequate safeguards to ensure integrity of the voting and absence of the dates of balloting attended the elections. 2. NOReasoning The resolutiion does not bear the signature of at leasT 2/3 of the members of the council, contrary to the requirements of the union constitution and by-laws. the resolution increasing union dues is they're struck down as illegal and void, arbitrary and oppressive. Disposition Petition is dimissed.

F. Major Policy Matter

HALILI VS CIR136 SCRA 112

MAKASIAR; April 30, 1985

NATUREMotion to cite Atty. Benjamin C. Pineda, Ricardo Capuno and Manila Bank in contempt

FACTS136 The original cases involve disputes about the claims for

overtime of more than five hundred bus drivers and conductors of Halili Transit. The disputes were eventually settled when the contending parties reached an Agreement wherein the Union shall withdraw and dismiss with prejudice the case that they have filed before the CIR in exchange for some parcel of land and negotiable checks (P 25,000).

137 The parcel of land was eventually registered in the name of the Union and the cases filed were disposed.

138 The Union, through Atty. Benjamin C. Pineda, filed an urgent motion with the Ministry of Labor and Employment requesting for authority to sell and dispose of the property. The motion was granted. A prospective buyer, the Manila Memorial Park Cemetery, inc. expressed its misgivings on the authority of the Union to sell the property so Atty. Pineda filed a motion with the Supreme Court requesting for authority to sell the property. However, Atty. Pineda, without authority from the Supreme Court but relying on the earlier authority given him by the Ministry of Labor, filed another urgent motion with the latter, praying that the Union be authorized to sell the lot to the Manila Memorial Park Cemetery, Inc. and to make arrangements with it such that payment will be advanced for the real estate taxes inclusive of penalties, attorney's lien which is equivalent to a thirty-five percent (35%) of the total purchase price, and home developer's fee of P69,000.00. Apparently, the prospective purchaser had decided to withdraw its objection regarding the Union's authority to sell. So, the sale was finally consummated and Atty. Pineda got 35% as attorney’s fee.

139 When Atty. Jose C. Espinas (herein movant and alleged original counsel for the Union) learned of the sale and apportionment of the proceeds from past Union president Amado Lopez, he requested Labor Arbiter to allow him to look into the records of the case. He questions the legality of the orders which authorized the sale of the awarded property and the distribution of the proceeds from such purchase as well as the Attorney’s fee that Atty. Pineda received.

140 Atty. Espinas claims that he is the original counsel. while Atty. Pineda replied by saying that he handled the said

Page 56: 33994637 Labor Rounds 1and2

Labor Law 2 A2010 - 56 - Disinicase ALONE.

ISSUE:1. WON Atty. Pineda should be held in contempt of court.

HELD: YES

When Atty. Pineda appeared for the Union in these cases, he was still an associate of the law firm of Atty. Espinas. Yet, his appearance carried the firm name “B.C. Pineda and Associates," giving the impression that he was the principal lawyer in these cases when in truth, he was merely an associate to the Espinas firm. When he left the law firm for a year, Atty. Pineda continued handling the case with the arrangement that he would report the developments to the Espinas firm. When Atty. Pineda rejoined the Espinas firm in 1968, he did not reveal to his partners (he was made the most senior partner) that he had a retainer's contract with the union. Only the officers of the Union knew of the contract. Atty. Pineda joined the law firm of Atty. Espinas in 1965 when these cases were pending resolution. He always held office in the firm's place at Puyat Building, Escolta until 1974, except in 1966 to 1967 when he transferred to the Lakas ng Manggagawa Offices. During this one-year stint at the latter office, When he rejoined the law firm in 1968, he continued working on these cases and using the Puyat Building office as his address in the pleadings. entered into on January 1, 1967 which allegedly took effect in 1966. He stayed with the law firm until 1974 and still did not divulge the 1967 retainer's contract. The alleged retainer's contract between Atty. Pineda and the Union appears anomalous and even illegal as well as unethical considering that- the contract was executed only between Atty. Pineda and the officers of the Union chosen by about 125 members only. It was not a contract with the general membership, only 14% of the total membership of 897 was represented.Thus, without notice to the other lawyers and parties, Atty. Pineda commenced the proceeds before the NLRC with the filing of a motion and manifestation wherein he asked for authority to sell the property.

Atty. Pineda made the Union officers believe that he would be the one to pay the fees of Attys. Espinas and Lopez for which reason, the 35% increased fees was approved by the Union's board in good faith. The Union likewise confirms that Atty. Pineda came into the picture only when he was assigned by Atty. Espinas in, 1965 to execute the CIR decision which, thru Atty. Espinas handling, was upheld by this Court in L-24864 in 1968. The Union officers were aware that Atty. Espinas was the principal counsel even after Atty. Pineda's assignment. They also knew of the original contract for 20% attorney's fees which was increased to 35% by Atty. Pineda upon the arrangement that with the increase, he would answer for the payment of Attys. Espinas and Lopez' fees and for necessary representation expenses. Thus, based on these malicious actions and fraudulent misrepresentations, Atty. Pineda must be held liable.

DispositiveWHEREFORE, ATTY. BENJAMIN PINEDA IS HEREBY FOUND GUILTY OF INDIRECT CONTEMPT OF COURT FOR WHICH HE IS HEREBY SENTENCED TO IMPRISONMENT IN THE MANILA CITY JAIL UNTIL THE ORDERS OF THIS COURT DATED SEPTEMBER 1 AND SEPTEMBER 13, 1983 ARE COMPLIED WITH. ATTY. BENJAMIN PINEDA IS ALSO DIRECTED TO SHOW CAUSE WHY HE SHOULD NOT BE DISBARRED UNDER RULE 138 OF THE REVISED RULES OF COURT.

G. Union Funds1. Source-Payment-Attys

Fees

PACIFIC BANKING CORP. V CLAVE128 SCRA 112

AQUINO; March 7, 1984

NATURE

Petition for Certiorari

FACTS- There had been negotiations between the Pacific Banking Corporation and the Pacific Banking Corporation Employees Organization (PABECO) for a CBA for 1979 to 1981. Because of a deadlock, the Minister of Labor assumed jurisdiction over the controversy and rendered a decision directing the parties to execute a CBA in accordance with the terms and conditions set forth in his decision.- The union was represented in the negotiations by its president Paug, allegedly assisted as consultant by Umali, Jr., the president of the National Union of Bank Employees (NUBE) with which it was formerly affiliated. Lawyer Saavedra's earliest recorded participation in the case was on July 15 and 27, 1979 when he filed a motion for reconsideration and a supplemental motion. No action was taken on said motions. - The parties appealed to the Office of the President. The CBA negotiations were resumed. The union president took part in the second phase of the negotiations. Saavedra filed a memorandum. The Office of the President issueddirected the parties to execute a CBA containing the terms and conditions of employment embodied in the resolution.- Even before the formalization of the CBA, Saavedra filed in the case his notice of attorney 's lien. - The union officials requested the bank to withhold around P345,000 out of the total benefits as ten percent attorney's fees of Saavedra. At first, the bank interposed no objection to the request in the interest of harmonious labor-management relations. - For nearly a year, the Office of the President in four resolutions wrestled with the propriety of Saavedra's ten percent attorney's fees.

ISSUESWON the deduction from the monetary benefits awarded in a collective bargaining agreement the attorney's fees of the lawyer who assisted the union president in negotiating the agreement is legal.

RULING- NO. The case is covered squarely by the mandatory and explicit prescription of article 222 which is another guarantee intended to protect the employee against unwarranted practices that would diminish his compensation without his knowledge and consent. - "ART. 242. Rights and conditions of membership in a labor organization. - The following are the rights and conditions of membership in a labor organization:"(n) No special assessment or other extraordinary fees may be levied upon the members of a labor organization unless authorized by a written resolution of a majority of all the members at a general membership meeting duly called for the purpose. The secretary of the organization shall record the minutes of the meeting including the list of all members present, the votes cast, the purpose of the special assessment or fees and the recipient of such assessment or fees. The record shall be attested to by the president;."(o) Other than for mandatory activities under the Code, no special assessment, attorney's fees, negotiation fees or any other extraordinary fees may be checked off from any amount due an employee without an individual written authorization duly signed by the employee. The authorization should specifically state the amount, purpose and beneficiary of the deduction- Saavedra is entitled to the payment of his fees but article 222 ordains that union funds should be used for that purpose. The amount of P345,000 does not constitute union funds. It is money of the employees. The union, not the employees, is obligated to Saavedra.DispositionWHEREFORE, the petition is granted. The resolutions dated August 12 and December 15, 1980 and April 13, 1981 are reversed and set aside. The questioned amount of about P345,000, with its increments, if any, should be paid by the bank directly to its employees.

2. Examination Books

Page 57: 33994637 Labor Rounds 1and2

Labor Law 2 A2010 - 57 - Disini

DUYAG V INCIONG98 SCRA 522

Aquino, J.; July 5, 1980

NATUREAppeal from the decision of the Director of Labor Relation

FACTS- This case is about the removal of private respondents as union officers due to alleged irregularities and anomalies in the administration of the affairs of the union.- The Medarbiter ordered the expulsion of the union officers for violations of the constitution of the Union and the refund of the dues illegally exacted from union members. The Director of Labor Relations, however, reversed the ruling of the medarbiter in so far as the expulsion is concerned taking the position that it is only the membership of the Union which can removed its officers and claiming that the issue is a political question.

ISSUE/SWON the dismissal of the labor union officers is valid

HELDYes.

Ratio: The Director of Labor Relations erred in holding that, as a matter of policy, the tenure of union officers, being a "political question" is, generally, a matter outside his Bureau's jurisdiction and should be passed upon by the union members themselves.After hearing and even without submitting the matter to the union members, erring union officials may be removed by the Director of Labor Relations as clearly provided in article 242. That paragraph provides that any violation of the rights and conditions of union membership, as enumerated in paragraphs (a) to (p) of Article 242, "shall be a ground for cancellation of union registration or expulsion of officer from office, whichever is appropriate. At least thirty percent (30%) of all the members of a union or any member or members specially concerned may report such violation to the Bureau (of Labor Relations). The Bureau shall have the power to hear and decide any reported violation to mete the appropriate penalty".Reasoning: The labor officials should not hesitate to enforce strictly the law and regulations governing trade unions even if that course of action would curtail the so-called union autonomy and freedom from government interference.For the protection of union members and in order that the affairs of the union may be administered honestly, labor officials should be vigilant and watchful in monitoring and checking the administration of union affairs.Laxity, permissiveness, neglect and apathy in supervising and regulating the activities of union officials would result in corruption and oppression. Internal safeguards within the union can easily be ignored or swept aside by abusive, arrogant and unscrupulous union officials to the prejudice of the members.It is necessary and desirable that the Bureau of Labor Relations and the Ministry of Labor should exercise close and constant supervision over labor unions, particularly the handling of their funds, so as to forestall abuses and venalities. Disposition: WHEREFORE, (1) that portion of the decision of the med-arbiter, removing respondents Manalad, Leano and Puerto as union officers, is affirmed. (Respondent Amparo is no longer an officer of the union.)

3. Union Dues

RODRIGUEZ V DIRECTOR, BUREAU OF LABOR RELATIONS

165 SCRA 239NARVASA; August 31, 1988

NATURE

Special civil action of certiorari (consolidated).

FACTS- The disputes concern the validity of the general elections for union officers and increase in union dues. - In PLDT, the fees to submit candidacy drastically increases. although some still paid the frees, complaints were filed with the Bureau of Labor Relations for its invalidation as excessive, prohibitive and arbitrary. - Elections for provinces of Visayas and Mindanao were held on dates different from those specified by the Legislative Council. The validity of the elections were challenged on the grounds of lack of due notice and adequate ground rules. - Med-Arbiter abdullah heard the cases and rendered judgment denying petitions to nullify the elections as well as the motion for contempt, but invalidated the increaseof filing fees for for certificates of candidacies. the OIC of Labor Relations overturned the judgment. The elections were nullifies for lack of notice to candidates and voters, failure to disseminate the election ground rules and disregard of the temporary restraining order of the Med-Arbiter. - Due to the proceedings assailing the validity of the elections pending before the Bureau of Labor Relations, the old officers continued to exercise the functions of their respective offices. the legislative council increased the union dues from P21-P50 a month. - Dir of Labor Relations rendered a decision revering that of the med-arbiter by ordering te cessation of the collection of the P29 increase aned return the amounts already collected.

ISSUE/S1. WON the general elections for union officers is valid2. WON the increase of the union dues is valid

HELD1. YESReasoning The decision is sustained for failure of the record to disclose any grave abuse of discretion on the part of the Director in finding that the elections was tainted with irregularities and therefore invalid. Undue haste, lack of adequate safeguards to ensure integrity of the voting and absence of the dates of balloting attended the elections. 2. NOReasoning The resolutiion does not bear the signature of at leasT 2/3 of the members of the council, contrary to the requirements of the union constitution and by-laws. the resolution increasing union dues is they're struck down as illegal and void, arbitrary and oppressive. Disposition Petition is dimissed.

H. Source—Payment—Special Assessment

PALACOLV FERRER-CALLEJA182 SCRA 710

GANCAYCO; February 26, 1990

FACTS- October 12, 1987 > The president of Manila CCBPI Sales Force Union as the collective bargaining agent of all regular salesmen, regular helpers, and relief helpers of the Manila Plant and Metro Manila Sales Office of Coca-Cola Bottlers (Philippines), Inc. submitted to the Company the ratification by the union members of the new CBA and authorization for the Company to deduct union dues equivalent to P10.00 every payday or P20.00 every month and, in addition, 10% by way of special assessment, from the CBA lump-sum pay granted to the union members for putting up a cooperative and credit union; purchase vehicles and other items needed for the benefit of the officers and the general membership; and for the payment for services rendered by union officers, consultants and others with an additional proviso stating that the "matter of allocation ... shall be at the discretion of our incumbent Union President." - This "Authorization and CBA Ratification" was obtained by the Union through a secret referendum held in separate local membership meetings on various dates. The total membership of the Union was about 800. Of this number, 672 members originally authorized the 10% special assessment, while 173 opposed the same.- Subsequently however, one hundred seventy (170) members of the

Page 58: 33994637 Labor Rounds 1and2

Labor Law 2 A2010 - 58 - DisiniUnion submitted documents to the Company stating that although they have ratified the new CBA, they are withdrawing or disauthorizing the deduction of any amount from their CBA lump sum. Later, 185 other union members submitted similar documents expressing the same intent. These members, numbering 355 in all (170 + 185), added to the original oppositors of 173, turned the tide in favor of disauthorization for the special assessment, with a total of 528 objectors and a remainder of 272 supporters. - Petitioners assailed the 10% special assessment as a violation of Article 241(o) in relation to Article 222(b) of the Labor Code. Article 222(b) provides as follows:

ART. 222. Appearances and Fees. (b) No attorney's fees, negotiation fees or similar charges of any kind arising from any collective bargaining negotiations or conclusion of the collective agreement shall be imposed on any individual member of the contracting union; Provided, however, that attorney's fees may be charged against union funds in an amount to be agreed upon by the parties. Any contract, agreement or arrangement of any sort to the contrary shall be null and void.On the other hand, Article 241(o) mandates that: ART. 241. Rights and conditions of membership in a labor organization.xxx xxx xxx(o) Other than for mandatory activities under the Code, no special assessments, attorney's fees, negotiation fees or any other extraordinary fees may be checked off from any amount due to an employee without an individual written authorization duly signed by the employee. The authorization should specifically state the amount, purpose and beneficiary of the deduction

ISSUEWON a special assessment can be validly deducted by a labor union from the lump-sum pay of its members, granted under a collective bargaining agreement (CBA), notwithstanding a subsequent disauthorization of the same by a majority of the union members

HELDNODoctine N o check-offs from any amount due employees may be effected without individual written authorizations duly signed by the employees specifically stating the amount, purpose, and beneficiary of the deduction (Galvadores v. Trajano). This is in accord with the constitutional principle of the State affording full protection to labor.- the failure of the Union to comply strictly with the requirements set out by the law invalidates the questioned special assessment. Substantial compliance is not enough in view of the fact that the special assessment will diminish the compensation of the union members. Their express consent is required, and this consent must be obtained in accordance with the steps outlined by law, which must be followed to the letter. No shortcuts are allowed. Disposition Petition is hereby GRANTED. Coca-Cola Bottlers (Philippines), Inc. is hereby ordered to immediately remit the amount of P1,267,863.39 to the respective union members from whom the said amount was withheld.

GABRIEL V SECRETARY OF LABOR328 SCRA 247

QUISUMBING; March 16, 2000

NATURESpecial civil action of certiorari

FACTS- Petitioners comprise the Executive Board of the Solidbank Union, while the Private Respondents are the members of the union.- the union decided to retain the services of an atty Lacsina as union counsel in connection with the negotiations for a new CBA. Thus, the board called a general membership meeting for this purpose, whereby majority of all union members approved and signed a resolution confirming the decision to engage the services of the atty.- the resolution provided that 10% of the total economic benefits that may be secured through the nego will be given to the Atty Lacsina as atty’s fees. It also contained an authorization for Solidbank Corp to check-off the

atty’s fees from the payment of benefits to the EEs and turn over the amount to the atty. The bank then complied with the check-off- private respondents then filed a complaint against the board for illegal deduction of atty’s fees. The arbiter ordered the board to refund the complainants for the illegally deducted anount. The order was appealed to the Sec. of labor, who in turn ordered that the refund be limited to those union members who have not signified their conformity to the check-off. In an MFR, the secretary affirmed and modified that the reimbursement should be charged to the union’s general fund. Thus, this petition.

ISSUE1. WON the check-off was validly made

HELD1. NORatio The Art.24(o) provides: Other than for mandatory activities under the Code, no special assessment, attorney's fees, negotiation fees or any other extraordinary fees may be checked off from any amount due to an employee without an individual written authorization duly signed by the employee. The authorization should specifically state the amount, purpose and beneficiary of the deduction.The check-off, made without valid individual authorizations, cannot be held valid. No deduction can be made from the salaries of the concerned employees other than those mandated by law.Reasoning Art.241 has 3 requisites for the validity of special assessment for the union’s incidental expenses, atty’s fees, and representation expenses: (1) authorization by a written resolution of the majority of all members at the general membership meeting called for the purpose; (2) secretary's record of the minutes of the meeting; and (3) individual written authorization for check off duly signed by the EEs concerned.- The General Membership Resolution of the SolidBank Union didn’t satisfy requirements laid down by law and jurisprudence. There were no individual written check off authorizations by the EEs concerned and so the assessment cannot be legally deducted by the ER.- Palacol v Ferrer-Calleja: express consent of EEs is required, and this consent must be obtained in accordance with the steps outlined by law, which must be followed to the letter. No shortcuts are allowed. - BPIEU-ALU vs. NLRC: Art.222(b) prohibits payment of atty's fees only when it is effected through forced contributions from workers from their own funds as distinguished from the union funds. The purpose is to prevent imposition on the workers of the duty to individually contribute their respective shares in the fee to be paid the atty for his services on behalf of the union in its negotiations with mgt. The obligation to pay the atty’s fees belongs to the union and cannot be shunted to the workers as their direct responsibility. Neither the lawyer nor the union itself may require the individual worker to assume the obligation to pay atty’s fees from their own pockets. So categorical is this intent that the law makes it clear that any agreement to the contrary shall be null and void ab initio Disposition Petition is denied.

ABS - CBN SUPERVISORS EMPLOYEE UNION MEMBERS V. ABS - CBN BROADCASTING CORP.

304 SCRA 489PURISIMA; March 11, 1999

NATURE:Special civil action for Certiorari

ABS-CBN Supervisors Emloyees Union and ABS-CBN Broadcasting Corporation signed and a Collective Bargaining Agreement providing for a 10% special assessment for union incidental expenses, attorneys fees and representation expenses.

ISSUE:WON the provisions for the check off regarding the 10% special

Page 59: 33994637 Labor Rounds 1and2

Labor Law 2 A2010 - 59 - Disiniassessment for union incidental expenses, attorneys fees and representation expenses are valid.

HELD: YesArticle 241 provides for three (3) requisites for the validity of

the special assessment for Union's incidental expenses, attorney's fees and representation expenses namely:1) authorization by a written resolution of the majority of all the members at the general membership meeting duly called for the purpose; 2) secretary's record of the minutes of the meeting; and 3) individual written authorization for check-off duly signed by the employee concerned.

After a thorough review of the records on hand, we find that the three (3) requisites for the validity of the ten percent (10%) special assessment for Union's incidental expenses, attorney's fees and representation expenses were met.

It can be gleaned that on July 14, 1989, the ABS-CBN Supervisors Employee Union held its general meeting, whereat it was agreed that a ten percent (10%) special assessment from the total economic package due to every member would be checked-off to cover expenses for negotiation, other miscellaneous expenses and attorney's fees. The minutes of the said meeting were recorded by the Union's Secretary and noted by its President.

On May 24, 1991, said Union held its General Membership Meeting, wherein majority of the members agreed that "in as much as the Union had already paid Atty. P. Pascual the amount of P500,000.00, the same must be shared by all the members until this is fully liquidated."Eighty-five (85) members of the same Union executed individual written authorizations for check-off. There is then, the presumption that such check-off authorizations were executed voluntarily by the signatories thereto.

Petitioners further contend that Article 241 (n) of the Labor Code, as amended, on special assessments, contemplates a general meeting after the conclusion of the collective bargaining agreement.Subject Article does not state that the general membership meeting should be called after the conclusion of a collective bargaining agreement.

In BPIEU-ALU, the petitioners, impugned the Order of the NLRC, holding that the validity of the five percent (5%) special assessment for attorney's fees is contrary to Article 222, paragraph (b) of the Labor Code, as amended. The court ratiocinated, thus:"The Court reads the aforecited provision as prohibiting the payment of attorney's fees only when it is effected through forced contributions from the workers from their own funds a distinguished from the union funds. The purpose of the provision is to prevent imposition on the workers of the duty to individually contribute their respective shares in the fee to be paid the attorney for his services on behalf of the union in its negotiations with the management. xxx"

However, the public respondent overlooked the fact that in the said case, the deduction of the stipulated five percent (5%) of the total economic benefits under the new collective bargaining agreement was applied only to workers who gave their individual signed authorizations. The Court explained:"xxx And significantly, the authorized deduction affected only the workers who adopted and signed the resolution and who were the only ones from whose benefits the deductions were made by BPI. No similar deductions were taken from the other workers who did not sign the resolution and so were not bound by it." The inapplicability of Palacol lies in the fact that it has a different factual milieu from the present case. In Palacol, the check-off authorization was declared invalid because majority of the Union members had withdrawn their individual authorizations.In this case, the majority of the Union members gave their individual written check-off authorizations for the ten percent (10%) special assessment. And they have never withdraw their individual written authorizations for check-off.Premises studiedly considered, we are of the irresistable conclusion and, so find, that the ruling in BPIEU-ALU vs. NLRC that 1) the prohibition against attorney's fees in Article 222, paragraph (b) of the Labor Code applies only when the payment of attorney's fees is effected through forced contributions from the workers; and 2) that no deductions must be taken from the workers who did not sign the check-off authorization, applies to the case under consideration.

I. Mandatory ActivityDefinition

AMBROCIO VENGCO ET. AL vs. DIR. CRESENCIO B. TRAJANO

170 SCRA 155 (1989)MEDIALDEA, J.:

NATURE Petition for certiorari

FACTS

-Sometime in the latter part of 1981, the Management of the Anglo-American Tobacco Corporation and the Kapisanan ng Manggagawa sa Anglo-American Tobacco Corporation. (FOI-TAF) entered into a compromise agreement whereby the company will pay to the union members the sum of P150,000.00 for their claims arising from the unpaid emergency cost of living allowance (ECOLA) and other benefits which were the subject of their compliant before the Ministry of Labor. -Respondent Emmanuel Timbungco (Timbungco, for short) who is the union president received the money which was paid in installments. Thereafter, he distributed the amount among the union members. Petitioners Ambrocio Vengco, Ramon Moises, Rafael Wagas and 80 others (Vengco, et al., for short) who are union members noted that Timbungco was not authorized by the union workers to get the money; and that ten percent (10%) of the P150,000.00 had been deducted to pay for attorney's fees without their written authorization in violation of Article 242(o) of the Labor Code. -Vengco et al. demanded from Timbungco an accounting of how the P150,000.00 was distributed to the members. Timbungco did not give in to their demand. Thus Vengco, et al. filed a complaint with the Ministry of Labor.

ISSUEWhether or not Timbungco is guilty of illegally deducting 10% attorneys' fees from petitioners' backwages

HELD YES.

RATIO-Other than for mandatory activities under the Code, no special assessment, attorney's fees, negotiation fees or any other extraordinary fees may be checked off from any amount due an employee without an individual written authorization duly signed by an employee. The authorization should specifically state the amount, purpose and beneficiary of the deduction. (Article 241, LC)

-A mandatory activity has been defined as a judicial process of settling dispute laid down by the law. In the instant case, the amicable settlement entered into by the management and the union CAN NOT BE CONSIDERED AS A MANDATORY ACTIVITY UNDER THE CODE.

REASONING-The law is explicit. IT REQUIRES THE INDIVIDUAL WRITTEN AUTHORIZATION OF EACH EMPLOYEE

CONCERNED, TO MAKE THE DEDUCTION OF ATTORNEY'S FEES VALID.

-Moreover, Book III, Rule VIII, Section II of the Implementing Rules cited by Timbungco which dispenses with the required written authorization from the employees concerned does not apply in this

Page 60: 33994637 Labor Rounds 1and2

Labor Law 2 A2010 - 60 - Disinicase. This provision envisions a situation where there is a judicial or administrative proceedings for recovery of wages. Upon termination of the proceedings, the law allows a deduction for attorney's fees of 10% from the total amount due to a winning party. In the herein case, the fringe benefits received by the union members consist of back payments of their unpaid emergency cost of living allowances which are totally distinct from their wages. Allowances are benefits over and above the basic salaries of the employees We have held that such allowances are excluded from the concept of salaries or wages. In addition, the payment of the fringe benefits were effected through an amicable settlement and not in an administrative proceeding.

DISPOSITIVE: ACCORDINGLY, the petition is granted.

CBA Negotiation

GALVADORES V TRAJANO144 SCRA 138(not yet available)

J. Union InformationUnion Officer-Obligation

CONTINENTAL CEMENT CORP. LABOR UNION V CONTINENTAL CEMENT

189 SCRA 134Company was engaged in the manufacture of cement which is a vital industry in which a strike or lockout is prohibited. The union members engaged in a lock-out and strike. Even assuming not engaged in a vital industry, the strike was illegal. It was not in connection with anyunresolved economic issue in collective bargaining which is the only ground for which a lawful strike can be held as found in Section 7 of the Rules and Regulations implementing PD 823. The issue concerned merely the implementation of an arbitration award of the NLRC. Union had a remedy by applying for a writ of execution to enforce that award. The strikers did violate the nostrike policy. They repeatedly defied the orders of the Director of Labor Relations. Officers had the duty to guide their members to respect the law. Instead, they urged them to violate the law and defy the duly constituted authorities. Their responsibility is greater than that of the members. Their (the officers) dismissal from the service is a just penalty. It is within the power of the NLRC to order the removal of the officers. In Art. 242, it shall be the duty of any labor organization and its officers to inform its members on provisions of the constitution and by-laws, collective bargaining agreement, the prevailing labor relations system and all their rights and obligations under existing labor laws. Any violation of the above rights and conditions shall be a ground for cancellation of union registration or expulsion of an officer from office. The officers misinformed the members and led them into staging an illegal strike.

K. Enforcement and Remedies-Procedure and Sanctions

Jurisdiction-ExhaustionInternal Remedies

Requirements:

VILLAR V INCIONG121 SCRA 44

GUERRERO; April 20, 1983

NATUREPetition for review by certiorari

FACTS- Petitioners were members of the Amigo Employees Union-PAFLU, a duly registered labor organization which was the existing bargaining agent of the employees in private respondent Amigo Manufacturing, Inc. They then signed a joint resolution which stated that they were disaffiliating themselves as members of the PAFLU, and were not further authorizing PAFLU to represent them in any CBA.- During a special meeting of the Union, a Resolution was approved which called for the investigation of all the petitioners for "continuously maligning, libelling and slandering not only the incumbent officers but even the union itself and the federation" and for causing divisiveness.- The union security clause was reincorporated in the new CBA: “any members who shall resign, be expelled, or shall in any manner cease to be a member of the UNION, shall be dismissed from his employment upon written request of the UNION to the Company.”- Petitioners contend that their acts do not constitute disloyalty as these are in the exercise of their constitutional right to self-organization.- The PAFLU President then rendered a decision finding them guilty of the charges and expelling them from the Union and as a consequence the Management of the employer is hereby requested to terminate them from their employment in conformity with the security clause.

ISSUE/S1. WON PAFLU had the authority to investigate the petitioners and, thereafter, expel them from the roll of membership of the Amigo Employees Union-PAFLU2. WON the petitioners were entitled to disaffiliate from the Union

HELD1.YESRatio That PAFLU had the authority to investigate petitioners on the charges filed by their co-employees in the local union and after finding them guilty as charged, to expel them from the roll of membership of the Amigo Employees Union-PAFLU is clear under the constitution of the PAFLU to which the local union was affiliated. And pursuant to the security clause of the new CBA, reiterating the same clause in the old CBA, PAFLU was justified in applying said security clause.UNION-MEMBER RELATIONS: Discipline IssuesInherent in every labor union, or any organization for that matter, is the right of self-preservation. When members of a labor union, therefore, sow the seeds of dissension and strife within the union; when they seek the disintegration and destruction of the very union to which they belong, they thereby forfeit their rights to remain as members of the union which they seek to destroy. Prudence and equity, as well as the dictates of law and justice, therefore, compelling mandate the adoption by the labor union of such corrective and remedial measures in keeping with its laws and regulations, for its preservation and continued existence; lest by its folly and inaction, the labor union crumble and fall. 2. YESRatio Although they are entitled to disaffiliate from their union and form a new organization of their own, they must, however, suffer the consequences of their separation from the union under the security clause of the CBA.Reasoning Disaffiliation from a labor union is not open to legal objection. It is implicit in the freedom of association ordained by the Constitution. But this Court has laid down the ruling that a closed shop is a valid form of union security, and such provision in a collective bargaining agreement is not a restriction of the right of freedom of association guaranteed by the Constitution.Local Union Disaffiliation: Rule - Legality act - DisaffiliationThere are two hundred thirty four (234) union members in the Amigo Employees Union-PAFLU, and only 96 signed the "Sama-Samang

Page 61: 33994637 Labor Rounds 1and2

Labor Law 2 A2010 - 61 - DisiniKapasiyahan." They constituted a small minority for which reason they could not have successfully disaffiliated the local union from PAFLU. It can be inferred that the majority wanted the union to remain an affiliate of PAFLU. The action of the majority must, therefore, prevail over that of the minority members.

Disposition The Order appealed from affirming the joint decision of the OIC granting clearance to terminate petitioners as well as dismissing their complaint with application for preliminary injunction, is hereby AFFIRMED.

RODRIGUEZ V DIRECTOR, BUREAU OF LABOR RELATIONS

265 SCRA 239NARVASA; August 31, 1988

NATURESpecial civil action of certiorari (consolidated).

FACTS- The disputes concern the validity of the general elections for union officers and increase in union dues. - In PLDT, the fees to submit candidacy drastically increases. although some still paid the frees, complaints were filed with the Bureau of Labor Relations for its invalidation as excessive, prohibitive and arbitrary. - Elections for provinces of Visayas and Mindanao were held on dates different from those specified by the Legislative Council. The validity of the elections were challenged on the grounds of lack of due notice and adequate ground rules. - Med-Arbiter abdullah heard the cases and rendered judgment denying petitions to nullify the elections as well as the motion for contempt, but invalidated the increaseof filing fees for for certificates of candidacies. the OIC of Labor Relations overturned the judgment. The elections were nullifies for lack of notice to candidates and voters, failure to disseminate the election ground rules and disregard of the temporary restraining order of the Med-Arbiter. - Due to the proceedings assailing the validity of the elections pending before the Bureau of Labor Relations, the old officers continued to exercise the functions of their respective offices. the legislative council increased the union dues from P21-P50 a month. - Dir of Labor Relations rendered a decision revering that of the med-arbiter by ordering te cessation of the collection of the P29 increase aned return the amounts already collected.

ISSUE/S1. WON the general elections for union officers is valid2. WON the increase of the union dues is valid

HELD1. YESReasoning The decision is sustained for failure of the record to disclose any grave abuse of discretion on the part of the Director in finding that the elections was tainted with irregularities and therefore invalid. Undue haste, lack of adequate safeguards to ensure integrity of the voting and absence of the dates of balloting attended the elections. 2. NOReasoning The resolutiion does not bear the signature of at leasT 2/3 of the members of the council, contrary to the requirements of the union constitution and by-laws. the resolution increasing union dues is they're struck down as illegal and void, arbitrary and oppressive. Disposition Petition is dimissed.

VERCELES VS BLR451 SCRA 338

CHICO-NAZARIO; Feb. 15, 2005

NATUREPetition for review on certiorari

FACTS- Private respondents Rodel E. Dalupan, Efren J. De Ocampo, Proceso Totto, Jr., Elizabeth Alarca, and Elvira S. Manalo are members of the

University of the East Employees Association (UEEA). - They each received a Memorandum from the UEEA charging them with spreading false rumors and creating disinformation among the members of the said association. The acts of the respondents allegedly fall under General Assembly Resolution No. 4, Series of 1979- they denied the allegations. They stated that any sanction that will be imposed by the committee would be violative of their right to due process.- Ernesto Verceles, in his capacity as president of the association, through a Memorandum, informed Rodel Dalupan, et al., that their membership in the association has been suspended and shall take effect immediately upon receipt thereof. Verceles said he was acting upon the disciplinary committee’s finding of a prima facie case against them. Respondent Uy also received a similar memorandum. - a complaint for illegal suspension, willful and unlawful violation of UEEA constitution and by-laws, refusal to render financial and other reports, deliberate refusal to call general and special meetings, illegal holdover of terms and damages was filed by the respondents against herein before DOLE-NCR- a decision was rendered by Regional Director Lim, adverse to petitioners- The petitioners appealed to the Bureau of Labor Relations of the DOLE. During the pendency of this appeal an election of officers was held by the UEEA. The appeal, however, was dismissed for lack of merit.- the petitioners filed a petition for review on certiorari before this Court.

ISSUES1. WON the CA committed an error in upholding the DOLE-NCR and BLR-DOLE decisions based only on the complaint and answer.2. WON the CA committed an error in holding the election as invalid and a nullity.3. WON it was erroneous to uphold the BLR-DOLE’s finding that the suspension was illegal 4. WON the alleged non-holding of meetings and alleged non-submission of reports are moot and academic, and whether the decision to hold meetings and submit reports contradict and override the sovereign will of the majority.

HELD1. NORatio The Court of Appeals was justified in upholding the DOLE-NCR and BLR-DOLE decisions based on the complaint and answer. The petitioners’ line of reasoning that since no position papers were submitted, no decision may be made by the adjudicating body cannot be accepted. As ruled by Regional Director Lim in his decision, the complaint and the answer thereto were adopted as the parties’ position papers. Thereafter, the case shall be deemed submitted for resolution. Labor laws mandate the speedy disposition of cases, with the least attention to technicalities but without sacrificing the fundamental requisites of due process. The essence of due process is simply an opportunity to be heard. In this case, it cannot be said that there was a denial of due process on the part of the petitioners because they were given all the chances to refute the allegations of the private respondents, and the delay in the proceedings before the DOLE-NCR was clearly attributable to them. The argument that there was failure to exhaust administrative remedies cannot be sustained. One of the instances when the rule of exhaustion of administrative remedies may be disregarded is when there is a violation of due process. In this case, the respondents have chronicled from the very beginning that they were indefinitely suspended without the benefit of a formal charge sufficient in form and substance. Therefore, the rule on exhaustion of administrative remedies cannot squarely apply to them.2. NORatio The election of UEEA officers cannot acquire a semblance of legality. First, it was conducted pursuant to the aforesaid (erroneous) order of the Regional Director as manifested by the petitioners. Second, it was purposely done to pre-empt the resolution of the case by the BLR and to deprive private respondents their substantial right to participate in the election. Third, petitioners cannot be allowed to

Page 62: 33994637 Labor Rounds 1and2

Labor Law 2 A2010 - 62 - Disinitake an inconsistent position to later on claim that the election was held because it was already due while previously declaring that it was made in line with the order of the Regional Director, for this would go against the principle of fair play. Thus, while the BLR was wrong in affirming the order of the Regional Director for the immediate holding of election, it was right in nullifying the election of officers. It was simply improper for the petitioners to implement the said order which was then one of the subjects of their appeal in the BLR. To hold otherwise would be to dispossess the BLR of its inherent power to control the conduct of the proceedings of cases pending before it for resolution.3. NORatio Petitioners have failed to show that the findings of facts and conclusions of law of both the DOLE-NCR and BLR-DOLE were arrived at with grave abuse of discretion or without substantial evidence. A careful review of the pleadings reveals that the decision and resolutions of the concerned agencies were correctly anchored in law and on substantial evidence.4. NORatio The passage of General Assembly Resolution No. 10 and Resolution No. 8, Series of 2000, which supposedly cured the lapses committed by the association’s officers and reiterated the approval of the general membership of the acts and collateral actions of the association’s officers cannot redeem the petitioners from their predicament. The obligation to hold meetings and render financial reports is mandated by UEEA’s constitution and by-laws. Prompt compliance in rendering financial reports together with the holding of regular meetings with the submission of the minutes thereon with the BLR-DOLE and DOLE-NCR shall negate any suspicion of dishonesty on the part of UEEA’s officers. This is not only true with UEEA, but likewise with other unions/associations, as this matter is imbued with public interest. Undeniably, transparency in the official undertakings of union officers will bolster genuine trade unionism in the country.

DISPOSITION The Decision and Resolution of the CA affirmed.

DIAMONON v DOLE (LAGUESMA)327 SCRA 282

De Leon; March 7, 2000

NATUREPetition for certiorari

FACTS- petitioner Diamonon filed 2 complaints before the DOLE after being removed from his capacity as vice president of the unions NACUSIP and PACIWU. In his first complaint, he questioned the validity of his removal from office and in the second, he accused both the national treasurer and national president of NACUSIP and PACIWU (private respondents) for wanton violation of the Constitution and By-Laws of both orgs, unauthorized and illegal disbursement of union funds and also, abuse of authority.- the 1st complaint was decided in his favor, his removal was declared null and void. The 2nd complaint was dismissed for lack of personality in view of his removal from the offices he held. Diamonon appealed but public respondent Laguesma, as undersecretary of DOLE, held that petitioner’s failure to show, in his complaint, that the administrative remedies provided for in the constitution and by-laws of both unions have been exhausted or such remedies are not available, was fatal. With his MFR denied, Diamonon filed this case. ISSUEWON respondent Laguesma acted with grave abuse of discretion in dismissing the appeal for non-exhaustion of administrative remedies.

HELDNo. Ratio When the Constitution and By-Laws of Unions dictate the remedy for internal dispute, they should be resorted to before recourse can be made to the appropriate administrative or judicial body. - A party with an administrative remedy must not merely initiate the

prescribed administrative procedure to obtain relief, but also pursue it to its appropriate conclusion before seeking judicial intervention.Reasoning Not only did petitioner fail to comply with Section 2, Rule VIII, Book V of the Implementing Rules of the Labor Code but also the record reveals that neither did he exhaust the remedies set forth by the Constitution and by-laws of both unions. In the National Convention of PACIWU and NACUSIP, nothing was heard of petitioner’s complaint against private respondents on the latter’s alleged unauthorized and illegal disbursement of union funds. His failure to seek recourse before the National convention on his complaint against private respondents taints his action with prematurity.Disposition Petition DISMISSED.

Remedy

KAPISANAN NG MANGGAGAWANG PINAGYAKAP (KMP) vs. TRAJANO

134 SCRA 236RELOVA; Jan 21 1985

NATUREPetition to review

FACTS- A petition was filed for the expulsion of the union officers of KMP on the ground of alleged falsification and misrepresentation.- Public respondent Director Trajano of the Bureau of Labor Relations directed the said Union to hold and conduct, pursuant to its constitution and by-laws and under the supervision of the Bureau of Labor Relations, a general membership meeting, to vote for or against the expulsion or suspension of the herein petitioner union officers.

ISSUEWON the proper remedy against erring union officers is not referendum but union expulsion

HELDYES.Ratio The remedy against erring union officers is not referendum but union expulsion. Reasoning If herein union officers (also petitioners) were guilty of the alleged acts imputed against them, said public respondent pursuant to Article 242 of the New Labor Code and in the light of Our ruling in Duyag vs. Inciong, should have meted out the appropriate penalty on them, i.e., to expel them from the Union, as prayed for, and not call for a referendum to decide the issue.

However, the alleged falsification and misrepresentation of herein union officers were not supported by substantial evidence. The fact that they disbursed the amount of P1,278.00 from Union funds and later on was disallowed for failure to attach supporting papers thereon did not of itself constitute falsification and/or misrepresentation. The expenditures appeared to have been made in good faith and the amount spent for the purpose mentioned in the report, if concurred in or accepted by the members, are reasonable.

DISPOSITIONResolution SET ASIDE and the petition for expulsion of herein union officers is DISMISSED for having been rendered moot and academic by the election of herein union officers in the general membership meeting/election.

UNION AFFILIATION LOCAL AND PARENT UNION RELATIONS—ILO Convention No. 87, Art. 5

Page 63: 33994637 Labor Rounds 1and2

Labor Law 2 A2010 - 63 - Disini1. Affiliation: Purpose of Nature of

RelationsPurpose

PHIL SKYLANDERS INC V NLRC (PAFLU)375 SCRA 369

BELLOSILLO; Jan 31, 2002

NATUREPetition for certiorari

FACTS375 Nov 1993, the Philippine Skylanders Employees Association (PSEA),

a local labor union affiliated with the Philippine Association of Free Labor Unions (PAFLU) September (PAFLU), won in the certification election conducted among the rank and file employees of Philippine Skylanders, Inc. (PSI). Its rival union, Philippine Skylanders Employees Association-WATU (PSEA-WATU) immediately protested the result of the election before the Secretary of Labor.

376 Several months later, pending settlement of the controversy, PSEA sent PAFLU a notice of disaffiliation citing as reason PAFLU's supposed deliberate and habitual dereliction of duty toward its members.

377 PSEA subsequently affiliated itself with the National Congress of Workers (NCW), changed its name to Philippine Skylanders Employees Association - National Congress of Workers (PSEA-NCW), and allowed the former officers of PSEA-PAFLU to continue occupying their positions as elected officers PSEA-NCW.

378 On 17 March 1994 PSEA-NCW entered into a collective bargaining agreement with PSI which was immediately registered with DOLE.

379 PAFLU Secretary General Serafin Ayroso wrote Mariles C. Romulo requesting a copy of PSI's audited financial statement. Ayroso explained that with the dismissal of PSEA-WATU's election protest the time was ripe for the parties to enter into a collective bargaining agreement. PSI through its personnel manager Francisco Dakila denied the request citing as reason PSEA's disaffiliation from PAFLU and its subsequent affiliation with NCW.

380 PAFLU through Serafin Ayroso filed a complaint for unfair labor practice against PSI, its president Mariles Romulo and personnel manager Francisco Dakila. PAFLU amended its complaint by including the elected officers of PSEA-PAFLU as additional party respondents.

381 Labor Arbiter declared PSEA's disaffiliation from PAFLU invalid and held PSI, PSEA-PAFLU and their respective officers guilty of unfair labor practice.

382 NLRC upheld the Decision of the Labor Arbiter and conjectured that since an election protest questioning PSEA-PAFLU's certification as the sole and exclusive bargaining agent was pending resolution before the Secretary of Labor, PSEA could not validly separate from PAFLU, join another national federation and subsequently enter into a collective bargaining agreement with its employer-company

ISSUE/SWON PSEA, which is an independent and separate local union, may validly disaffiliate from PAFLU pending the settlement of an election protest questioning its status as the sole and exclusive bargaining agent of PSI's rank and file employees

HELDYES. The pendency of an election protest involving both the mother federation and the local union did not constitute a bar to a valid disaffiliation.Reasoning In Liberty Cotton Mills Workers Union vs. Liberty Cotton Mills, Inc. the SC upheld the right of local unions to separate from their mother federation on the ground that as separate and voluntary associations, local unions do not owe their creation and existence to the national federation to which they are affiliated but, instead, to the will of their members. The sole essence of affiliation is to increase, by collective action, the common bargaining power of local unions for the effective enhancement and protection of their interests. Yet the local unions remain the basic units of association, free to serve their own interests subject to the restraints imposed by the constitution

and by-laws of the national federation, and free also to renounce the affiliation upon the terms laid down in the agreement which brought such affiliation into existence. Policy considerations dictate that in weighing the claims of a local union as against those of a national federation, those of the former must be preferred. Parenthetically though, the desires of the mother federation to protect its locals are not altogether to be shunned. It will however be to err greatly against the Constitution if the desires of the federation would be favored over those of its members. If it were otherwise, instead of protection, there would be disregard and neglect of the lowly workingmen.

Disposition Petition is granted.

Nature Relationship

FILIPINO PIPE AND FOUNDRY CORP V NLRC, NLU-TUCP, LERUM

00 SCRA 00PURISIMA; NOV 16, 1999

NATUREPetition for certiorari

FACTS- National Labor Union-Trade Union Congress of the Philippines (NLU-TUCP) filed w/ the Ministry of Labor and Employment, in behalf of its local chapter, the Filipino Pipe Workers Union-National Labor Union (FPWU-NLU), a notice of strike signed by its national president, Atty. Lerum, against Filipino Pipe and Foundry Corporation (Fil Pipe), alleging union busting and non-implementation of the Collective Bargaining Agreement.- A conciliation conference was set, but before the conciliation was done, the FPWU-NLU staged a strike.-Fil Pipe filed petition to declare the strike illegal and for damages against FPWU-NLU, NLU-TUCP and Lerum. Later, Fil Pipe moved to dismiss the complaint against FPWU-NLU.-Labor Arbiter declared the strike illegal, and ordered NLU-TUCP to pay damages. Lerum was absolved. Appeals were filed. -NLRC set aside decision for lack of merit in so far as NLU-TUCP and Lerum is concerned.-Filipino Pipe: NLRC erred in declaring that NLU-TUCP and Lerum are not primarily responsible, and that the company has lost its cause of action. They argued that the responsibility for the damages allegedly sustained by petitioner company on account of the illegal strike, should be borne by NLU-TUCP and Lerum, for having directly participated in aiding and abetting the illegal strike. Also, that FPWU-NLU is a mere agent of respondent NLU-TUCP, because FPWU-NLU, a local union, cannot act as the principal of respondent NLU-TUCP, a mother federation, because it is not a legitimate labor organization.

ISSUES1. WON the strike was illegal2. WON NLU-TUCP and Lerum are primarily responsible

HELD1. YES-1st, the strike staged by FPWU-NLU was baseless. 2nd, the union failed to serve the company a copy of the notice of strike (see Sec 3 Rule XXII, Book V, Implementing Rules). 3rd, the strike blatantly disregarded the prohibition on the doing of any act which may impede or disrupt the conciliation proceedings (see Sec 6)

2. NORatio The mother union, acting for and in behalf of its affiliate, had the status of an agent while the local union remained the basic unit of the association, free to serve the common interest of all its members subject only to the restraints imposed by the constitution and by-laws of the association. The same is true even if the local union is not a legitimate labor organization

Page 64: 33994637 Labor Rounds 1and2

Labor Law 2 A2010 - 64 - DisiniReasoning The direct and primary responsibility for the damages fall on the local union FPWU, being the principal, and not on respondent NLU-TUCP, a mere agent of FPWU-NLU which assisted the latter in filing the notice of strike. Being just an agent, the notice of strike filed by Lerum is deemed to have been filed by its principal, the FPWU-NLU. Having thus dismissed the claim for damages against the principal, FPWU-NLU, the action for damages against its agent, respondent NLU-TUCP, and Atty. Lerum, has no more leg to stand on and should also be dismissed.

Disposition Petition is dismissed.

Effect-Legal Personality

ADAMSON & ADAMSON V CIR (ADAMSON & ADAMSON SUPERVISORY UNION)

127 SCRA 268GUTIERREZ; January 31, 1984

FACTS- The Adamson & Adamson Inc. Supervisory Union (FFW) informed the petitioner Adamson & Adamson about its having organized on the same date that another union, the Adamson and Adamson Inc. Salesmen Association (FFW), also advised Adamson & Adamson that the rank and file salesmen had also formed their own union.- The CIR held that the Adamson and Adamson, Inc. Supervisory Union (FFW) can legally represent supervisors of the petitioner corporation notwithstanding the affiliation of the rank and file union of the same company with the same labor federation, the Federation of Free Workers.- Subsequently and during the pendency of the present petition, the rank and file employees formed their own union, naming it Adamson and Adamson Independent Workers (FFW).- The Adamson company argues that the affiliation of the respondent union of supervisors, the salesmen's association and the Adamson and Adamson Independent Worker's Union with the same national federation (FFW) violates Section 3 of the Industrial Peace Act because:

1) it results in the indirect affiliation of supervisors and rank-and-file employees with one labor organization,2) since respondent union and the unions of nonsupervisors in the same company are governed by the same constitution and by-laws of the national federation, in practical effect, there is but one union,3) it would result in the respondent union's losing its independence because it becomes the alter ego of the federation. Should affiliation be allowed, this would violate the requirement of separateness of bargaining units under Section 12 of the Act because only one union will in fact represent both supervisors and rank-and-file employees of the petitioner.

- The respondents on the other hand argue that the supervisory employees of an employer may validly join an organization of the rank-and-file employees so long as the said rank and file employees are not under their supervision.

ISSUE

1. WON Adamson and Adamson, Inc. Supervisory Union (FFW) has the legal personality to supervisors of the petitioner corporation notwithstanding the affiliation of the rank and file union of the same company with the same labor federation2. WON a supervisor's union may affiliate with a federation with which unions of rank-and-file employees of the same employer are also affiliated)

HELD1. YESRatio According to Elisco Elirol Labor Union vs. Nortel and Liberty Colton Mills Workers Union v. Liberty Cotton Mills. Inc, the local unions are separate and distinct units primarily designed to secure and maintain an

equality of bargaining power between the employer and their employee-members in the economic struggle for the fruits of joint productive effort of labor and capital; and the association of the locals into the national union was in furtherance of the same end.Reasoning - There is nothing in the provisions of the Industrial Peace Act which provides that a duly registered local union affiliating with a national union or federation loses its legal personality, or its independence.- Notwithstanding affiliation, the local union remained the basic unit free to serve the common interest of all its members.2. YESRatio The supervisory employees of an employer cannot join any labor organization of employees under their supervision but may validly form a separate organization of their own.Reasoning- The right of supervisors employees to organize under the Industrial Peace Act carries certain restrictions but the right itself may not be denied or unduly abridged. - In the case of Elisco Labot Union vs. Nortel: notwithstanding affiliation, the local union remained the basic unit free to serve the common interest of all its members.- The confusion seems to have stemmed from the prefix of FFW after the name of the local unions in the registration of both. - The inclusion of FFW in the registration is merely to stress that they are its affiliates at the time of registrations. It does not mean that said local unions cannot stand on their own. Neither can it be construed that their personalities arc so merged with the mother federation that for one difference or another they cannot pursue their own ways ways.Disposition Petition is dismissed.

2. Supervisor—Rank and File Union Affiliation

Rule-Affiliation

ADAMSON & ADAMSON V CIR (ADAMSON & ADAMSON SUPERVISORY UNION)

127 SCRA 268GUTIERREZ; January 31, 1984

FACTS- The Adamson & Adamson Inc. Supervisory Union (FFW) informed the petitioner Adamson & Adamson about its having organized on the same date that another union, the Adamson and Adamson Inc. Salesmen Association (FFW), also advised Adamson & Adamson that the rank and file salesmen had also formed their own union.- The CIR held that the Adamson and Adamson, Inc. Supervisory Union (FFW) can legally represent supervisors of the petitioner corporation notwithstanding the affiliation of the rank and file union of the same company with the same labor federation, the Federation of Free Workers.- Subsequently and during the pendency of the present petition, the rank and file employees formed their own union, naming it Adamson and Adamson Independent Workers (FFW).- The Adamson company argues that the affiliation of the respondent union of supervisors, the salesmen's association and the Adamson and Adamson Independent Worker's Union with the same national federation (FFW) violates Section 3 of the Industrial Peace Act because:

1) it results in the indirect affiliation of supervisors and rank-and-file employees with one labor

Page 65: 33994637 Labor Rounds 1and2

Labor Law 2 A2010 - 65 - Disiniorganization,2) since respondent union and the unions of nonsupervisors in the same company are governed by the same constitution and by-laws of the national federation, in practical effect, there is but one union,3) it would result in the respondent union's losing its independence because it becomes the alter ego of the federation. Should affiliation be allowed, this would violate the requirement of separateness of bargaining units under Section 12 of the Act because only one union will in fact represent both supervisors and rank-and-file employees of the petitioner.

- The respondents on the other hand argue that the supervisory employees of an employer may validly join an organization of the rank-and-file employees so long as the said rank and file employees are not under their supervision.

ISSUE

1. WON Adamson and Adamson, Inc. Supervisory Union (FFW) has the legal personality to supervisors of the petitioner corporation notwithstanding the affiliation of the rank and file union of the same company with the same labor federation2. WON a supervisor's union may affiliate with a federation with which unions of rank-and-file employees of the same employer are also affiliated)

HELD1. YESRatio According to Elisco Elirol Labor Union vs. Nortel and Liberty Colton Mills Workers Union v. Liberty Cotton Mills. Inc, the local unions are separate and distinct units primarily designed to secure and maintain an equality of bargaining power between the employer and their employee-members in the economic struggle for the fruits of joint productive effort of labor and capital; and the association of the locals into the national union was in furtherance of the same end.Reasoning - There is nothing in the provisions of the Industrial Peace Act which provides that a duly registered local union affiliating with a national union or federation loses its legal personality, or its independence.- Notwithstanding affiliation, the local union remained the basic unit free to serve the common interest of all its members.2. YESRatio The supervisory employees of an employer cannot join any labor organization of employees under their supervision but may validly form a separate organization of their own.Reasoning- The right of supervisors employees to organize under the Industrial Peace Act carries certain restrictions but the right itself may not be denied or unduly abridged. - In the case of Elisco Labot Union vs. Nortel: notwithstanding affiliation, the local union remained the basic unit free to serve the common interest of all its members.- The confusion seems to have stemmed from the prefix of FFW after the name of the local unions in the registration of both. - The inclusion of FFW in the registration is merely to stress that they are its affiliates at the time of registrations. It does not mean that said local unions cannot stand on their own. Neither can it be construed that their personalities arc so merged with the mother federation that for one difference or another they cannot pursue their own ways ways.Disposition Petition is dismissed.

ATLAS LITHOGRAPHIC SERVICES, INC. V LAGUESMA205 SCRA 12

GUTIERREZ; January 6, 1992

NATURE Petition for certiorari

FACTS

- The supervisory, administrative personnel, production, accounting and confidential employees of the petitioner Atlas Lithographic Services, Inc. (ALSI) affiliated with private respondent Kaisahan ng Manggagawang Pilipino, a national labor organization. - The local union adopted the name Atlas Lithographic Services, Inc. Supervisory, Administrative, Personnel, Production, Accounting and Confidential Employees Association or ALSI-SAPPACEA-KAMPIL - Respondent Kampil-Katipunan filed on behalf of the "supervisors" union a petition for certification election so that it could be the sole and exclusive bargaining agent of the supervisory employees.- The petitioners opposed the private respondent's petition claiming that under Article 245 of the Labor Code the respondent cannot represent the supervisory employees for collective bargaining purposeless because the private respondent also represents the rank-and-file employees' union.- The Med-Arbiter issued an order in favor of the private respondent- The petitioners appealed for the reversal of the order. The public respondent, however, issued a resolution affirming the Med-Arbiter's order.- The petitioners, in turn, filed a motion for reconsideration but the same was denied. - Hence, this petition for certiorari.

ISSUE WON under Article 245 of the Labor Code, a local union of supervisory employees may be allowed to affiliate with a national federation of labor organizations of rank-and-file employees and which national federation actively represents its affiliates in collective bargaining negotiations with the same employer of the supervisors and in the implementation of resulting collective bargaining agreements.

HELDNO. - The interests of supervisors on the one hand, and the rank-and-file employees on the other, are separate and distinct. The functions of supervisors, being recommendatory in nature, are more identified with the interests of the employer. The performance of those functions may, thus, run counter to the interests of the rank-and-file.- The peculiar role of supervisors is such that while they are not managers, when they recommend action implementing management policy or ask for the discipline or dismissal of subordinates, they identify with the interests of the employer and may act contrary to the interests of the rank-and-file.- The Court agreed with the petitioner's contention that a conflict of interest may arise in the areas of discipline, collective bargaining and strikes.- Members of the supervisory union might refuse to carry out disciplinary measures against their co-member rank-and-file employees. In the area of bargaining, their interests cannot be considered identical. The needs of one are different from those of the other. Moreover, in the event of a strike, the national federation might influence the supervisors' union to conduct a sympathy strike on the sole basis of affiliation.- The factual issues in the Adamson case are different from the present case. First, the rank-and-file employees in the Adamson case are not directly under the supervisors who comprise the supervisors' union. In the case at bar, the rank-and file employees are directly under the supervisors organized by one and the same federation. Second, the national union in the Adamson case did not actively represent its local chapters. In the present case, the local union is actively represented by the national federation. In fact, it was the national federation, the KAMPIL-KATIPUNAN, which initially filed a petition for certification in behalf of the respondent union.- If the intent of the law is to avoid a situation where supervisors would merge with the rank and-file or where the supervisors' labor organization would represent conflicting interests, then a local supervisors' union should not be allowed to affiliate with the national federation of union of rank-and-file employees where that federation actively participates in union activity in the company.- The prohibition against a supervisors' union joining a local union of rank-and-file is replete with jurisprudence. The Court emphasizes that the limitation is not confined to a case of supervisors wanting to

Page 66: 33994637 Labor Rounds 1and2

Labor Law 2 A2010 - 66 - Disinijoin a rank-and-file local union. The prohibition extends to a supervisors' local union applying for membership in a national federation the members of which include local unions of rank-and-file employees. The intent of the law is clear especially where, as in the case at bar, the supervisors will be co-mingling with those employees whom they directly supervise in their own bargaining unit.- Supervisors are not prohibited from forming their own union. What the law prohibits is their membership in a labor organization of rank-and-file employees or their joining a national federation of rank-and-file employees that includes the very local union which they are not allowed to directly join.Disposition Petition GRANTED.

COLLEGE OF MEDICINE V LAGUESMA249 SCRA

MENDOZA; August 12, 1998

NATUREPetition for certiorari

FACTS-DLSUMCCM is a hospital and medical school at Dasmariñas, Cavite.-Private respondent Federation of Free Workers-DLSUMCCM Supervisory Union Chapter, on the other hand, is a labor organization composed of the supervisory employees of petitioner DLSUMCCM. FFW-DLSUMCCMSUC is a recognized local chapter of FFW. The latter filed on behalf of the former a petition for certification election among the supervisory employees of DLSUMCCM. -This petition was opposed by DLSUMCCM on the grounds that several employees who signed the petition for certification election were managerial employees and that the FFW-DLSUMCCMSUC was composed of both supervisory and rank-and-file employees in the company.-DOLE Regioin IV med-arbiter issued an order granting union's petition for certification election. DOLE USec Laguesma affirmed and denied MFR. Hence, this petition for certiorari.

ISSUEWON unions formed independently by supervisory and rank-and-file employees of a company may validly affiliate with the same national federation.

HELDYES.-Supervisory employees have the right to self-organization as do other classes of employees save only managerial ones. The framers of the Constitution intended to restore the right of supervisory employees to self-organization which had been withdrawn from them during the period of martial law. Thus:-Conformably with the constitutional mandate, Art. 245 of the Labor Code now provides for the right of supervisory employees to self-organization, subject to the limitation that they cannot join an organization of rank-and-file employees. The reason for the segregation of supervisory and rank-and-file employees of a company with respect to the exercise of the right to self-organization is the difference in their interests. Supervisory employees are more closely identified with the employer than with the rank-and-file employees. If supervisory and rank-and-file employees in a company are allowed to form a single union, the conflicting interests of these groups impair their relationship and adversely affect discipline, collective bargaining and strikes. These consequences can obtain not only in cases where supervisory and rank-and-file employees in the same company belong to a single union but also where unions formed independently by supervisory and rank-and-file employees of a company are allowed to affiliate with the same national federation. -Atlas Lithographic Services Inc. v. Laguesma: To avoid a situation where supervisors would merge with the rank-and-file or where the supervisors' labor organization would represent conflicting interests, then a local supervisors' union should not be allowed to affiliate with a national federation of unions of rank-and-file employees where that federation actively participates in union activities in the company. Such a situation

would obtain only where two conditions concur: First, the rank-and-file employees are directly under the authority of supervisory employees. Second, the national federation is actively involved in union activities in the company.-The affiliation of two local unions in a company with the same national federation is not by itself a negation of their independence since in relation to the employer, the local unions are considered as the principals, while the federation is deemed to be merely their agent. This conclusion is in accord with the policy that any limitation on the exercise by employees of the right to self-organization guaranteed in the Constitution must be construed strictly. Workers should be allowed the practice of this freedom to the extent recognized in the fundamental law.

Q: WON the rank-and-file employees of petitioner DLSUMCCM who compose a labor union are directly under the supervisory employees whose own union is affiliated with the same national federation (Federation of Free Workers) and whether such national federation is actively involved in union activities in the company so as to make the two unions in the same company, in reality, just one union.Ans: NO. Although FFW-DLSUMCCMSUC and another union composed of rank-and-file employees of petitioner DLSUMCCM are indeed affiliated with the same national federation, the FFW, petitioner DLSUMCCM has not presented any evidence showing that the rank-and-file employees composing the other union are directly under the authority of the supervisory employees.-Adamson & Adamson, Inc. v. CIR: the fact that the two groups of workers are employed by the same company and the fact that they are affiliated with a common national federation are not sufficient to justify the conclusion that their organizations are actually just one. Their immediate professional relationship must be established.Disposition Petition dismissed

3. Local Union DisaffiliationNature Right Disaffiliation

VOLKSHCEL LABOR UNION V BUREAU OF LABOR RELATIONS137 SCRA 42

PHIL. LABOR ALLIANCE COUNCIL V BLR75 SCRA 162

MALAYANG SAMAHAN NG MGA MANGGAGAWA SA GREENFIELD V RAMOS

326 SCRA 428

Rule-Legality Act-Disaffiliation

VILLAR V INCIONG121 SCRA 44

GUERRERO; April 20, 1983

NATUREPetition for review by certiorari

FACTS- Petitioners were members of the Amigo Employees Union-PAFLU, a duly registered labor organization which was the existing bargaining agent of the employees in private respondent Amigo Manufacturing, Inc. They then signed a joint resolution which stated

Page 67: 33994637 Labor Rounds 1and2

Labor Law 2 A2010 - 67 - Disinithat they were disaffiliating themselves as members of the PAFLU, and were not further authorizing PAFLU to represent them in any CBA.- During a special meeting of the Union, a Resolution was approved which called for the investigation of all the petitioners for "continuously maligning, libelling and slandering not only the incumbent officers but even the union itself and the federation" and for causing divisiveness.- The union security clause was reincorporated in the new CBA: “any members who shall resign, be expelled, or shall in any manner cease to be a member of the UNION, shall be dismissed from his employment upon written request of the UNION to the Company.”- Petitioners contend that their acts do not constitute disloyalty as these are in the exercise of their constitutional right to self-organization.- The PAFLU President then rendered a decision finding them guilty of the charges and expelling them from the Union and as a consequence the Management of the employer is hereby requested to terminate them from their employment in conformity with the security clause.

ISSUE/S1. WON PAFLU had the authority to investigate the petitioners and, thereafter, expel them from the roll of membership of the Amigo Employees Union-PAFLU2. WON the petitioners were entitled to disaffiliate from the Union

HELD1.YESRatio That PAFLU had the authority to investigate petitioners on the charges filed by their co-employees in the local union and after finding them guilty as charged, to expel them from the roll of membership of the Amigo Employees Union-PAFLU is clear under the constitution of the PAFLU to which the local union was affiliated. And pursuant to the security clause of the new CBA, reiterating the same clause in the old CBA, PAFLU was justified in applying said security clause.UNION-MEMBER RELATIONS: Discipline IssuesInherent in every labor union, or any organization for that matter, is the right of self-preservation. When members of a labor union, therefore, sow the seeds of dissension and strife within the union; when they seek the disintegration and destruction of the very union to which they belong, they thereby forfeit their rights to remain as members of the union which they seek to destroy. Prudence and equity, as well as the dictates of law and justice, therefore, compelling mandate the adoption by the labor union of such corrective and remedial measures in keeping with its laws and regulations, for its preservation and continued existence; lest by its folly and inaction, the labor union crumble and fall. 2. YESRatio Although they are entitled to disaffiliate from their union and form a new organization of their own, they must, however, suffer the consequences of their separation from the union under the security clause of the CBA.Reasoning Disaffiliation from a labor union is not open to legal objection. It is implicit in the freedom of association ordained by the Constitution. But this Court has laid down the ruling that a closed shop is a valid form of union security, and such provision in a collective bargaining agreement is not a restriction of the right of freedom of association guaranteed by the Constitution.Local Union Disaffiliation: Rule - Legality act - DisaffiliationThere are two hundred thirty four (234) union members in the Amigo Employees Union-PAFLU, and only 96 signed the "Sama-Samang Kapasiyahan." They constituted a small minority for which reason they could not have successfully disaffiliated the local union from PAFLU. It can be inferred that the majority wanted the union to remain an affiliate of PAFLU. The action of the majority must, therefore, prevail over that of the minority members.

Disposition The Order appealed from affirming the joint decision of the OIC granting clearance to terminate petitioners as well as dismissing their complaint with application for preliminary injunction, is hereby AFFIRMED.

TROPICAL HUT EMPLOYEES UNION V TROPICAL HUT FOOD MARKET, INC

181 SCRA 173

ALEX FERRER V NLRC

224 SCRA 410

PHIL SKYLANDERS INC V NLRC (PAFLU)375 SCRA 369

BELLOSILLO; Jan 31, 2002

NATUREPetition for certiorari

FACTS383 Nov 1993, the Philippine Skylanders Employees Association

(PSEA), a local labor union affiliated with the Philippine Association of Free Labor Unions (PAFLU) September (PAFLU), won in the certification election conducted among the rank and file employees of Philippine Skylanders, Inc. (PSI). Its rival union, Philippine Skylanders Employees Association-WATU (PSEA-WATU) immediately protested the result of the election before the Secretary of Labor.

384 Several months later, pending settlement of the controversy, PSEA sent PAFLU a notice of disaffiliation citing as reason PAFLU's supposed deliberate and habitual dereliction of duty toward its members.

385 PSEA subsequently affiliated itself with the National Congress of Workers (NCW), changed its name to Philippine Skylanders Employees Association - National Congress of Workers (PSEA-NCW), and allowed the former officers of PSEA-PAFLU to continue occupying their positions as elected officers PSEA-NCW.

386 On 17 March 1994 PSEA-NCW entered into a collective bargaining agreement with PSI which was immediately registered with DOLE.

387 PAFLU Secretary General Serafin Ayroso wrote Mariles C. Romulo requesting a copy of PSI's audited financial statement. Ayroso explained that with the dismissal of PSEA-WATU's election protest the time was ripe for the parties to enter into a collective bargaining agreement. PSI through its personnel manager Francisco Dakila denied the request citing as reason PSEA's disaffiliation from PAFLU and its subsequent affiliation with NCW.

388 PAFLU through Serafin Ayroso filed a complaint for unfair labor practice against PSI, its president Mariles Romulo and personnel manager Francisco Dakila. PAFLU amended its complaint by including the elected officers of PSEA-PAFLU as additional party respondents.

389 Labor Arbiter declared PSEA's disaffiliation from PAFLU invalid and held PSI, PSEA-PAFLU and their respective officers guilty of unfair labor practice.

390 NLRC upheld the Decision of the Labor Arbiter and conjectured that since an election protest questioning PSEA-PAFLU's certification as the sole and exclusive bargaining agent was pending resolution before the Secretary of Labor, PSEA could not validly separate from PAFLU, join another national federation and subsequently enter into a collective bargaining agreement with its employer-company

ISSUE/SWON PSEA, which is an independent and separate local union, may validly disaffiliate from PAFLU pending the settlement of an election protest questioning its status as the sole and exclusive bargaining agent of PSI's rank and file employees

HELDYES. The pendency of an election protest involving both the mother federation and the local union did not constitute a bar to a valid disaffiliation.Reasoning In Liberty Cotton Mills Workers Union vs. Liberty Cotton Mills, Inc. the SC upheld the right of local unions to separate from their mother federation on the ground that as separate and voluntary associations, local unions do not owe their creation and existence to the national federation to which they are affiliated but, instead, to the will of their members. The sole essence of affiliation is to increase, by collective action, the common bargaining power of local unions for the effective enhancement and protection of their interests. Yet the local unions remain the basic units of association, free to serve their own interests subject to the restraints imposed by the

Page 68: 33994637 Labor Rounds 1and2

Labor Law 2 A2010 - 68 - Disiniconstitution and by-laws of the national federation, and free also to renounce the affiliation upon the terms laid down in the agreement which brought such affiliation into existence. Policy considerations dictate that in weighing the claims of a local union as against those of a national federation, those of the former must be preferred. Parenthetically though, the desires of the mother federation to protect its locals are not altogether to be shunned. It will however be to err greatly against the Constitution if the desires of the federation would be favored over those of its members. If it were otherwise, instead of protection, there would be disregard and neglect of the lowly workingmen.

Disposition Petition is granted.